Download Neet PG Surgery MCQs

Download Neet PG Surgery MCQs Question Bank

1.Post-transplantationhypertensioncanbe causedby:I.Rejection.II.Cyclosporinenephrotoxicity.III.Renaltransplantarterystenosis(RTAS).IV.Recurrentdiseaseintheallograft. a)I,II,III,IVarecorrect b)I,II,IVarecorrect. c)I&IIIarecorrect d)Noneoftheaboveiscorrect. CorrectAnswer-ABothacuteandchronicrejectionmayresultinhypertension.Theformercausesacutefluidretentionandpluggingofperitubularcapillarieswithinflammatorycells.Thismayprogresstointimalswellingandmedialnecrosisandeventuateinischemiasecondarytoendothelialproliferationandobliterationofsmallvessels.Chronicrejection,thoughttoberelatedtoprotractedhumoralinjury,resultsinobliterationofcapillariesviathedevelopmentofintimalhyperplasia.Cyclosporinehasavasoconstrictiveeffectwhich,throughactivationoftherenin-angiotensinsystem,mayleadtohypertension.RTASisresponsibleforhypertensionin4%to12%ofrenalallograftrecipients.Itrespondswelltopercutaneousangioplasty.Acarefultrialofangiotensin-convertingenzymeinhibitorsmaybediagnosticofRTAS.Recurrentdiseasesuchasmembranoproliferativeglomerulonephritisandfocalglomerularsclerosismayresultinsignificanthypertensioninrenalallograftrecipients.
2.Mostcommoncauseofsmallintestine obstructionis? a)Intussception b)IdiopathicadhesionsTumors c)Tumors d)Postoperativeadhesions CorrectAnswer-DPostoperativeadhesionsREF:Bailey&Love25theditionpage1188,http://emedicine.medscape.com/article/774140overview"Themostcommoncauseofsmall-bowelobstruction(SBO)ispostsurgicaladhesions"Themostcommoncausesofintestinalobstructioninadultsare:Intestinaladhesions--bandsoffibroustissueintheabdominalcavitythatcanformafterabdominalorpelvicsurgeryInchildren,themostcommoncauseofintestinalobstructionistelescopingoftheintestine(intussusception).
3.Bestmethodtotreatalargeport-wine hemangiomais? a)Radiotherapy b)Tatooing c)Excisionwithskingrafting d)PulseddyeLaser CorrectAnswer-DPulseddyeLaserREF:Roxburgh'scommonSkinDiseases17theditionpage194&205,ClinicalDermatologybyJohnHunter,JohnSavin&MarkDahl3'editionpage275"SelectivePhotothermolysisorPulseddyelaseristhetreatmentofchoiceforPortwinehemangioma"Port-winestainsarecrimsonblotchesinwhichthereismarkedcapillarydilatationcomparedtoacapillaryangioma,whichisarednoduleorplaquecontainingproliferatingendothelialcells.Thelattertendtoflattenanddisappearatpuberty.Largeronesmaycauseproblemsfrombleedingand/orerosion.Cavernoushaemangiomataarelargerandcompressible,containinglargevascularspaces.Occasionallyaport-winestainofthetrigeminalareaisassociatedwithavascularmalformationoftheleptomeningesonthesameside,whichmaycauseepilepsyorhemiparesis(theSturgeWebersyndrome),orwithglaucoma.Excellentresultshavebeenobtainedwithcarefulandtime-consumingtreatmentwitha585-nmflashlamp-pumpedpulseddyelaser.Treatmentsessionscanbegininbabiesandanaesthesiaisnotalwaysnecessary.Ifatrialpatchissatisfactory,40-50pulsescanbedeliveredinasessionandtheprocedurecanberepeatedat3-monthlyintervals.

4.Cock'speculiartumoris a)BasalcellCA b)SquamouscellCA c)Ulceratedsebaceouscyst d)Cylindroma CorrectAnswer-CUlceratedsebaceouscyst[Ref.Love&Bailey23/epage173,595;DastextbookofSurgery3/ep81]RepeatfromMay04Cock'speculiartumourisasebaceouscystlinkedgrowththatcanresembleasquamouscellcarcinoma.Theproliferatingcystisusuallysolitary,butitoftenarisesfromasimpletrichilemmalcystsinthehairfollicleepithelium.
5.FalseaboutGastriclymphomais: a)Stomachisthemostcommonsite b)AsosciatewithH.pyloriinfection c)Totalgastrectomywithadjuvantchemotherapyisthetreatment ofchoice d)5yrsurvivalrateaftertreatmentis60%. CorrectAnswer-CTotalgastrectomywithadjuvantchemotherapyisthetreatmentofchoice[Ref:Sabiston18/ep1269(17/cp1312);Harrison17/e,p573;Schwartz9/ep935(8/e,p981)]Stomachisthemostcommonextranodalsiteoflymphoma.Itisoftwotypes-Primary&SecondaryPrimarygastriclymphoma-islymphomaofthestomachwhichexhibitsnoevidenceofliver,spleen,mediastinallymphnodesorbonemarrowinvolvementatthetimeofdiagnosis(regionallymphnodeinvolvementmaybepresent)Primarygastriclymphomaismainly(>95%)Non-Hodgkin'slymphomaofBcellorigin.Mostoftheprimarylymphomas(about60%)ariseinMALT(mucosaassociatedlymphoidtissue).MALTisusuallyassociatedwithchronicH.pyloriinfection.Solow-gradeMALTlymphomasarethoughttoarisebecauseofchronicH.pyloriinfection.TheselowgradelymphomasmaylaterondegeneratetohighgradelymphomasImmunodeficiencyandH.pyloriinfectionareriskfactorsforBcelllymphoma.Mostcommonsiteofinvolvement(likethatofadenocarcinoma)isgastricantrum.
SecondarygastriclymphomaStomachisthemostcommonextranodalsiteofsystemiclymphoma.AlmostallofthemareNon-HodgkinstypesTreatmentTheroleofgastricresectioniscontroversial.Recentstudieshaveshownsimilardiseasefree5yearsurvivalratesinpatientstreatedwithsurgery+chemotherapy+radiotherapyandpatientstreatedwithchemotherapy+radiationtherapyalone.(Mostpatientswithhighgradegastriclymphomasarenowtreatedwithchemoradiationalone,withoutsurgicalresection.Fordiseaselimitedtothestomachandregionalnodes,radicalsubtotalgastrectomymaybeperformed,especiallyforbulkytumorswithbleedingand/orobstruction.)Mostcommonchemotherapeuticregimenis?CHOPplusrituximab[CHOPstandsforcyclophosphamide,doxorubicin,vincristine,&prednisone]aboutoptiondi.e.(5yrsurvivalrateaftertreatmentis60%.)Harrison17/ewrites-"Subtotalgastrectomy,usuallyfollowedbycombinationchemotherapy,hasledto5-yearsurvivalratesof40-60%inpatientswithlocalizedhigh-gradelymphomas."
6.Whichoneofthefollowingpreservativeisusedwhilepackingcatgutsuture? a)Isopropylalcohol b)Colloidaliodine c)Glutaraldehyde d)Hydrogenperoxide CorrectAnswer-A10%isopropylalcoholistheroutinepackingfluidusedinpackingcatgutsutures.Catgutisavailableinglasstubescontainingisopropylalcoholandsmallquantityofwater.Catgutispreparedfromtheintestineofsheep.Ref:PharmaceuticsByDr.R.S.Gaud,Page196;VeterinaryMedicine/smallanimalclinician,Volume72,Issues1-6,Page835.
7.Primaryhyperparathyroidism,phaeochromocytomamaybeassociatedwith whichtypeofthyroidcancer? a)Medullarycarcinomaofthethyroid b)Papillarycarcinomaofthethyroid c)Anaplasticcarcinomaofthethyroid d)Follicularcarcinomaofthethyroid CorrectAnswer-AItischaracterizedclinicallybymedullarythyroidcarcinoma(MTC),pheochromocytoma,andhyperparathyroidism.Approximately70-95%ofindividualswithMEN2AdevelopMTC,50%developpheochromocytoma,and15-30%develophyperparathyroidism. Ref:BaileyandLove'sShortPracticeofSurgery,24thEdition,Page802;HarissonsInternalMedicine,18thEdition,Chapter351.
8.Thetreatmentofchoiceforsquamouscellanalcancer? a)Laserfulgaration b)Chemoradiotherapy c)Abdominoperennialresection d)Platinum-basedchemotherapy CorrectAnswer-BSquamouscellanalcancerisrarebutassociatedwithHumanpapillomavirusinfection(HPV),Analintra-epithelialneoplasia(AIN)andimmunosuppression.Thecurrenttreatmentofanalcanalcarcinomaischemoradiotherapy(combinedmodalitytherapy).Chemotherapyincludeacombinationof5-FUwithmitomycinCorCisplatin.Ref:BaileyandLoveShortPracticeofSurgery,25thEdition,Page1269;TheMDAndersonManualofMedicalOncology,2ndEdition,Chapter22.
9.Meconiumileusisassociatedwith: a)Cysticfibrosis b)Infantofdiabeticmother c)Hypothyroidism d)Noneoftheabove CorrectAnswer-AInfantswithcysticfibrosishavecharacteristicpancreaticenzymedeficienciesandabnormalchloridesecretionintheintestinethatresultintheproductionofviscous,water-poormeconium. Meconiumileusoccurswhenthisthick,highlyviscousmeconiumbecomesimpactedintheileumandleadstohigh-gradeintestinalobstruction.Ref:Schwartz'sprincipleofsurgery9thedition,chapter39.
10.Commonesttypeofanalcanalcarcinomais? a)Squamouscellcarcinoma b)Adenocarcinoma c)Adenoacanthoma d)Papillarytype CorrectAnswer-ASquamouscellcarcinomaisthemostcommontypeofanalcancer.Thiscancerbeginsintheouterliningoftheanalcanal.Mainlythreetypesofmalignantneoplasmareseeninanalcanal Squamouscellcarcinoma(mostcommon) Basalcellcarcinoma(2ndmostcommon) MelanomaAlsoremember:MostcommontypeofrectalandcoloncancerAdenocarcinomaRef:S.Dastextbookofsurgery3/e,Page1078.
11.WhichofthefollowingnerveiscommonlydamagedduringMcBurney's incision? a)Subcostalnerve b)10ththoracicnerve c)11ththoracicnerve d)Iliohypogastricnerve CorrectAnswer-D ThenervecommonlydamagedduringMcBurney'sincisionisIliohypogastricNerve.Damagetothisnerveresultinthedevelopmentofrightinguinalhernia. McBurney'sincision/gridironincisioniscommonlydoneforappendectomy.Inthisanobliqueincisionismadeintherightiliacfossaabout5cmaboveandmedialtotheanteriorsuperioriliacspineatrightangletothespinoumbilicalline.Ref:AnatomyofAbdomenandLowerLimbBySingh,Page46.
12.Dysphagialusoriaisdueto? a)Esophagealdiverticulum b)Aneurysmofaorta c)Esophagealweb d)Compressionbyaberrantbloodvessel CorrectAnswer-DDysphagialusoriaisadisorderofswallowingcausedduetovascularanomaliesandincludes: Arightaorticarch Adoubleaorticarch AvascularconstrictingringformedbyaPDAoraligamentumarteriosumandpulmonary arteryoraorticarch Anabnormalrightsubclavianartery AnabnormalinnominatearteryDiagnosisismadebylipoidalswalloworarteriography. Ref:BaileyandLove24/e,Page995
13.Allofthefollowingarecausesofhemobilia,EXCEPT: a)TraumatoAbdomen b)Malignancy c)Ruptureofhepaticarteryaneurysm d)Hepatitis CorrectAnswer-DHemobiliapresentswiththetriadofbiliarycolic,obstructivejaundice,andoccultorgrossintestinalbleeding.Causesare, Hepatictrauma Ductalparasitism(Ascarislumbricoides) Orientalcholangiohepatitis Hepaticneoplasms Ruptureofahepaticarteryaneurysm Hepaticabscess CholedocholithiasisThediagnosismaybesuspectedfromatechnetium-99m-labeledredbloodcellscan,butanarteriogramisusuallyrequiredfordiagnosisandplanningoftherapy.Ref:DohertyG.M.(2010).Chapter25.BiliaryTract.InG.M.Doherty(Ed),CURRENTDiagnosis&Treatment:Surgery,13e.
14.Linitisplasticaisatypeof: a)Gastriculcer b)Castomach c)Duodenalulcer d)Noneoftheabove CorrectAnswer-BLinitisplasticaisatypeofadenocarcinoma.Adenocarcinomaisthemostcommonformofstomachcancer.Linitisplasticaspreadstothemusclesofthestomachwallandmakesitthickerandmorerigid.Linitisplasticaisasubtypeofgastriccancerthatischaracterizedbydiffuseinfiltratingadenocarcinomawithoutobviouscartersandulcers. Itisthoughttooriginatefromparietalcellportionofgastricmucosa. Becauseofitsdiffusenature,thisformofgastriccancerususallyinvolvesthewholestomach.Ref:ClinicalScenariosinSurgicalOncologyeditedbyVijayP.Khatri
15.Surgeryforundescendedtestisisrecommendedatwhatage? a)6months b)12months c)24months d)36months CorrectAnswer-AOptimumageforsurgeryincaseofundescendedtestisis6months. Thesurgicalproceduredoneiscalledorchidopexy. Earlierintervention(Sixmonthsofage)shouldbeconsideredinordertotheoreticallypreventthecomplicationsofcrytorchidismthatmaybemanifestedbefore1yearofage.Ref:NursingcareofthepediatricsurgicalpatientByNancyTkaczBrowne,Page147-8;CPDT,20thEdition,Page974;Campbell'sUrology,9thEdition,Page3775
16.Apatientwhohasfallenoveraloosemanholecoverisbroughttotheclinic. Onexaminationaperinealhematomaandbloodintheexternalmeatusisnoted.Ruptureofwhichofthefollowingstructurehasresultedinperinealhematoma? a)Ruptureofmembranousurethra b)Ruptureofbulbarurethra c)Pelvicorganblunttrauma d)Ruptureofbladder CorrectAnswer-BThebulbarurethraiscrushedupwardsontopubicbone,typicallywithsignificantbruising.Cyclingaccidents,loosemanholecoverandgymnasiumaccidentsastridethebeamaccountfornumbrofcases.CLINICALFEATURESThesignsofarupturedbulbarurethraareperinealbruisingandhaematoma,typicallywithabutterflydistribution'Thereisusuallybleedingiromtheurethralmeatusandretenrionofurineisalsotypicallypresent.Ruptureofthemembranousurethratypicallyoccursinassociationwithafracturedpelvisandmaybeassociatedwithanextraperitonealruptureofthebladder.Themostcommoncausesofpelvicfractureareroadtrafficaccidents,severecrushinjuriesandfalls.Theclinicalfearuresincludeurinaryretention,bloodattheurethralmeatus.Thereistypicallymarkedbruisingofthepubicarea,scrorumandpenis.
17.Commonestcarcinomathatcancausesplenicmetastasisiswhichofthe following? a)Ca.Pancreas b)Ca.Stomach c)Ca.Ovary d)Ca.Cervix CorrectAnswer-CAlthoughisolatedmetastasistospleenisrare,studiesfoundthemostcommonprimaryneoplasmswithsplenicmetastasistobegynecologic(61%),withmajoritybeingovarian,colorectal(15%),lung(9%),andstomach(4%).Ref:CTandMRIoftheAbdomenandPelvis:ATeachingFileeditedbyPabloR.Ros,KoenraadJ.Mortele,2006,Page218.
18.Spleniculiareseenmostcommonlyin: a)Colon b)Hilum c)Liver d)Lungs CorrectAnswer-BSpleniculioraccessoryspleensaremostcommonly(about80%)seenatthehilumofthespleen.Alsoknow:Locationofaccessoryspleenindecreasingorderare: Splenichilum Gastrocolicligament Tailofpancreas Greateromentum Greatercurvatureofstomach Splenocolicligament Smallandlargebowelmesentery Leftbroadligamentofwomen Leftspermaticcordinmen Ref:Schwartz9/e,Page1246;Bailey&Love25/e,Page25/e,Page1103
19.WhichofthefollowingstageofBreastCacorrespondswithfollowingfeature? Breastmassof6x3cm.sizeIpsilateralsupraclavicularlymphnodeDistantmetastasiscannotbeassessed a)T4N3MX b)T4N1M1 c)T4N0M0 d)T3N3cMX CorrectAnswer-DAccordingtoTNMstagingsystemforbreastcancer,T3:Tumor>5cmingreatestdimensionN3c:Metastasisinipsilateralsupraclavicularlymphnode(s)MX:DistantmetastasiscannotbeassessedRef:HuntK.K.,NewmanL.A.,CopelandE.M.,BlandK.I.(2010).Chapter17.TheBreast.InF.C.Brunicardi,D.K.Andersen,T.R.Billiar,D.L.Dunn,J.G.Hunter,J.B.Matthews,R.E.Pollock(Eds),Schwartz'sPrinciplesofSurgery,9e.
20.Dacronvasculargraftisa: a)Textilebiologic b)Textilesynthetic c)Nontextilebiologic d)Nontextilesynthetic CorrectAnswer-BDacrongraftisatypeoftextilesyntheticgraft.Itisassociatedwithincreasedriskofthrombosisduetolowflowrates.Theyareprecoatedwithcollagen,whicheliminatestheneedforpre-clotting.Itisnowreservedforaorticandhighpressure,largediameterbypassgraftsorusedasaorticendografts.Ref:DiabeticFoot:LowerExtremityArterialDiseaseandLimbSalvage,AntonN.Sidawy,2006Edition,Chapter22,Page234;MasteryofVascularandEndovascularSurgeryByGeraldB.Zelenock,2006Edition,Chapter50,Page414;VascularAccess:PrinciplesandPracticeBySamuelEricWilson,5thEdition,Page115;VascularSurgeryByAlunH.Davies,Page135.
21.TheMOSTcommontypeofdislocationofelbowjointis: a)Posterior b)Posterolateral c)Posteromedial d)Lateral CorrectAnswer-AInadults,theelbowisthesecondmostfrequentlydislocatedmajorjoint,aftertheshoulder.Itisthemostcommonlydislocatedjointinchildren.Morethan90%ofallelbowdislocationsareposteriordislocations. Thisinjuryentailsdisengagementofthecoronoidprocessoftheulnafromthetrochleaofthehumeruswithmovementposteriorly.Themechanismofinjuryistypicallyafallontoanoutstretchedhandwiththeelbowinextensionuponimpact.
22.Whichofthefollowinggasismostcommonlyusedinlaproscopy? a)CO2 b)N2O c)O2 d)Helium CorrectAnswer-ACO2isusedinlaproscopybecauseitiscommontothehumanbodyandcanbeabsorbedbytissueandremovedbytherespiratorysystem.Itisalsonon-flammable,whichisimportantbecauseelectrosurgicaldevicesarecommonlyusedinlaparoscopicprocedures.
23.Whatisthemostcommonpositionofappendix? a)Pelvic b)Paracolic c)Retrocaecal d)Retroperitoneal CorrectAnswer-CRetrocecalisthemostcommonpositionoftheappendix.Positionsofappendix:Retrocecal:74%Subcecal:1.5%Pelvic:21%Post-ileal:0.5%Pre-ileal/retro-ileal:1%
24.Mostcommonprimarybonetumourofhandis: a)Osteoma b)Osteochondroma c)Enchondroma d)Noneoftheabove CorrectAnswer-CEnchondromasarisefromcartilageandarethemostcommonprimarybonetumorsofthehand.Theselesionsaccountfor>90%ofbonetumorsseeninthehand. Ref:Schwartz'sprincipleofsurgery9thedition,chapter44.Commonestbonemalignancies->Secondaries(metastases)Commonestprimarymalignanttumor>MultiplemyelomaCommonestprimarymalignanttumorolongbonesOsteosarcomaCommonestbenigntumorofbonne-Osteochondroma(Osteochondromaisnottureneoplasmsinceitsgrowthstopswithcessationofgrowthattheepiphysealplate)Commonesttruebenigntumorofbone->OsteroidosteomaCommonestbenigntumorofhand-Enchondroma
25.
WhichofthefollowingVitamindeficiencyisseeninshortbowelsyndromewithilealresection? a)VitaminK b)VitaminB12 c)VitaminB1 d)Folicacid CorrectAnswer-BAnswer-B(VitaminB12)Short-bowelsyndromeisadisorderclinicallydefinedbymalabsorption,diarrhea,steatorrhea,fluidandelectrolytedisturbances,andmalnutrition.ChangesseeninTerminalIlealResectionMalabsorptionofbilesaltsandvitaminB12(whicharenormallyabsorbedinthisregion)VitaminB12MalabsorptionMegaloblasticanemiaBilesaltsMalabsorptionUnabsorbedbilesaltsescapeintothecolonandstimulatefluidsecretionfromthecolonwaterydiarrheaDecreasedbilesaltsinthebileCholesterolgallstones
26.Doublebubblesignseenin: a)Duodenalatresia b)Duodenalstenosis c)Volvulus d)All CorrectAnswer-DAns.isD'a','b'&'c'i.e.Duodenalatresia,DuodenalstenosisandVolvulusDoublebubblesignisseeninduodenalatresia,duodenalweb,deudenalstenosis,Ladd'sband,Annularpancreas,Malrotationofgut,preduodenalvein.Distalgasismoreoftenseenwithmidgutvolvulus,duodenalstenosisandduodenalweb
27.Ingastrectomyfollowingoccursexcept- a)Calciumdeficiency b)Steatorrhoea c)Fe.deficiency d)Fluidloss CorrectAnswer-DAnsweris'd'i.e.FluidlossPostgastrectomycomplicationsAnemiaasaresultofvitaminB12orironmalabsorptionandosteoporosis.Irondeficiencyanemiadevelopsbecauseremovalofthestomachoftenleadstoamarkeddecreaseintheproductionofgastricacid.Osteoporosisdevelopsasaresultofpoorcalciumabsorption,anotherproblemthatoccursaftergastricsurgery.DumpingsyndromeDiarrhoea-maybeduetodifferentreasons.associatedwithdumpingsyndromepost-vagotomydiarrhoeaassociatedwithfatmalabsorption.Fatmalabsorptionleadingtosteatorrhoeaoccursduetoacidinactivationofpancreaticenzymesorpoorlycoordinatedmixingoffood&digestivejuices.Fatmalabsorptionleadstomalabsorptionoffatsolublevitaminsi.e.A,D,E&K.Pushingfoodfromyourstomachtoyoursmallboweltooquickly(dumpingsyndrome)AcidrefluxChestinfections,includingbronchitisandpneumoniaInternalbleedingNauseaandvomiting
Stomachacidleakingintoyouresophagus,causingscarringandnarrowing(stricture)VitamindeficienciesWeightloss
28.Allofthefollowingarefeaturesof ZollingerEllisonsyndromeexcept a)Intractablepepticulcers b)Severediarrhoea c)Betacelltumorsofthepancreas d)Veryhighacidoutput. CorrectAnswer-CAns.is'c'i.e.,BetacelltumoursofpancreaseGastrinomaorZollingerEllisonsyndromeisanon13cellneuroendocrinetumourofthepancreasItsecretesgastrinPathophysiologyofGastrinomaGastrinoma-->Increasesecretionofgastrin-->markedgastricacidhypersecretionpepticulcerPancreaticNeuroendocrineTumors Tumour Biologically Tumour active location Malignant Mainsymptoms peptide percentage andsigns secreted Gastrinoma - (non13cell ?Pain(79 Duodenum tumour) 100%) (70%) - Pancreas Gastrin 60-90 'Diarrhoea(30 (25%) 73%) othersites ?GERD(30- (5%) 35%)?Pepticulcer
Pancreas>99% ?Symptomsof (Insulinomas hypoglycemia Insulinoma are ?Symptoms (f3cell Insulin <10 distributed releiveon tumour) equallyon administrationof headbody glucose andtailofpancrease) ?Waterydiarrhoea VIPOMA (90-100%) (Verner- ?- Morrison Vasoactive Hypokalemia(80 Pancreas syndrome, intestinal 40-70 100%) 90% pancreatic peptide ? cholera, Hypochlorhydria WDHA) ?Dehydration(83%)?Flushing(20%) Pancreas ?Dermatitis 100% (migratory (usually necrolytic occursusuall erythema) singlyin 67-90% pancreatic ?Glucose tail) intolerance(40-90%) Glucagonoma Glucagon 50-80% ?Weightloss(66 to96%)?Anemia(33-85%)?Diarrhoea(15-29%)?Thromboembolism
29.Curlingsulcerisseenin- a)Burnpatients b)Patientswithheadinjuries c)ZollingerEllisonsyndrome d)Analgesicdrugabuse CorrectAnswer-AAns.is'a'i.e.,BurnpatientsCurlingulcers:arestressulcersassociatedwithburnsandmostcommonlyfoundinthefirstpartofduodenum.
30.Themostcommonlypracticedoperative procedureforaperforatedduodenalulceris- a)Vagotomyandpyloroplasty b)Vagotomyandantrectomy c)Vagotomyandperforationclosure d)Graham'somentumpatchrepair CorrectAnswer-DAns.is'd'i.e.,Graham'somentumpatchrepair
31.Prepyloricorchannelulcerinthestomach istermedas- a)type1 b)type2 c)type3 d)type4 CorrectAnswer-CAns.is'c'i.e.,Type3.Situatedinprepyloricregion.Associatedwithgastricacidhypersecretion
32.Commonestcauseofpyogenicliver abscess? a)StrictureofCBD b)BiliaryColic c)Appendicitis d)SigmoidDiverticulitis CorrectAnswer-AAnsis'a'ieStrictureofCBDMostcommonrouteofinfectiontoliverisalongthebileduct.Itmaybedueto:-i)StoneimpactedinCBD.ii)BenignormalignantstrictureofCBD.MostcommoninfectingorganismsareE.coliandKlebsiellapneumonia
33.Allaretrueaboutamoebicliverabscess except: a)Morecommoninfemales b)Morecommoninleftlobeofliver c)Metronidazoleismainstayoftreatment d)aandb CorrectAnswer-DAnsisDai.e.Morecommoninfemales&bi.e.Morecommoninleftlobeofliver"Malepreponderanceofgreaterthan10:1hasbeenreportedinallstudies"Therightlobeoftheliverismorecommonlyaffectedthantheleftlobe.amebicliverabscessis7-12timesmorecommoninmenthaninwomen,withapredominanceamongmenaged18-50years.Metronidazole800mgTDSfor5?10daysAspirationRepeatedimagingofliver
34.Falseabouthydatidcystofliver: a)Mostlyasymptomatic b)Mostcommonlylocatedinrightlobeofliver c)Mostcommoncausativeorganismisechinococcusgranulosus d)Hepaticresectionisneverdone CorrectAnswer-DAnsis'd'i.e.HepaticresectionisneverdoneLaparoscopicaspiration,unroofingandevacuationofthehepatichydatidcystscanbedone.
35.IndicationsofLivertransplantationare All/Except a)Biliaryatresia b)Sclerosingcholangitis c)HepatitisA d)Cirrhosis CorrectAnswer-CAns.is'c'i.e.HepatitisALivertransplantationisindicatedforthosechildrenandadults,whointheabsenceofcontraindicationssufferfromsevere,irreversibleliverdiseaseforwhichalternativemedicalorsurgicaltreatmentshavebeenexhaustedorareunavailable.Mostcommonindicationinchildren________Biliaryatresiainadults-->CirrhosisHarrison17/ewrites-"Currently,chronichepatitisCandalcoholicliverdiseasearethemostcommonindicationsforlivertransplantation,accountingforover40%ofalladultcandidateswhoundergotheprocedure." Indicationsfor verTransplantation Children Adults Biliaryatresia PrimarybiliarycirrhosisSecondarybiliary Neonatalhepatitis cirrhosis Congenitalhepatic Primarysclerosing fibrosis cholangitis Alagille'sdisease Autoimmunehepatitis
Alagille'sdisease Autoimmunehepatitis Byler'sdisease Caroli'sdisease al-antitrypsin Cryptogeniccirrhosis deficiencyInheriteddisordersof Chronichepatitiswithmetabolism cirrhosis Wilson'sdisease Hepaticveinthrombosis Tyrosinemia Fulminanthepatitis Glycogenstorage Alcoholiccirrhosis diseasesLysosomalstorage Chronicviralhepatitis diseases Primaryhepatocellular Protoporphyria malignancies Crigler-Najjardisease Hepaticadenomas typeIFamilial Nonalcoholic hypercholesterolemia steatohepatitisPrimaryhyperoxaluria FamilialamyloidtypeI polyneuropathy Hemophilia ContraindicationstoLiverTransplantationAbsolute Relative Uncontrolledextrahepatobiliary Age>70 infection Priorextensivehepatobiliary Active,untreatedsepsis surgery Uncorrectable,life-limiting Portalveinthrombosis congenitalanomaliesActivesubstanceoralcohol Renalfailure abuseAdvancedcardiopulmonary Previousextrahepatic disease malignancy(notincludingnonmelanomaskincancer) Extrahepatobiliarymalignancy Severeobesity
Extrahepatobiliarymalignancy Severeobesity (notincludingnonmelanomaskincancer)Metastaticmalignancytothe Severemalnutrition/wasting liverCholangiocarcinoma Medicalnoncompliance AIDS HIVseropositivity Life-threateningsystemic Intrahepaticsepsis diseases Severehypoxemiasecondarytoright-to-leftintrapulmonaryshunts(P02<50mmHg)Severepulmonaryhypertension(meanPApressure>35mmHg)Uncontrolledpsychiatricdisorder
36.Kehr'ssignseeninsplenicruptureis- a)Painoverleftshoulder b)Painoverrightscapula c)Periumbilicalpain d)Painoverrenalangle CorrectAnswer-AAns.is'A'i.e.,PainoverleftshoulderInsplenicrupturethepainmaybereferredtothetipoftheleftshoulder.ThisisknownasKehr'ssign.Itoccursduetoirritationoftheundersurfaceofthediaphragmwithbloodandthepainisreferredtotheshoulderthroughtheaffectedfibresofphrenicnerve(C4andC5).Kehr'ssigncanbeelicitedbybimaualcompressionoftheleftupperquadrantafterthepatienthasbeeninTrendelenburg'spositionforabout10minutespriortothemanoeuvre.
37. Inwhichcondition,medicaltreatmentof gallstoneisindicated- a)Stoneis<15mmsize b)Radioopaquestone c)Calciumbilirubinatestone d)Nonfunctioninggallbladder CorrectAnswer-AAnsis'a'isStoneis<15mmsizeMedicaltreatmentsforgallstones,usedaloneorincombination,includethefollowing-Oralbilesalttherapy(ursodeoxycholicacid)(particularlyforx-ray-negativecholesterolgallstonesinpatientswithnormalgallbladderfunction)Extracorporealshockwavelithotripsy(particularlyfornoncalcifiedcholesterolgallstonesinpatientswithnormalgallbladderfunction)Medicalmanagementismoreeffectiveinpatientswithgoodgallbladderfunctionwhohavesmallstones(<1cm)withahighcholesterolcontent.Bilesalttherapymayberequiredformorethan6monthsandhasasuccessrateoflessthan50%.
38.Inapatientofacutecholecystitis,referred paintotheshoulderisk/a a)Murphy'ssign b)GrayTurnersign c)Boa'ssign d)Cullen'ssign CorrectAnswer-CAnsis'c'i.e.Boas'signBoas'sign:Incasesofacutecholecystitispainradiatestothetipoftherightshoulderandanareaofskinbelowthescapulaisfoundtobehypersensitive.Thisisk/aBoas'sign.Sensitivityisquiteless.AlsoKnow:Murphy'ssign:Seeninacutecholecystitis.Murphy'ssigniselicitedbyaskingthepatienttobreatheoutandthengentlyplacingthehandbelowthecostalmarginontherightsideatthemid-clavicularline(theapproximatelocationofthegallbladder).Thepatientistheninstructedtoinspire(breathein).Normally,duringinspiration,theabdominalcontentsarepusheddownwardasthediaphragmmovesdown(andlungsexpand).Ifthepatientstopsbreathingin(asthegallbladderistenderand,inmovingdownward,comesincontactwiththeexaminer's.fingers)andwinceswitha'catch'inbreath,thetestisconsideredpositive.Inorderforthetesttobeconsideredpositive,thesamemaneuvermustnotelicitpainwhenperformedontheleftside.GreyTurner&Cullen'ssign:positiveinseverenecrotizingpancreatitis.GreyTurnerssignisbluishdiscolourationseenattheflanks.Bluishdiscolourationaroundtheumbilicusisknownas
Cullen'ssign.
39.InPatey'smastectomythestepnotdone is a)Nippleandareolaremoved b)Surroundingnormaltissueoftumorisremoved. c)Pectoralismajorremoved d)Pectoralisminorremoved CorrectAnswer-CAns.is'c'i.e.(Pectoralismajorremoved)LetsseethenomenclatureofvarioussurgeriesonbreastSimpleorTotalmastectomy*itremovesallbreasttissue,thenipple-areolacomplex,andskin.Extendedsimplemastectomy*Simplemastectomy+removaloflevelIaxillarylymphnodes.Modifiedradicalmastectomyitremovesallbreasttissue,thenipple-areolacomplex,skinandthelevelIandlevelIIaxillarylymphnodes.Halstead'sRadicalmastectomyremovesallbreasttissueandskin,thenippleareolacomplex,thepectoralismajorandminormusclesandthelevelI,IIandIIIaxillarylymphnodes.ModifiedRadicalMastectomyTwoformsofmodifiedradicalmastectomyareinuseAuchincloss(pronouncedas'aushincloss')procedure*HereboththepectoralismajorandminormusclesarepreservedwithremovaloflevelIandIIaxillarylymphnodesPatey'sProcedureherethepectoralisminormuscleisremovedtoallowcompletedissectionoflevelIIIaxillarylymphnodes
Scanlon'smodificationofPatey'sprocedure*herethepectoralisminormuscleisdividedinsteadofremoving.DivisionofpectoralisminormuscleallowscompleteremovaloflevelIIIlymphnodesHalsteadRadicalMastectomyInthisoperationfollowingstructuresareremoved.i)thewholebreastii)theportionofskinoverlyingthetumor,whichincludesthenipple-areolacomplex.iii)thesubcutaneousfatandthedeepfasciaverticallyfromthelowerborderoftheclavicleuptotheupperquarterofthesheathoftherectusabdominisandhorizontallyfromthestrenumtotheanteriorborderoflattissimusdorsiiv)pectoralismajormusclev)pectoralisminormuscleandclavipectoralfasciavi)upperpartoftheaponeurosisoftheexternalobliqueandanteriorpartsofafewdigitationsoftheserratusanteriormusclevii)allfattyandlooseareolartissuealongwithlevelI,II&IIIaxillarylymphnodesStructuressavedare:i)theaxillaryveinandthecephalicveinii)thelongthoracicnerveofBell(Nervetoserratusanterior).Thenervetolatissimusdorsimaybesacrifiedifrequired.AlsoknowExtendedRadicalMastectomy-Radicalmastectomy+removalofinternalmammarylymphnodesSuperRadicalMastectomy-Radicalmastectomy+removalofinternalmammary,mediastinalandsupraclavicularlymphnodes.
40.Breastconservationsurgerynotindicated ? a)Largependularbreast b)SLE c)Diffusemicrocalcification d)All CorrectAnswer-DAnswer(a)Largependularbreast;(b)SLE;(c)Diffusemicrocalcification
41.Inbreastcarcinomametastasis, prognosisdependsbestupon- a)Estrogenreceptorstatus b)Axillarylymphnodestatus c)Sizeoftumour d)Siteoftumour CorrectAnswer-AAnsis'a'i.e.,EstrogenreceptorstatusIncaseofmetastasis,theprognosisnomoredependsuponthelymphnodestatus.Thelymphnodestatusisthemostimportantprognosticindicatorfortumorlocalizedtobreast,asthepresenceofnodalmetastasisimpliessystemicdisseminationofcancerandhenceabadprognosis.Butonceametastasisisdiscovered,thelymphnodestatusisoflittlesignificance.MetastaticdiseaseconfersthebreasttumorTNMstageIVstatus,withworstprognosis.CurrentlystageIVdiseasesarenotcurative.Schwartz9/ewrites-"TreatmentforstageIVbreastcancerisnotcurative,butmayprolongsurvivalandenhanceawomen'squalityoflife.Hormonaltherapiesthatareassociatedwithminimaltoxicityarepreferredtocytotoxicchemotherapy.Appropriatecandidatesforinitialhormonaltherapyincludewomenwithhormonereceptor-positivecancers;womenwithboneorsofttissuemetastasisonly;andwomenwithlimitedandasymptomaticvisceralmetastasis.Abouthormonereceptors,CSDT11/e,p329writes-"thepresenceorabsenceofestrogenandprogesteronereceptorsinthecytoplasmoftumorcellsisofparamountimportanceinmanagingallpatientswithbreastcancer,especiallythosewithrecurrentormetastatic
disease.Theyareofprovedvalueindeterminingadjuvanttherapyandtherapyforpatientswithadvanceddisease.Upto60%ofpatientswithmetastaticbreastcancerwillrespondtohormonalmanipulationiftheirtumorscontainestrogenreceptors.Howeverfewerthan5%ofpatientswithmetastaticER-negativetumorscanbesuccessfullytreatedwithhormonalmanipulation."
42. WhatistrueaboutHER2/neuoverexpressioninCabreast: a)Goodprognosis b)Respondswelltotaxanes c)Respondswelltomonoclonalantibodies d)Seenonlyinbreastcancer CorrectAnswer-CAnsis'c'i.e.RespondswelltomonoclonalantibodiesTheHER2receptor(previouslycalledHER2/neu,orERBB-2receptor)belongstotheepidermalgrowthfactorreceptor(EGFR)familyofreceptors,whicharecriticalintheactivationofsubcellularsignaltransductionpathwayscontrollingepithelialcellgrowthanddifferentiationandpossiblyangiogenesis.AmplificationofHER2oroverexpressionofitsproteinproductisobservedin18to20percentofhumanbreastcancers.HER2overexpressionisalsonotedinothertumorssuchasesophagogastrictumors,lung,ovary&headandnecksquamouscellca.(Inallofthesesites,HER2overexpressionhasbeenidentifiedasanegativeprognosticfactor.)FollowingpointsaretobenotedaboutHER2overexpressioninbreastCa:PrognosticvalueofHER2--HER2overexpressionisapoorprognosticmarker.HER2overexpressionisassociatedwithhighratesofdiseaserecurrenceanddeathintheabsenceofadjuvantsystemictherapy.PredictivevalueofHER2--HER2statuspredictsresponsetospecifictherapies:PatientswithhighlevelsofHER2expressionbenefitfromtreatment
withagentsthattargetHER2,suchastrastuzumab(amonoclonalantibody)andlapatinib.HER2statusappearstopredictresistanceorsensitivitytodifferenttypesofchemotherapeuticagents,includinganthracyclinesandtaxanes.WomenwhosetumorsoverexpressHER2appeartoderivegreaterbenefitfromanthracycline-basedadjuvanttherapythanfromadjuvanttherapythatisalkylatingagent-based,suchasCMF(cyclophosphamide,methotrexate,fluorouracil).RelationshipbetweenHER2overexpressionandtaxanesisstillunderstudywithvariousstudiesgivingconflictingreports.HER-2positivityisassociatedwithresistancetoendocrinetherapies. ScoringofHER-2ImmunohistochemistryAssays Score HER-2Status StainingPattern 0 Nostainingormembranestainingin Negative <10%oftumorcells? 1+ Negative ?Afaintbarelyperceptible membranestainingisdetectedin >10% oftumorcells.Thecellsareonlystainedinpartofthemembrane?. 2+ Equivocal ?Weaktomoderatecomplete membranestainingisseenin>10% of tumorcellsor<30%withstrongstaininga 3+ Positive ?Strongcompletemembrane stainingisseenin>30%oftumor cells?
43.Mondor'sdiseaseis? a)ThrombophlebitisoftheSuperficialveinsofBreast b)Carcinomaofthebreast c)Premalignantconditionofthebreast d)Filariasisofthebreast CorrectAnswer-AAnsis'a'i.e.,ThrombophlebitisofSuperficialveinsofBreastMondor'sdiseaseisthrombophlebitisofthesuperficialveinsofanteriorchestwallandbreastalthoughithasalsobeenseeninthearm.frequentlyinvolvedveinsarelateralthoracicvein,thoracoepigastricveinandsuperficialepigastricveins.aetiologyisunknownalsoknownas'stringphlebitis',itpresentsasatendercord-likestructure.Thewomenmaypresentwithacutepaininthelateralasepctofbreastortheanteriorchestwall.Atendercord-likesuperficialthrombosedveinisformedandwhentheskinoverthebreastisstretchedbyraisingthearm,anarrowshallowsubcutaneousgroovealongsidethecordbecomesapparent.rarelyitmaybebilateral.Managementit'sabenignself-limiteddisorderThedifferentialdiagnosisislymphaticpermeationfromanoccultcarcinomaofbreastWhenthediagnosisisuncertainoramassispresentnearthecord,abiopsymaybedone.Treatmentantiinflammatorydrugsandwarmcompresses
restrictedarmmovementsaswellasbrassieresupportofbreastitusuallyresolveswithin4to6weeks.Whensymptomspersistsorarerefractorytotreatment,theinvolvedveinsegmentmaybeexcised.
44.Flapcommonlyusedinbreast reconstructionis? a)Serratusanterior b)TRAM c)Flapfromarm d)Deltopectoralflap CorrectAnswer-BAns.is'b'i.e.TRAM(Mostcommon)SinglePedicleDoublePedicleFreeflapDeepinferiorepigastricperforatorflap
45.Lateralaberrantthyroidrefersto- a)Congenitalthyroidabnormality b)Metastaticfocusinlymphnodes c)Strumaovarii d)Lingualthyroid CorrectAnswer-BAnsis"B"i.e.MetastaticfocusinlymphnodesLateralaberrantthyroidreferstoametastaticfocusinlymphnodes.Lateralaberrantthyroidisamisnomerandalwaysreferstothepresenceofmetastaticthyroidcarcinoma(papillary)incervicallymphnodes.Itdoesnotindicatethepresenceofectopicthyroidtissueasthenamesuggests.
46. Treatmentofchoiceformedullary carcinomaofthyroidis: a)Totalthyroidectomy b)Partialthyroidectomy c)1131ablation d)Hemithyroidectomy CorrectAnswer-AAns.is'a'i.e.TotalthyroidectomyTreatmentofthyroidmalignanciesisasfollows:-(Note-thisisaveryimportantandoftenrepeatedtopicinPGexams,soiftimepermitsonemustturnthepagesofSchwartzSxoranyotherstandardbookfordetailedstudy).PapillaryThyroidCarcinoma(PCT)High-risktumorsorbilateraltumors-Totalthyroidectomy(orneartotalthyroidectomy).Lowrisk-Thetreatmentiscontroversial.Conservativeapproachadvocateshemithyroidectomy(lobectomy+isthmusectomy).-Moreradicalapproachadvocatestotalthyroidectomy(orneartotalthyroidectomy).-SchwartzSxandDevita'sOncologyareinfavourofradicalapproach.(HighandLowriskdecidedbyanyoneofthemanyclassificationsystems)Ifenlargedlymphnodesarefound-Modifiedradicalneckdissectionisdoneoftheaffectedside.Whenpatientsarefoundtohaveaminimalpapillarythyroidcarcinomainathyroidspecimenremovedforotherreasons,unilateralthyroidlobectomyandisthmusectomyisusually
consideredtobeadequatetreatment,unlessthetumorhasevidenceofangioinvasion,multifocality,orpositivemargins.FollicularThyroidCarcinoma(FTC)FNAbiopsyisunabletodistinguishbenignfollicularlesionsfromfollicularcarcinomastherefore,preoperativediagnosisofcancerisdifficultunlessdistantmetastasesarepresent.PatientsdiagnosedbyFNAbiopsyasfollicularlesionsshouldundergothyroidlobectomy+isthmusectomy(becauseatleast80%ofthesepatientswillhavebenignadenomas).Theresectedlobeissubjectedtohistology(intraoperativefrozen?sectionexamination,thoughusuallynothelpfulshouldbeperformedinhighriskcases).
47.Incaseofhypothyroidismwhich investigationismostinformativeandmostcommonlyused? a)SerumTSHLevel b)SerumT3,T4Level c)SerumCalcitoninassay d)SerumT3level CorrectAnswer-AAnsisAieSerumTSHLevel"AnormalTSHLevelexcludesprimary(butnotsecondary)hypothyrodism.IftheTSHiselevatedafreeT4levelisneededtoconfirmthepresenceofclinicalhypothyrodism,butfreeT4isinferiortoTSHwhenusedasascreeningtest,asitwillnotdetectsubclinicalormildhypothyroidism.CirculatingfreeT3levelsarenormalinabout25%ofthepatients,reflectingadaptiveresposetohypothyroidism.T3measurementsarethereforenotindicated."-Harrison"Amongthevariousavailableserumthyroidfunctiontests,TSHisthemostusefultoassessglanddysfunction."?EndocrineSurgeryofHead&Neck,p83.
48.M.C.siteofCAoesophagusis- a)Middle1/3rd b)Upper1/3rd c)Lower1/3rd d)Lowerendofesophagus CorrectAnswer-AAnsis(a)ieMiddle1/3rdWell,Iamnotquitesureoftheanswer.EsophagealCarcinomaisoftwocommonhistologicaltypesi)SquamouscellCa-theMCtypeinworld(-95%accordingtoSabiston)ii)Adenocarcinoma-whereincidenceisincreasingatarapidrateandisnowtheMCtypeinUSA(Ref.Harrison,Schwartz)DistributionofSquamousCellCa.Upper1/310%Middle11360%Lower1/330%Adenocarcinomaismainlylocatedinlower1/3.Nowhere,IcouldgettheMCsiteofesophagealcarcinomaoverall(includingbothsquamousandadenovariety).[HarrisonwritestheMCsitetobelower1/3,butitgivesincidenceforUSpopulationonly,notforthewholeworld]Butonethingissure-squamouscellCaistheMCtypeofesophagealcancerinworld[(Ref:Bailey&Love,25/ep1026(24/ep1009);Robbins8/ep772(7/ep806);SabistonI8/ep1090(I7/ep1118)]SoIpresumethattheoverallMCsitewouldbetheMCsiteinvolvedbytheMCtypeofesophagealcanceri.e.-*Middle1/3
SorememberMCtypeofesophagealCainworld->SquamousCellCa(--95%)MCsite->Middle1/3
49.WhichisM.C.siteforiatrogenic oesophagealperforation- a)Abdominalportion b)Cervicalportion c)Abovearchofaorta d)Belowarchofaorta CorrectAnswer-BAnsis'b'ieCervicalportionOesophagealperforationisoftwotypesIatrogenic(MC):-CommonsiteiscervicalesophagusJustabovetheuppersphinctcr.Spontaneousrupture:asseeninBoerhaavessyndrome(Ruptureofesophagusaftervomitting)-commoninthelower1/3ofesophagus.
50.Corkscrewesophagusisseeninwhichof thefollowingconditions? a)Carcinomaesophagus b)Scleroderma c)Achalasiacardia d)Diffuseesophagusspasm CorrectAnswer-DAns.isdi.e.,DiffuseEsophagealspasmRadiologicalappearancesofdiffuseesophagealspasmhavebeendescribedas:curlingesophagusCorkscrewesophagusor-pseudodiverticulosisDiffuseesophagealspasmisamotordisorderofesophaguscharacterizedbyrepetitivesimultaneousnon-peristalticcontractions.Symptomsaresubsternalchestpainand/ordysphagiaDiagnosedbymanometryAlsoknowNutcrackeresophagusalsoknownas'supersqueezer'esophagus-itischaracterizedbyextremelyhigh-amplitudeperistalticcontraction-SymptomsarepainanddysphagiaDiagnosedbymanometrystudywhichshowsperistalticesophagealcontractionswithpeakamplitudesgreaterthantwostandarddeviationsabovethenormalvalues.
51.ThemostcommontypeofTracheo- OesophagealFistulais- a)Esophagealatresiawithouttracheoesophagealfistula b)Esophagealatresiawithproximaltracheoesophagealfistula c)Esophagealatresiawithdistaltracheoesophagealfistula d)Esophagealatresiawithproximalanddistalfistula CorrectAnswer-CAns.is'c'i.e.,EsophagealatresiawithdistaltracheoesophagealfistulaTEFisclassifiedintofivetypesbasedonpresenceofesophagealatresiaandlocationoffistula:?1)TypeA:ThereisesophagealatresiawithoutTEF.Thereisnogasinabdomen.Itis2'dmostcommontype.2)TypeB:ThereisproximalTEF.Thereisnogasinabdomen.3)TypeC:ThereisproximalesophagealatresiawithdistalTEF.Gasinabdomenispresent.Itismostcommon.4)TypeD:Bothproximalanddistalfistulaarepresent.Gasinabdomenispresent.Itisleastcommon.5)TypeE:IsolatedTEF(HorNtype)isthere.
52.commonestbenigntumorofthe esophagus? a)Leiomyoma b)Papilloma c)Adenoma d)Hemangioma CorrectAnswer-AAnsisa.i.e.Leiomyoma"Leiomyomasconstitutemorethan50%ofbenignesophagealtumors"-Schwartz
53.Allofthefollowingareriskfactorsfor carcinomagallbladder,EXCEPT- a)Typhoidcarriers b)Adenomatousgallbladderpolyps c)Choledochalcysts d)Oralcontraceptives CorrectAnswer-DAns.isdi.e.,OralContraceptivesRiskfactorsforCaGallbladderare:i)Gallstonesii)Adenomatousgallbladderpolyps(particularlypolypslargerthan10mm)iii)Calcified(porcelain)gallbladderiv)Choleduochalcystv)Estrogensvi)Anomalouspancreaticobiliaryductjunctionvii)Exposuretocarcinogens(azotoulene,nitrosamine)viii)Typhoidcarriersix)Sclerosingcholangitis
54.RegardingCagallbladder- a)Squamouscellcaisthemostcommon b)Presentwithjaundice c)Goodprognosis d)All CorrectAnswer-BAnswer(b)PresentswithjaundiceBiliarytractcancerstendtobeslow-growingtumoursthatinvadelocallyandmetastasisetolocallymphnodes.Distantmetastasestotheperitonealcavity,liverandlungdooccur.Jaundiceisthemostcommonpresentingfeature.Abdominalpain,earlysatietyandweightlossarealsocommonlyseen.Onexamination,jaundiceisevident,cachexiaoftennoticeableandapalpablegallbladderispresentiftheobstructionisinthedistalcommonbileduct(Courvoisier'ssign).
55.TheGallstonepainisreferredtothe shoulderthroughwhichofthefollowingnerves: a)C2-C8 b)T1-T4 c)T8-T12 d)C3-05 CorrectAnswer-DAnsis'd'i.e.C3-05Gallstonediseasemayreferpaintotherightshouldertip(k/aKehr'ssign).Thisisbecause,aninflamedgallbladderirritatesthediaphragmwhichissuppliedbythephrenicnerve(C3-05).Thesecervicalnerveroots,alsoprovidesensorysupplytotherightshoulderthroughsupraclavicularnerves.HencethegallbladderpainisreferredtotherightshoulderthroughtheC3-05nerveroots.Kehr'ssignisaclassicexampleofreferredpain:irritationofthediaphragmissignaledbythephrenicnerveaspainintheareaabovethecollarbone.Thisisbecausethesupraclavicularnerveshavethesamecervicalnervesoriginasthephrenicnerve,C3andC4.Boas'signcanalsoindicatestomachandduodenaldisease.WhenthetransverseprocessesofthoracicvertebraeT10-T12arepressedoreffleuragedwiththebottomofthehand,paincanappearatleftofspinousprocesses(instomach'slessercurvatureulcer)oratright(inpyloricorduodenalulcer).Boas'orBoas'ssignishyperaesthesia(increasedoralteredsensitivity)belowtherightscapulacanbeasymptominacutecholecystitis(inflammationofthegallbladder).
cholecystitis(inflammationofthegallbladder).
56.Thebestinvestigativemodalityforgall bladderstones- a)Oralcholecystogram b)Percutaneoustranshepaticcholangiography c)Ultrasound d)Intravenouscholangiogram CorrectAnswer-CAns.is'c'i.e.UltrasoundUltrasonography:-Thisistheinvestigationofchoiceforthedetectionofgallstones,andobstructivejaundice.USGshowsechogenicfocuswithposterioracousticshadowing.Themostspecificsignofacontracted,stone-filledgallbladderisahypoechoicwallsuperficialtoacurvilinearechofromthestonesandanacousticshadow:-WEStriad(Wall,Echo,Shadow)orthe"Doublearcshadowsign".IOCfordiagnosisofgallstone-acutecholecystitis,acuteacalculouscholecystitis,chroniccholecystitisisUltrasound
57.Strawberrygallbladderisseenin? a)Gangreneofgallbladder b)Porcelaingallbladder c)Adenomatosis d)Cholesterosis CorrectAnswer-DAns.is'i.e.CholesterosisCholecystosisisthechronicinflammatoryconditionofthegallbladderwithcholesteroldeposits.TypesI.Aggregationsofcholesterolcrystalsinthemucosaorsubmucosa--cholesterosis(Strawberrygallbladder).Lipoidcontentsarepresentinlargefoamycellsthathavephagocytosedcholesterol.Herecysticductisnormal.Thediseaseoccursonlyingallbladder.Itisapremalignantcondition.II.Cholesterolladenpolypoidprojectionsinthemucosa--cholesterolpolyposis(Gallbladderpolyp).III.Granulomatousthickeningandhyperplasiaofthegallbladder--cholecystitisglandularisproliferans.IV.Diverticulaformationinthewallofthegallbladder--diverticulosisofgallbladder.V.Gallbladderwallfistula.
58.VIPomaisassociatedwithwhich syndrome: a)ZollingerEllison b)VernerMorrison c)Carcinoidsyndrome d)Cushing'ssyndrome CorrectAnswer-BAnsis'b'i.e.VernerMorrisonVlPomasareendocrinetumorsthatsecreteexcessiveamountsofvasoactiveintestinalpeptide(VIP),ExcessiveVIPcausesadistinctsyndromecharacterizedbylarge-volumediarrhea,hypokalemia,anddehydration.ThissyndromealsoiscalledVerner-Morrisonsyndrome,pancreaticcholera,andWDHAsyndromeforwaterydiarrhea,hypokalemia,andachlorhydria.Theprincipalsymptomsarelarge-volumediarrhea(100%)severeenoughtocausehypokalemia(80-100%),dehydration(83%),hypochlorhydria(54-76%),andflushing(20%).Inadults80-90%ofVlPomasarepancreaticinlocation,withtherestduetoVIP-secretingpheochromocytomas,
59.Allareresectedinwhipplesoperation except? a)Duodenum b)Headofpancreas c)Neckofpancreas d)Commonbileduct CorrectAnswer-CAnsis'c'Whipplesoperation(Pancreaticoduodenectomy)isthemostcommonlyperformedoperationforcarcinomaofheadofpancreas.Itincludesresectionof:-distalstomach-duodenum-gallbladder-proximaljejunum-CBD-regionallymphatics-headofpancreasRestorationofgastrointestinalcontinuityrequires-pancreaticojejunostomycholedochojejunostomy&-gastrojejunostomy
60.Followinggeneticcounsellinginafamily forFamilialpolyposiscoli(FPC)nextscreeningtestis a)Flexiblesigmoidoscopy b)Colonoscopy c)Occultbloodinstools d)APCgene CorrectAnswer-DAns.is'd'i.e.APCgeneSchwartzwrites-"Flexiblesigmoidoscopyoffirst-degreerelativesofFAP(Familialadenomatouspolyposis)patientsbeginningatage10to15yearshasbeenthetraditionalmainstayofscreening.Todayfollowinggeneticcounselling,APCgenetestingmaybeusedtoscreenfamilymembersprovidinganAPCmutationhasbeenidentifiedinafamilymember."Familialadenomatouspolyposis(FAP)isadominantlyinheritedcoloncancersyndromeduetogermlinemutationsintheadenomatouspolyposiscoli(APC)tumorsuppressorgeneonchromosome5.Patientswiththissyndromedevelophundredstothousandsofadenomasinthecolonwhichifleftuntreatedwilleventuallydevelopintocoloncancer.EachoftheFAPadenomashaslostthenormalremainingalleleofAPCgenebuthasnotyetaccumulatedtherequiredadditionalmutationstogeneratefullymalignantcells.Induecourseoftimemanyoftheseadenomasacquirefurthergeneticabnormalitiesanddevelopintofullymalignantcancers.APCgeneisthusconsideredto
beagatekeeperforcolontumorigenesis.ThedetectionofAPCgenemutationinfamilymembersofaFAPpatienthelpsinmakingadefinitivediagnosisbeforethedevelopmentofpolyps.
61.Thetendencyofcoloniccarcinomato metastasizeisbestassessedby- a)Sizeoftumor b)Carcinoembryonicantigen(CEA)levels c)Depthofpenetrationofbowelwall d)Proportionofbowelcircumferenceinvolved. CorrectAnswer-CAns.isc)i.e.depthofpenetrationofbowelwallsSchwartzwrites"Regionallymphnodeinvolvementisthemostcommonformofspreadofcolorectalcarcinomaandusuallypreceedsdistantmetastasisorthedevelopmentofcarcinomatosis.theTstage(depthofinvasion)isthesinglemostsignificantpredictoroflymphnodespread."Fromtheabovegivenlines'depthofpenetrationofbowelwall'appearstobepredictorofdistantmetastasesaswell.CEAlevelisamarkerforrecurranceofcolorectalcaaftersurgicalresection.Thoughitspreoperativelevelshassomeprognosticsignificance,itisnotapredictorfordistantmetastasis.CEAlevelisusedtofollowuppostoperativecasesofcolorectalcancer,forearlydetectionofrecurrance.
62.Followingisleastcommonabout angiodyplasiaofcolon- a)Involvementofcecum b)Involvementofrectumin50%ofcases c)Affectingagegroup>40yrs. d)CauseoftroublesomelowerG.I.hemorrhage CorrectAnswer-BAns.is'b'i.e.,Involvementofrectumin50%casesSite:Occurmostcommonlyintheascendingcolonandcaecum;howevertheycanalsooccureinrestofcolonandsmallbowel.ClinicalfeaturesAnemia-mostcommonpresentationHemoatocheziaMalenaThereisanassociationwithaorticstenosis(heyd'ssyndrome).
63.MostcommontypeofIntussusceptionis- a)Ileocolic b)ileoileal c)Colo-colic d)Caeco-colic CorrectAnswer-AAns.is'a'i.e.,IleocolicIleo-ileo-colic(-12)Ileoileal(-5%)Colocolic(-2%)Multiple(1%)RetrogradeThemostcommontypeofintussusceptionisileocolic(alsoknownasileocecal)(90%).
64.Featuresofintestinalobstruction: clinically/investigationby: a)Abdominaldistension b)Vomiting c)FluidlevelinX-ray>4 d)aandb CorrectAnswer-DAns.is'a'&'b'i.e.Abdominaldistension&VomitingAboutoption'c'>5air-fluidlevelsinx-rayabdomensuggestintestinalobstructionGrainger'sDiagnosticradiologywrites-"3to5fluidlevelslessthan2.5cminlengthmaybeseen,particularlyintherightlowerquadrant,withoutanyevidenceofintestinalobstructionorparalyticileus."
65.Commonestcauseofintussusceptionis- a)Submucouslipoma b)Meckel'sdiverticulum c)Hypertrophyofsubmucouspeyer'spatches d)Polyp CorrectAnswer-CAns.is'c'i.e.,Hypertrophyofsubmucosapeyer'spatches
66.WhatistheT/tofptwithcarcinoidtumor ofappendixofsizemorethan2cm- a)Righthemicolectomy b)Appendicectomy c)Appendicectomy+abdominalCTscan d)Appendicectomy+24hrsurinaryHIAA CorrectAnswer-AAnsis'a'ie.Righthemicolectiomy
67.A25yearoldmanpresentswith3days historyofpainintherightlowerabdomenandvomitings.patient'sgeneralconditionissatisfactoryandclinicalexaminationrevealsatenderlumpinrightiliacfossa.Themostappropriatemanagementinthiscasewouldbe a)Immediateappendicectomy b)Exploratorylaprotomy c)OschnerSherrenregimen d)Externaldrainage CorrectAnswer-CAns.is'c'i.e.,OschnerSherrenregimenThepatientispresentingwithtypicalclinicalfeaturesofappendicalmass.Ifanappendixmassispresentandtheconditionofthepatientissatisfactory,thestandardtreatmentistheconservativeOchsner-Sherrenregimen.Thisstrategyisusedastheinflammatoryprocessisalreadylocalisedandthatinadvertentsurgeryisdificultandmaybedangerous.Itmaybeimpossibletofindtheappendixand,occasionally,afaecalfistulamayform.Forthesereasons,itiswisetoobserveanon-operativeprogrammebuttobepreparedtooperateshouldclinicaldeterioration
68.Whichofthefollowingisnotasignseen inacuteapendicitis a)Rovsing's b)Rosenstein'ssign c)Boa'ssign d)Hamburgersign CorrectAnswer-CAns.is'C'AccessorysignsofappendicitisAure-Rozanova'ssign:IncreasedpainonpalpationwithfingerinrightPetittriangle(canbeapositiveShchetkin-Bloomberg's).Bartomier-Michelson'ssign:Increasedpainonpalpationattherightiliacregionasthepersonbeingexaminedliesonhisorherleftsidecomparedtowhenhe/sheliesontheback.Dunphy'ssign:Increasedpainintherightlowerquadrantwithcoughing.Hamburgersign:Thepatientrefusestoeat(anorexiais80%specificforappendicitis)Kocher's(Kosher's)sign:Fromtheperson'smedicalhistory,thestartofpainintheumbilicalregionwithasubsequentshifttotherightiliacregion.Massouhsign:DevelopedinandpopularinsouthwestEngland,theexaminerperformsafirmswishwithhisorherindexandmiddlefingeracrosstheabdomenfromthexiphoidprocesstotheleftandtherightiliacfossa.ApositiveMassouhsignisagrimaceofthepersonbeingexamineduponarightsided(andnotleft)sweep.Obturatorsign:Thepersonbeingevaluatedliesonherorhisbackwiththehipandkneebothflexedatninetydegrees.Theexaminer
holdstheperson'sanklewithonehandandkneewiththeotherhand.Theexaminerrotatesthehipbymovingtheperson'sankleawayfromhisorherbodywhileallowingthekneetomoveonlyinward.Apositivetestispainwithinternalrotationofthehip.Psoassign,alsoknownas"Obraztsova'ssign",isrightlower-quadrantpainthatisproducedwitheitherthepassiveextensionoftherighthiporbytheactiveflexionoftheperson'srighthipwhilesupine.Thepainthatiselicitedisduetoinflammationoftheperitoneumoverlyingtheiliopsoasmusclesandinflammationofthepsoasmusclesthemselves.Straighteningoutthelegcausespainbecauseitstretchesthesemuscles,whileflexingthehipactivatestheiliopsoasandcausespain.Rovsing'ssign:Paininthelowerrightabdominalquadrantwithcontinuousdeeppalpationstartingfromtheleftiliacfossaupwards(counterclockwisealongthecolon).Thethoughtistherewillbeincreasedpressurearoundtheappendixbypushingbowelcontentsandairtowardtheileocaecalvalveprovokingright-sidedabdominalpain.Sitkovskiy(Rosenstein)'ssign:Increasedpainintherightiliacregionasthepersonisbeingexaminedliesonhis/herleftside
69.Incarcinomaofanusdistalmarginof clearanceofanalcanalofatleast a)2cm b)5cm c)4cm d)7cm CorrectAnswer-AAns.is'a'i.e.,2cm[RefBailey&Love25th/ep.1233]
70.Incaseoffemalecommonestherniais? a)Directinguinalhernia b)Indirectinguinalhernia c)FemoralHernia d)Incisionalhernia CorrectAnswer-BAnsis'b'ieIndirectInguinalherniaInIndirectinguinalherniathecontentsoftheabdomenenterthedeepinguinalringandtraversethewholelengthoftheinguinalcanaltocomeoutthroughthesuperficialinguinalring.Itisthemostcommonofallformsofhernia.Itismostcommonintheyoung(cfadirectherniaismostcommonintheold)Femoralherniasoccurmostcommonlyinwomenbutlowerincidenceoverallthaninguinalhernias.
71. TriangleofDoomisboundedbyallofthe followingexcept: a)Cooper'sligament b)Vasdeferens c)Gonadalvessels d)Peritonealreflection CorrectAnswer-AAnsisai.e.Cooper'sligamentThetriangleofdoomisboundedMediallybythevasdeferensLaterallybythevesselsofthespermaticcord(gonadalvessels)InferiorlybyperitonealreflectionApexorientedsuperiorlyatinternalring.ThecontentsofthespaceincludeExternaliliacvesselsDeepcircumflexiliacveinFemoralnerveGenitalbranchofthegenitofemoralnerve

72.Regardingdesmoidtumourwhichisnot correct? a)Oftenseenbelowtheumbilicus b)Unencapsulated c)Morecommoninwomen d)Highlyradiosensitive CorrectAnswer-DAns.is`d'i.e.,HighlyradiosensitiveDesmoidtumourisanaggressivefibromatosisormusculoaponeuroticfibromatosisisamonoclonalfibroblasticproliferationarisinginmusculoaponeuroticstructures.Histologicallythesetumoursareconsistofspindleshapedcellsincollagenousmatrixandlackthepleomorphic,atypicalorhyperchromaticnucleiofmalignancy.AminorityofdesmoidtumoursareassociatedwithGardenersyndromeandmutationsoffamilialadenomatouspolyposis(FAP)gene.Mostspontaneousdesmoidtumoursareassociatedwithmutationsofbeta-cateningene.
73.Burstabdomenmostcommonlyoccurs onthe a)2ndday b)3rdday c)7thday d)9thday CorrectAnswer-CAns.is'c'i.e.,7"'dayBurstabdomenoccursmostlybetweenthe7"and10thdayafteroperation(butmayoccuranytimeaftersurgeryfrom1tomorethan20days)(refSabiston18/e)
74.Falseregardinghypernephromais- a)Radiosensitive b)Arisefromcortexusuallyfrompreexistingadenoma c)Maypresentwithrapidlydevelopingvaricocele d)Usuallyadenocarcinoma CorrectAnswer-AAns.is'a'i.e.,RadiosensitiveRCCorhypernephromaisoneofthemostradioresistantandchemoresistanttumors.InmenrapidlydevelopingvaricoceleisrarebutimpressivesignforRCCoccuringmostoftenontheleftside.Itoccursbecauseleftgonadalveinisobstructedwhereitjoinstheleftrenalvein.AsalreadystatedRCCoriginatesinthecortexandtendstogrowoutintoperinephrictissuecausingcharacteristicbulgeormasseffect.AboutadenomasAdenomasarethemostcommonbenignrenalparenchymallesions.-Despitetheclassificationofadenomaasabenigntumor,noclinical,histologicorimmunohistochemicalcriteriacandifferentiaterenaladenomafromrenalcarcinoma.Smith'sUrologywritesaboutadenomas?-"Previously,allrenaltumorslessthan3cmwereconsideredadenomas.However,evensuchsmalltumorscanmetastasizeandarenowclassifiedasrenalcellcarcinoma.Adenomasofanysizeshouldbetreatedasafortuitousfindingrepresentativeofanearlyrenalcancer,andthepatientshouldbeevaluatedandtreatedappropriately."
75.Theposteriorurethraisbestvisualizedby ? a)Staticcystogram b)Retrogradeurethrogram c)Voidingcystogram d)CTcystogram CorrectAnswer-CAns.is'c'i.e.,VoidingCystogramVoidingcystourethrographyisthebestmethodtovisualizeposteriorurethra.Remember,Urethracanbeimagedradiographicallyintwoways.Anterogradetechniques-->Bestforvisualizationofposteriorurethra.(Thisisdonealongwithvoidingcystourethrographyorwithvoidingfollowingexcretoryurography)Retrogradetechnique-->Bestforexaminingtheanterior(penile)urethra(Contrastisinjectedthroughtipofurethra).
76.TheGrayhackshuntisestablished between: a)Corporacavernosaanddorsalvein b)Corporacavernosaandsaphenousvein c)Corporacavernosaandglans d)Corporacavernosaandcorporaspongiosa CorrectAnswer-BAnsis'b'i.e.CorporacavernosaandsaphenousveinTheGrayhackshuntisasurgicalshuntbetweencorporacavernosaandthesaphenousveindoneforthetreatmentofischemicpriapism.Priapismisanuncommonconditionofprolongederection.Itisusuallypainfulforthepatient,andnosexualexcitementordesireispresent.Priapismmaybeclassifiedintohigh-andlow-flowtypes(Nonischemicandischemic).Nonischemic(Highflow)priapism:Nonischemicpriapism,alsotermedarterialorhigh-flowpriapism,featureselevatedvascularflowthroughthecorporacavernosa.Itusuallyoccurssecondarytoperinealtrauma,whichinjuresthecentralpenilearteriesandresultsinlossofpenileblood-flowregulation.Aspirationofpenilebloodforblood-gasdeterminationdemonstrateshighoxygenandnormalcarbondioxidelevels.Arteriographyisusefultodemonstrateaneurysmsthatwillrespondtoembolization.Erectilefunctionisusuallypreserved.Ischemic(low-flow)priapism:Ischemicpriapism,alsotermedveno-occlusiveorlow-flowpriapism,
featureslittleorabsentintracorporalbloodflow.Itrepresentsatruecompartmentsyndromeinvolvingthepenis,needingemergencymanagement.Itistypicallypainful.Thecorporacavernosaistensewithcongestedbloodandtendertopalpation.Theglanspenisandcorpusspongiosumaresoftanduninvolvedintheprocess.Thecurrenttheoriesregardingthemechanismofpriapismremainindebate,butmostauthoritiesbelievethemajorabnormalitytobephysiologicobstructionofthevenousdrainage.Thisobstructioncausesbuildupofhighlyviscous,poorlyoxygenatedblood(low02,highCO2)withinthecorporacavernosa.Iftheprocesscontinuesforseveraldays,interstitialedemaandfibrosisofthecorporacavernosawilldevelop,causingimpotence.TreatmentIschernicpriapismisaurologicemergency.-First-linetreatmentconsistsofaspirationofbloodandirrigationofthecorporacavernosa(viaaneedleputinthecorporacavernosa)alongwithintracavernousinjectionofana-adrenergicsympathomimeticagent(phenylephrine).(Sympathomimeticagentscanbeexpectedtoexertcontractileeffectsonthecavernoustissueandthusfacilitatedetumescence.)-Surgicalshuntingisneededwhentheintracavernoustreatmentfails.Asurgicalshunthastheobjectiveoffacilitatingblooddrainagefromthecorporacavernosa,bypassingthevenoocclusivemechanismofthesestructures.Avarietyofshuntproceduresmaybeperformed.Adistalcavernoglanular(corporoglanular)shuntisthefirstchoice.
77.Whichofthefollowingurethralanomalyis themostcommon- a)Hypospadias b)Pinholemeatus c)Epispadias d)Strictureurethra CorrectAnswer-AAns.is'a'i.e.,HypospadiasHypospadiasisaconditioninwhichtheurethralmeatusopensontheundersideofpenis*ortheperineum(i.e.ventralsurfaceofpenis)proximaltothetipoftheglanspenis.Hypospadiasisthemostcommon*congenitalmalformationoftheurethra.(alsoknowthatthemostcommoncongenitalmalformatiooftheurinarytractisDuplicationofrenalpelvis*)Occursin1:250malebirths.
78.Allthefeaturesofmembranousurethral injuryexcept a)bloodofmeatus b)Retentionofurine c)Pelvicfracture d)Perinealbutterflyhematoma CorrectAnswer-DAns.is'd'1.Urethralruptureisalsooftwotypes:a.Bulbarurethralinjury-MCb.Membranousurethralinjury2.Extravasationofurine(+blood)inbulbarurethralinjurya.Itisasuperficialextravasation.b.IftheBuck'sfasciaremainsintact,extravasationofbloodandurineareconfinedtothepenileshaft.HoweverdisruptionofBuck'sfasciaallowsextravasatedcontentsintoaspacelimitedbycolle'sfasciaformsascrotalandperinealbutterflyhematomawhichcanextenduptheabdominalwall.3.ExtravasationofurineinMembranousurethralinjury&Extraperitonealbladderrupture.a.Extravasationissameinbothb.Itisadeepextravasationc.Urineextrvasatesinthelayersofthepelvicfasciaandtheretroperitonealtissuesd.Urinecollectsintheperivesicalspace.e.Thetypicalfindingoncystogramisextravasationofcontrastmaterialintothepelvisaroundthebaseofthebladder.
79.Congenitalhydroceleisbestt/tby- a)Eversionofsac b)Excisionofsac c)Lordsprocedure d)herniotomy CorrectAnswer-DAns.is'd'ie.HerniotomyBaileywrites?"Congenitalhydroceleareaspecialformofindirectinguinalherniaandaretreatedbyherniotomy."CongenitalhydroceleInthisconditontheprocessusvaginalisremainpatentsothereisdirectcommunicationofthetunicavaginaliswiththeperitonealcavity.Thecommunicatingorificeatthedeepinguinalringistoosmallforthedevelopmentofhernia.Itispresentsincebirth*.Incontradicationtoassumption.congenitalhydroceleisnoteasilyreducible*duetonarrownessofthedeepinguinalringbutwhenthechildliessupine,itdisappears.
80.Whichoneofthefollowingstatementis trueofundescendedtestis- a)Usuallydescendsspontaneouslyatpuberty b)Orchipexytobedoneifnodescentbypuberty c)Hasahigherincidenceofmalignancy d)Maintainsnormalspermproduction CorrectAnswer-CAns.is'c'i.e.,HasahigherincidenceofmalignancyTREATMENT-*Orchidopexydonebefore6monthsofage*Orchidectomy-patientwithincompletedescendedtestisisatrophic,pastpubertyandnormaltestis*Ombredanne'soperationinbilateralcases*HCGorGnRH-cryptorchidismassociatedwithhypogenitalismandobesity*ApproximatelyUDTspontaneouslydescendby3monthsofage*Secondarysexualcharactersticsarenormal
81.Whatisoliguria- a)Excretionoflessthan300m1in24hrs b)Excretionoflessthan500m1in24hrs c)Excretionoflessthan300ml.in12hrs d)Excretionoflessthan100ml.in24hrs CorrectAnswer-BAns.is'b'i.e.,Excretionoflessthan500mlin24hrs(NotethatBailey&Love25/ep1279(24/ep1300&23/ep1168)writeOliguriatobelessthan300mlin24hrs,butwewouldfollowHarrison&CMDT;NotethatSmith'sUrology17/ep531(16/e,p538)writeitto<400ml/day!!whatanutterconfusion.)
82.Tissuesuturinggluecontains: a)Cyanoacrylate b)Ethanolamineoleate c)Methacrylate d)Polychloroprene CorrectAnswer-AAnsis'a'i.e.CyanoacrylateMosttissueadhesivesorgluearecyanoacrylatepolymers,suchasn-butyl-2-cyanoacrylate(eg,Histoacryl?,PeriAcryl?)or2-octylcyanoacrylate(eg,Dermabond?,Surgiseal).Cyanoacrylatetissueadhesivesareliquidmonomersthatundergoanexothermicreactiononexposuretomoisture(eg,ontheskinsurface),changingtopolymersthatformastrongtissuebond.Whenappliedtoalaceration,thepolymerbindsthewoundedgestogethertoallownormalhealingoftheunderlyingtissue.Comparedwithwoundsclosedwithsutures,thetensilestrengthofwoundsclosedbytissueadhesivesislessatthetimeofinitialapplication,butequalizesbyoneweekpost-repair.Advantages:Lesspainfulapplication,andsometimesnoneedforlocalanestheticinjectionMorerapidapplicationandrepairtimeCosmeticallysimilarresultsat12monthspost-repairWaterproofbarrierAntimicrobialpropertiesBetteracceptancebypatientsNoneedforsutureremovalorfollow-upIndicationsandcontraindications:
Foruseoftissueadhesivesthewoundneedstobeclean,drywithnearperfecthemostasisandundernotension.Complexstellatelesionsorcrushinjuriesshouldnotbeclosedwithtissueadhesivessincegoodwoundapproximationisdifficulttoachieve.Tissueadhesivesarenotrecommendedforlacerationsofthehands,feet,orjoints,sincerepetitivemovementscouldcausetheadhesivebondtobreakbeforesufficienttensilestrengthisachieved.Tissueadhesivesarenotrecommendedfortheoralmucosaorothermucosalsurfacesorareasofhighmoisturesuchastheaxillaeandperineum.Lacerationsinvolvingthehairlineorvermilionborderrequiremoreprecision,andshouldberepairedwithtraditionalsutures.
83.Thetensilestrengthofwoundreaches thatoftissueby? a)6weeks b)2months c)4months d)None CorrectAnswer-DAns.is'None'Itneverequalsthetensilestrengthofunwoundedskin.Robbin'swrites-"Howlongdoesittakeforaskinwoundtoachieveitsmaximalstrength?Whensuturesareremovedfromanincisionalsurgicalwound,usuallyattheendofthefirstweek,woundstrengthisapproximately10%thatofunwoundedskin.Woundstrengthincreasesrapidlyoverthenext4weeks,slowsdownatapproximatelythethirdmonthaftertheoriginalincision,andreachesaplateauatabout70%to80%ofthetensilestrengthofunwoundedskin."
84.Thefollowingstatementaboutkeloidis true? a)Theydonotextendintonormalskin b)Localrecurrenceiscommonafterexcision c)Theyoftenundergomalignantchange d)Theyaremorecommoninwhitesthaninblacks CorrectAnswer-BAns.is'b'i.e.,LocalrecurrenceiscommonafterexcisionAkeloidscarisdefinedasexcessivescartissuethatextendsbeyondtheboundariesoftheoriginalincisionorwound.Features-Itcontinuestogetworseevenafter1yearanduptoafewyears.SevereitchingispresentMarginistenderVascular,red,erythematous(immaturebloodvessels)Extendstonormaltissues,hasaclaw-likeprocess.Hencethename.Riskfactors-BlackraceTuberculosispatientsIncisionoverthesternum,earlobeEqualinbothsexesHereditaryandfamilialVaccinationsites,injectionsitesTreatment-Injectionofsteroidpreparationsuchastriamcinoloneacetate(Kenacort)hasbeenfoundtobeextremelyuseful.
Itflattensthekeloid.Intrakeloidalexcisionandskingraftingaretobetriedlast.Recurrenceiscommon.(Anyformofexcisionhasahighchanceofrecurrence)Careshouldbetakennottoextendtheincisionontothenormalsurroundingtissues.SiliconeapplicationTopicalretinoids
85.TheVitaminwhichhasinhibitoryeffecton woundhealingis- a)Vitamin-A b)Vitamin-E c)Vitamin-C d)VitaminB-complex CorrectAnswer-BAns.is'b'i.e.,Vitamin-E.SystemicvitaminEandglucocorticoidsinhibittheinflammatoryresponseandcollagensynthesis,therebypossiblyimpedingthehealingprocess.
86.Bestskindisinfectantforcentralline insertionis: a)Povioneiodine b)Alcohol c)Cetrimide d)Chlorhexidine CorrectAnswer-DAnsis'd'i.e.Chlorhexidine"Chlorhexidineisnowthedisinfectantrecommendedforallcatheterplacementproceduresandforroutinesitecleansingduringdressingchanges."-ComprehensiveHospitalMedicine:AnEvidence-BasedAndSystemsApproachByMarkV.Williams,ScottA.Flanders,p320"Useofantisepticsolutionforskindisinfectionatthecatheterinsertionsitehelpspreventcatheter-relatedinfection.Chlorhexidine-basedsolutionsappeartobesuperiortobothaqueousandalcohol-basedpovidone-iodineinreducingtheriskforcathetercolonizationandcatheter-relatedbloodstreaminfection.Ifthereisacontraindicationtochlorhexidine,tinctureofiodine,aniodophoror70percentalcoholcanbeusedasalternatives"-uptodate.com
87.Complicationoftotalparenteralnutrition include? a)Hyperglycemia b)Hyperkalemia c)Hyperosmolardehydration d)aandb CorrectAnswer-DAns.is'a'i.e.Hyperglycemia,'b'i.e.HyperkalemiaMetaboliccomplicationAzotemiaEssentialfattyaciddef.FluidoverloadMetabolicboneds.LiverdysfunctionGlucoseimbalance(Hyperglycemia,hypoglycemia)Traceelements&vitamindeficiencyElectrolyteabnormalitiesnHypernatremia,hyponatremianHyperkalemia,hypokalemiaaHyperphosphatemia,hypophosphatemianHypermagnesemia,hypomagnesemianHypercalcemia,hypocalcemianHighserumzinc,lowserumzincnHighserumcopper,lowserumcopper
88.ContentofNa'inringerlactateismeq/1- a)154 b)12 c)130 d)144 CorrectAnswer-CAns.is'c'i.e.,130 1. 5%Dextrose/10%DxNil2. N/2saline77meq3. N/5saline30meq4. 3%salime513meq5. Ringerlactate130meq6. IsolyteP.26meq
89.Actinomycosisissensitiveto? a)Streptomycin b)Nystatin c)PenciIlin d)Iodox-uridine CorrectAnswer-CAns.is'c'i.e.,Pencillin
90.Livertransplantationwasfirstdoneby? a)Starzl b)Huggins c)Carrel d)ChristianBenard CorrectAnswer-AAns.is'a'i.e.,StarzlThefirsthumanlivertransplantwasperformedin1963byDr.ThomasStarzl.
91.TrueaboutMarjolinsulcer- a)Developsinlongstandingscar b)SqcellCadevelops c)Slowgrowinglesion d)All CorrectAnswer-DAnsis'a'i.e.Developsinlongstandingscar;'b'i.e.sqcellCadevelops;'c'i.e.SlowgrowinglesionBaghdadsoreororientalsoreorDelhiboiliscausedbyLeishmaniaTropica.
92.Ainhumisseenin? a)Baseofgreattoe b)Baseoffingerstips c)Baseoftoe d)Ankle CorrectAnswer-CAns.is'c'i.e.,BaseofToeAinhum:isaconditionusuallyaffectingNegromales(butsomefemales)whohaverunbarefootinchildhood.ClinicalFeatures:Afissureappearsatinterphalangealjointoftoe-usuallythefifth.Thisfissurebecomesafibrousband,encirclesthedigitandcausesnecrosis.
93.HowmuchlengthisincreasedinZplasty whenitisdoneat60degrees? a)25% b)50% c)75% d)100% CorrectAnswer-CAnsis'c'i.e.75% Z-plastyisaverycommoninterpositionsurgicaltechniqueutilizedinplasticand reconstructivesurgerytorevisescars. Ingeneral,thegreatertheangle,thegreaterthegaininwoundlength. TissueLengtheningwithZ-PlastyTypeofZ-Plasty IncreaseinLengthofCentralLimb(%) Simple45-degree 50 Simple60-degree 75 Simple90-degree 100 Four-flapwith60-degree 150 anglesDouble-opposing 75 Five-flap 125 OrAnglescomparedtogaininlengthareasfollows:30-degreeangleresultsina25%gaininlength45-degreeangleresultsina50%gaininlength60-degreeangleresultsina75%gaininlength75-degreeangleresultsina100%gaininlength90-degreeangleresultsina125%gaininlength
94.Theprocedureofchoicefortheevaluation ofaorticaneurysmis- a)Ultasonography b)Computedtomography c)Magneticresonanceimaging d)Arteriography CorrectAnswer-BAns.is'b'i.e.Computedtomography"CTisthemostprecisetestforimagingaorticaneurysm"-Sabiston'Asapreoperativescanningtool,CTscanisthegoldstandard.AngiographyhaslargelybeenreplacedbycontrastCTscanning"-Schwartz
95.Pseudoclaudicationiscausedby a)FemoralArterystenosis b)PoplitealArterystenosis c)Lumbarcanalstenosis d)RadialArterystenosis CorrectAnswer-CAnsisCi.e.LumbarCanalStenosisPseudoclaudicationisthetermusedforneurogenicclaudicationcausedduetolumbarcanalstenosis.ItssymptomsmimicintermittentclaudicationofPVD.Neurogenicclaudication,consistsofbackandbuttockorlegpaininducedbywalkingorstandingandrelievedbysittingorchangingposture.Symptomsinthelegsareusuallybilateral.Unlikevascularclaudication,symptomsareoftenprovokedbystandingwithoutwalking.Spinalstenosiscanbeacquired,congenital,orduetoacombinationofthesefactors.Congenitalcausesincludeachondroplasia,idiopathic.Acquiredcausesaredegenerativediseases(spondylosis,spondylolisthesis,scoliosis),trauma,spinesurgery,metabolicorendocrinedisorders(epidurallipomatosis,osteoporosis,acromegaly,renalosteodystrophy,hypoparathyroidism),andPaget'sdisease.DifferencebetweenVascularandNeurogenicclaudicationPatientswithvascularclaudicationalwayshavepainwhentheywalkarelativelyconstantdistanceonlevelground;theydonothavevariabledayswhentheycanwalkforconsiderablygreaterdistanceswithoutpain.Patientsofvascularclaudication,oftenknowexactlyhowfarorforhowlongtheycanwalkbeforethesymptomswill
occur.Thisisincontrasttopatientswithneurogenicclaudicationwherethesymptomsoccasionallyoccuratrestoratwithhighlyvariablewalkingdistances.Paininneurogenicclaudicationcanevenoccuronstanding.Painofvascularclaudicationgetsrelievedonstoppingwithinfewminutes,andthisresolutionofpainoccursevenifthepatientsimplystopsandstandsinplace.Patientswithneurogenicclaudicationusuallyhavetositdowntorelievetheirpain.Neurogenicclaudicationpaingetsrelievedbychangingpostureandleaningforwards.Patientswithneurogenicclaudicationoftencanwalkfurtheronanincline,whereasvascularclaudicationismarkedworsenedifthepatientisonanincline.
96.Kaposisarcomaiscommonlyseenin? a)Upperlimbs b)Lowerlimbs c)HeadandNeck d)Trunk CorrectAnswer-BAns.is'b'i.e.,Lowerlimbs"Itoccurspredominantlyontheskinandcaninvolvevirtuallyanyorgan,perhapsexceptthebrain.Theinitiallydescribedform,nowknownasclassicKS,predominantlyinvolvesthelowerextremitiesofelderlymen.KSisnowthemostcommontumorseeninHIV-infectedpatients"-Devita'sOncology
97.Allofthefollowingareriskfactorsfor deepveinthrombosis(DVT)except- a)Durationofsurgerymorethanthirtyminutes b)Obesity c)Agelessthanfortyyears d)Useoftheoestrogen-progesteronecontraceptivepills CorrectAnswer-CAns.is'c'i.e.,Agelessthan40years
98.DVT,investigationofchoiceis- a)Doppler b)Plethysmography c)Venography d)X-ray CorrectAnswer-AAnsweris'a'i.e.DopplerDopplerUSGisthefirstinvestigationofchoiceforDVT.
99. SurgeryinvaricoseveinsisNOT attemptedinpresenceof- a)Deepveinthrombosis b)Multipleincompetentperforators c)Varicoseveinswithlegulcer d)Alloftheabove CorrectAnswer-AAns.is'a'i.e.,DeepveinthrombosisDeepveinthrombosisisacontraindicationforvaricoseveins.Varicoseveinsurgeryshouldneverbeattemptedinacasewheredeepveinthrombosisexistsalongwithvaricoseveins,becauseinthesecasessuperficialveinsaretheonlyvalvedvenouspathwayandexcisingthemwillonlyaggravatethecondition.
100.Treatmentofribfracture? a)Immediatethoracotomy b)IPPV c)Analgesics d)bandc CorrectAnswer-CAnsweris'c'i.e.AnalgeicRibfracturesarethemostcommoninjuriesafterbluntchesttrauma.Ribs4to10areusuallyfractured.Managementof#rib-Paincontrolwithoralori.v.analgesicsorintercostalnerveblocksorepiduralanalgesia.(Poorpaincontrolsignificantlycontributestocomplicationssuchasatelectasisandpneumonia)-Cheststrappingisnolongeradvised.Managementofflailchest,asmentionedinpreviousquestion,mayneedIPPV.
101.TraumaandInjurySeverityScore (TRISS)includes: a)GCS+BP+RR b)RTS+ISS+Age c)RTS+ISS+GCS d)RTS+GCS+Age CorrectAnswer-BAnsis`b'i.e.RTS+ISS+AgeTRISS(TraumaandInjurySeverityScore):InjurySeverityScoreRevisedTraumaScoreAgeMechanismofInjury(blunt/penetrating)
102.Treatmentofchoiceforstabinjury caecum? a)Caecostomy b)Ileo-transverseanastomosis c)Transversecolostomy d)Sigmoidcolostomy CorrectAnswer-BAns.is'b'i.e.,ileo-transverseanastomosisStabandlow-velocityinjuriestothecolonwithminimalcontaminationandhemodynamicalstabilitycanbemanagedbyprimaryrepairmechanismslikeileotransverseanastomosis.
103. Brainabscessincyanoticheartdisease iscommonlylocatedin a)Cerebellarhemisphere b)Thalamus c)Temporallobe d)Parietallobe CorrectAnswer-DAns.is'd'i.e.ParietallobeBrainabscessesincongenitalcyanoticheartdiseasesoccurduetohematogenousseedingofbloodbornebacteria.Thesebloodbornebacteriabypassthepulmonarycapillarybedd/trighttoleftshunt.Theycommonlyinfectparietal&frontallobes(territoryofmiddlecerebralartery). LocationofBrainAbscessesEtiology Location Otitismedia, Temporallobe>Cerebellum mastoiditisParanasalsinusitis, Frontallobes dentalinf. Parietallobe,post-frontallobes Hematogenous (MCAterritory)
104.Allaretrueaboutcarcinomapalate, except- a)Slowgrowing b)Bilaterallymphaticspread c)Adenocarcinoma d)Presentswithpain CorrectAnswer-DAns.is'd'i.e.,Presentswithpain
105.Epulisarisesfrom- a)Enamel b)Rootofteeth c)Gingiva d)Pulp CorrectAnswer-CAnsweris'c'i.e.GingivaEpulisliterallymeans'uponthegum'.Thusitisaswellingsituatedonthegum.Itcanoriginatefromthemucousmembrane,theperiosteumorthebonegivingrisetodifferentvarietiesofEpulis.
106.Commonestsiteofcarcinomatongue- a)Apical b)Lateralborders c)Dorsum d)Posterior1/3 CorrectAnswer-BAns.is'b'i.e.,LateralbordersMostcommonsiteismiddleofthelateralborderortheventralaspectofthetongue.
107.Incaseofbenignmixedparotidtumours T/tofchoiceis- a)SuperficialParotidectomy b)Totalparotidectomy c)Leavefacialnerveandremoveallgland d)RadicalParotidectomy CorrectAnswer-AAnsis'a'ieSuperficialparotidectomySchwartzwrites-"Treatmentofbenignneoplasmsissurgicalexcisionoftheaffectedglandor,inthecaseoftheparotid,excisionofthesuperficiallobewithfacialnervedissectionandpreservation.Theminimalsurgicalprocedureforneoplasmsoftheparotidissuperficialparotidectomywithpreservationofthefacialnerve.'Shellingout'ofthetumormassisnotrecommendedbecauseoftheriskofincompleteexcisionandtumorspillage."Superficialparotidectomyisthetreatmentofchoiceformostbenigntumorsinthesuperficiallobe.
108.Trueregardingcystichygromais- a)Nontransilluminant b)Linedbycolumnarepitheliumepithelium c)Developsfromjugularlymphaticsequestration d)All CorrectAnswer-CAns.isci.e.,DevelopsfromjugularlymphaticsequestrationCystichygromaCystichygromaisaswellingusuallyoccuringinthelowerthirdoftheneckItismostcommonlyseeninposteriortriangleoftheneck,butmayalsooccurinaxilla,groin&mediastinumItresultsduetosequestrationofaportionofthejugularlymphsacfromthelymphaticsystem.Itusuallymanifestsintheneonateorinearlyinfancy(occasionallypresentatbirth)Theswellingissoftandpartiallycompressibleandinvariablyincreasesinsizewhenthechildcoughsorcries.Thecharacteristicthatdistinguishesitfromallotherneckswellingsisthatitisbrilliantlytranslucent.Thecystsarefilledwithclearlymphandarelinedbyendothelium.Mostlythesearemultiplecystsbutoccasionallytheycanbeunilocular.Itmayshowspontaneousregression.Treatment[Ref.:Sabiston18/ep2053;Schwartz9/ep1415]Therearetwomethodsoftreatment:Surgicalexcision&SclerotherapySabistonwrites-"Completesurgicalexcisionisthepreferred
treatment;however,thismaybeimpossiblebecauseofthehygromainfiltratingwithinandaroundimportantneurovascularstructures.Becausehygromasarenotneoplastictumors,radicalresectionwithremovalofmajorbloodvesselsandnervesisnotindicated.InjectionofsclerosingagentssuchasbleomycinorthederivativeofStreptococcuspvogenesOK-432havealsobeenreportedtobeeffectiveinthemanagementofcystichygromas.Intracysticinjectionofsclerosantsappearstobemosteffectiveformacrocystichygromas,asopposedtothemicrocysticvariety.""Themodernmanagementofmostcystichygromasincludesthecombinationofsurgicalexcisionandimage-guidedsclerotherapy."-Schwartz
109.Thebesttreatmentforcystichygromais - a)Surgicalexcision b)Radiotherapy c)Sclerotherapy d)Chemotherapy CorrectAnswer-AAns.is'a'i.e.,SurgicalExcisionDefinitivetreatmentiscompleteexcisionofthecystatanearlyage.
110.Sistrunk'soperationisusedin a)Parotidtumour b)Thyroglossalfistula c)Thyroglossalcyst d)bandc CorrectAnswer-DAns.Twooptionsarecorrecti.e.,'b'i.e.Thyroglossalfistula&'c'i.e.ThyroglossalcystSistrunkprocedureisusedforexcisionofthyroglossalductcyst
111.Adson'stestispositivein- a)Cervicalrib b)Cervicalspondylosis c)Cervicalfracture d)Cervicaldislocation CorrectAnswer-AAns.is'a'i.e.,Cervicalrib Adson'stestisaprovocativetestforThoracicOutletSyndromeaccompaniedbycompressionoftheSubclavianarterybyacervicalribortightenedanteriorandmiddlescalenemuscles.
112.Neuroblastomas-goodprognositcfactor is? a)N-mycamplification b)RASoncogene c)Hyperdiploidy d)Translocations CorrectAnswer-CAnsweris'c'i.e.Hyperdiploidy
113.Opsoclonus-Myoclonusisa phenomenonseenin? a)Wilmstumor b)Neuroblastoma c)Meningioma d)Corticaltuberculoma CorrectAnswer-BAns.is'b'i.e.,NeuroblastomaOpsoclonusisadisorderofeyemovementcharacterizedbyinvoluntary,chaoticsaccadesthatoccurinalldirectionsofgaze.Itisfrequentlyassociatedwithmyoclonusandataxia.Opsoclonus-myoclonusmaybecancer-relatedoridiopathic.Whenthecauseisparaneoplastic,thetumorsinvolvedareusuallycancerofthelungandbreastinadultsandneuroblastomainchildren.
114.Inthymoma,allareseenexcept- a)Hypogammaglobulinemia b)Hyperalbuminemia c)Redcellaplasia d)MyaestheniaGravis CorrectAnswer-BAnsis(b)i.e.hyperalbuminemiaThymomaisthemostcommonAnteriormediastinalmass.ThymomasareseentobeassociatedwithMyastheniagravisAcquiredhypogammaglobulinemiaPureredcellaplasiaGrave'sds-Perniciousanemia-Dermatomyositis-polymyositis-Cushingsyndrome
115. Intralobarsequestrationoflungtakesits bloodsupplyfrom- a)Internalmammaryartery b)Descendingabdominalaorta c)Pulmonaryartery d)Noneoftheabove CorrectAnswer-BAns.is'b'i.e.,DescendingabdominalaortaAsequestrationconsistsofnormallydevelopedbronchiolesandalveolisuppliedbysystemicratherthanpulmonaryarteries.ThisbloodsupplyisfromtheAortaeitheraboveorbelowthediaphragm.Mostly(approx95%)thisisfromthedescendingthoracicaorta.Othercharacteristicfeaturesofsequestration-Theyoccurmostcommonlyinthelowerlobes,L>R.Lungsequestrationsareoftwotypes
116.Foreignbodyaspirationinsupine positioncauseswhichofthefollowingpartsofthelungcommonlytobeaffected- a)Apicalleftlobe b)Apicallobeofrightlung c)Apicalpartofthelowerlobe d)Posterobasalsegmentofleftlung CorrectAnswer-CAns.is'c'i.e.ApicalpartofrightlowerlobeRightupperlobe-posteriorsegmentRightlowerlobe-superiorsegmentLeftlowerlobe-superiorsegmentRightbasilarsegments(oflowerlobe)
117.IVCfilterisusedinfollowingexcept- a)Toreducessymptoms b)Negligiblesizeofemboli c)Toprolonglife d)Topreventprogressofnativebloodvesseldisease CorrectAnswer-BAnsweris'b'i.e.Negligiblesizeofemboli
118.Empyemanecessitansisdefinedasso when? a)Pluralempyemaisunderpressure b)Pleuralempyemahasrupturedintobronchus c)Pleuralempyemahasrupturedintothepericardium d)Pleuralempyemaisshowingextensiontothesubcutaneous tissue CorrectAnswer-DAns.is'd'i.e.,Pleuralempyemaisshowingextensiontothesubcutaneoustissue
119.Allofthefollowingabout Gastrointestinalcarcinoidtumorsaretrue,Except: a)Smallintestineandappendixaccountforalmost60%ofall gastrointestinalcarcinoid b)5yearsurvivalforcarcinoidtumorsis>60% c)Rectumisspared d)Appendicialcarcinoidsaremorecommoninfemalesthan males CorrectAnswer-CAnsis'c'i.e.RectumissparedRectumisnotspared,butisinfactacommonsiteforcarinoidtumor.Aboutotheroptions:5yearsurvivalforcarcinoidtumorsis>60%Sabistonwrites-"Carcinoidtumorshavethebestprognosisofallsmallboweltumors,whetherthediseaseislocalizedormetastatic.Resectionofacarcinoidtumorlocalizedtoitsprimarysiteapproachesa100%survivalrate.Five-yearsurvivalratesareabout65%amongpatientswithregionaldiseaseand25%to35%amongthosewithdistantmetastasis."Appendicialcarcinoidsaremorecommoninfemalesthanmales"Appendicialcarcinoidsaremorecommoninfemales.Twotothreecasesofappendecialcarcinoidsarefemale."-SleisengerandFordtran'sGastrointestinalandLiverDisease8/ep609 Smallintestineandappendixaccountforalmost60%ofall gastrointestinalcarcinoidDatafromvariousbooksvarybutasfarasthequestiongoesthis
canbetakentobetrueasoption'c'isdefinitelywrong.InGITsmallintestineandtheappendixarethemostcommonsites.Infactsmallintestineisthe2ndmostcommonsiteinbodyafterbronchus.[NotethatseveraltextbooksincludingSchwartzandSabiston,mentionAppendixasthemostcommonsiteofGIcarcinoids,whichisnottrueaccordingSEERdata(giveninHarrison)]
120.Radiationexposureduringinfancyhas beenlinkedtowhichoneofthefollowingcarcinoma- a)Breast b)Melanoma c)Thyroid d)Lung CorrectAnswer-CAns.is'c'i.e.,Thyroid"Differentiatedthyroidcarcinomaparticularlypapillaryvarietyfrequentlyfollowsaccidentalirradiationofthyroidininfancyandchildhood"-BaileyRadiationinducedcancersRadiationmayinducesomenon-lethalchangesinDNAsequenceswhichmaycausemalignanttransformation
121. Smokingmaybeassociatedwithallof thefollowingcancers,except: a)CaLarynx b)CaNasopharynx c)CaBladder d)None CorrectAnswer-DAnsisNone>CaNasopharynxAllthegivenoptionsarementionedbyHarrison:Harrison17/ep2737writes-"Tobaccosmokingcausescancerofthelung,oralcavity,naso-,oro-,andhypopharynx,nasalcavityandparanasalsinuses,larynx,esophagus,stomach,pancreas,liver,kidney(bodyandpelvis),ureter,urinarybladder,anduterinecervixandalsocausesmyeloidleukemia.Thereisevidencesuggestingthatcigarettesmokingmayplayaroleinincreasingtheriskofcolorectalandpossiblypremenopausalbreastcancer,butthereisnoassociationwithpostmenopausalbreastcancer.Theredoesnotappeartobeacausallinkbetweencigarettesmokingandcanceroftheendometrium,andthereisalowerriskofuterinecanceramongpostmenopausalwomenwhosmoke."Thustheanswershouldbenone,howeverNasopharyngealcarcinomaisfoundtobeleastassociatedwithsmoking.Head&NeckCancersbyEnslowJacobs2003ep492writes-"Associationbetweensmokingandnasopharyngealcarcinomahasbeenshownonlybyafewstudies.IftheassociationoftobaccoandNPCisreal,thepossiblemechanismoftumorinductionwouldbethenitrosaminesanditsprecursorsintobacco."
122.Whatisthetreatmentofchoicein desmoidtumors? a)Irradiation b)Wideexcision c)Localexcision d)Localexcisionfollowingradiation CorrectAnswer-BAns.is'b'i.e.,Wideexcision
123.Thecommandooperationis- a)Abdomino-perinealresectionoftherectumforcarcinoma b)Disarticulationofthehipforgasgangreneoftheleg c)Extendedradicalmastectomy d)Excisionofcarcinomaofthetongue,thefloorofthemouth,part ofthejawandlymphnodesenbloc CorrectAnswer-DAns.is'd'i.e.,Excisionofcarcinomaofthetongue,thefloorofthemouth,partofthejawandlymphnodesenbloc
124.Corporaamylaciaeisseenin- a)Thymus b)Lymphnode c)Spleen d)Prostate CorrectAnswer-DAns.is'd'i.e.,Prostate
125.Stereotacticradiosurgeryisdonefor- a)Glioblastomamultiforme b)Medulloblastomaspinalcord c)Ependymoma d)AVmalformationofbrain CorrectAnswer-DAns.is'D'i.e.,AVmalformationofbrainItisalsousedfor- 1. Solitarycerebralmetastasis2. Arteriovenousmalformation3. Smallmeningiomas4. Schwannomas5. Pituitaryadenomas
126.Inwhichoneofthefollowingconditions isgasunderdiaphragmnotseen- a)Perforatedduodenalulcer b)Typhoidperforation c)Afterlaparotomy d)Spontaneousruptureofoesophagus CorrectAnswer-DAns.is'd'i.e.,SpontaneousruptureofoesophagusChilaiditi'ssyndrome:Conditioncharacterisedbyinterpositionofsmallorlargebowelbetweenliverandrightdiaphragm.Radiologicallyitgivesgasunderdiaphragm.Iatrogenicpneumoperitoneum:Certainprocedurelikeperitonealdialysis,IatrogenicallyairpushedbeforeputtingPDcannulatoavoidinjuryofviscerainsuchcasegasunderdiaphragmcanbeseen.Allcaseswhenintestineorviscerapreforatwecangetgasunderdiaphragm.
127.Allofthefollowingaretrueregarding fluidresuscitationinburnpatientsexcept: a)Considerintravenousresuscitationinchildrenwithburns greaterthan15%TBSA b)Oralfluidsmustcontainsalts c)MostpreferredfluidisRinger'slactate d)Halfofthecalculatedvolumeoffluidshouldbegiveninfirst8 hours CorrectAnswer-AAns.A:Considerintravenousresuscitationinchildrenwithburnsgreaterthan15%TBSAInchildrenwithburnsover10%TBSAandadultswith15%TBS,considertheneedforintravenousfluidresuscitation.Iforalresuscitationistobecommenced,itisimportantthatthewatergivenisnotsaltfree.Preferredfluid:LactatedRinger'sSolution,becauseitis:*Isotonic*Cheap*Easilystored-Resuscitationformulas:Parklandformulamostcommonlyused-Fluidcalculation:4xweightinkgx%TBSAburn-Give1/2ofthatvolumeinthefirst8hours.Giveother1/2innext16hours-TBSA:Totalburnssurfacearea.
128.Thefollowingisthecommonestsitefor venousulcer:March2013(a,e) a)Lowerthirdoflegandankle b)Instepoffoot c)Lower2/3rdofleg d)Middle1/3rdofleg CorrectAnswer-AAns.Ai.e.LowerthirdoflegandankleVenousulcersusuallyliejustproximaltothemedialorlateralmalleolus.Venousulcersareaccompaniedbylipodermatosclerosisandhemosiderosis(ifthesearenotpresentthentheulcerisprobablynotofvenousorigin).
129.Whichofthefollowingissparedin lumbarsympathectomy:September2009 a)L1 b)L2 c)L3 d)L4 CorrectAnswer-AAns.A:LlTopreservesexualfunctions,Llispreserved.
130.Brodie-Trendelenburgtestispositivein: a)Deepveinthrombosis b)Sapheno-femoralincompetence c)Thromboangiitisobliterans d)Below-kneeperforatorsincompetence CorrectAnswer-BAns.B:Sapheno-femoralIncompetenceAtesttoassessthecompetenceofthesaphenofemoraljunction.TheBrodie-TrendelenburgtestisusedtodetectvenousincompetenceandtodifferentiatebetweenperforatorandGSVincompetence.TheBrodie-Trendelenburgtestishighlysensitivefortheidentificationofsuperficialandperforatorreflux.SFJ(saphenofemoraljunction)incompetenceisdiagnosedifthedistalveinsfillrapidlyuponreleaseofthetourniquet.SometextbooksrefertotheTrendelenburgtestandthetourniquettestinterchangeably.
131.Cullen'ssignisseenin: March2004 a)Acutecholecystitis b)Acutehepatitis c)Acutepancreatitis d)Bluntinjuryabdomen CorrectAnswer-CAns.Ci.e.Acutepancreatitis
132.Acutepancreatitiscausesallofthe followingexcept:March2005,March2013(g) a)Hypercalcemia b)Increasedamylaselevel c)Subcutaneousfatnecrosis d)Hyperlipidemia CorrectAnswer-AAns.A:HypercalcemiaComplicationsofacutepancreatitis:Intra-abdominal:Pancreatic"collections":PhlegmonPseudocystAbscessNecrosisIntestinal:ParalyticileusGastrointestinalhaemorrhage-fromstressulceration,gastricvaricesduetosplenicveinobstruction,orruptureofpseudoaneurysmNecrotisingobstructionorfistulisationofcolon*Hepatic-obstructivejaundiceduetooedematousnarrowingofcommonbileduct*Renal-hydronephrosisandhydroureterofrightkidneyduetoperipancreaticinflammationinperirenalspace*Spleen-ruptureorhaematoma,fromspreadofperipancreatic
inflammationSystemiccomplicationsrangefromminorpyrexiatorapidlyfatal,multipleorgan-systemfailure.Theyinclude:*Circulatoryshock-kininactivation,haemorrhage*Disseminatedintravascularcoagulation*Respiratoryinsufficiency:Milde.g.hypoxaemia,atelectasis,pleuraleffusionSeveree.g.adultrespiratorydistresssyndrome*Acuterenalfailure*Metabolic:HypocalcaemiaHyperglycaemia/diabetesmellitusHypertriglyceridaemia*Pancreaticencephalopathy-confusions,delusions,coma*RetinalarteriolarobstructioncausingsuddenblindnessPurtscher'sretinopathy*MetastaticfatnecrosisAserumlevelthreetofourtimesabovenormalserumamylaselevel,isindicativeofacutepancreatitis,butanormalleveldoesnotexcludethedisease.
133.Charcot'striadisdefinedbyallofthe followingexcept:September2007,March2009 a)Fever b)Gallstones c)Jaundice d)Pain CorrectAnswer-BAns.B:GallstonesCholangitisisduetopartialorcompleteobstructionofthebiliarytreewithresultingbilestasisandsecondarybacterialormicrobialinfectionofthebiliarytreeCauses?CommonbileductstonesBenignbiliarystricture(primarysclerosingcholangitis)Malignancy(headofpancreasadenocarcinoma,ampullaofVater,bileducttumors)ChronicpancreatitisProsthesisorstentsinthecommonbileductFeatures:Charcot'striad-rightupperquadrantpain,fever,jaundiceReynolds'pentad-rightupperquadrantpain,fever,jaundice,hypotension,andmentalstatuschanges(delirium,anxiety,andcoma)Nauseaand/orvomitingRightupperquadranttenderness(mildtomoderate)
134.Mostsensitiveandspecificforacute pancreatitisamongstthefollowingis:September2008 a)S.amylase b)S.Alaninetransaminase c)S.lipase d)C-reactiveprotein CorrectAnswer-CAns.C:S.lipaseSerummarkersfordiagnosisofacutepancreatitis:Alaninetransaminase-Associatedwithgallstonepancreatitis;threefoldelevationorgreaterinthepresenceofacutepancreatitishasapositivepredictivevalueof95percentindiagnosingacutegallstonepancreatitisAmylaseMostaccuratewhenatleasttwicetheupperlimitofnormal;amylaselevelsandsensitivitydecreasewithtimefromonsetofsymptomsC-reactiveproteinLatemarker;highlevelsassociatedwithpancreaticnecrosisLipaseIncreasedsensitivityinalcohol-inducedpancreatitis;morespecificandsensitivethanamylasefordetectingacutepancreatitisPhospholipaseA2AssociatedwithdevelopmentofpancreaticnecrosisandpulmonaryfailureProcalcitonin
Earlydetectionofseverity;highconcentrationsininfectednecrosisTrypsinogenactivationpeptideEarlymarkerforacutepancreatitisandclosecorrelationtoseverity
135.Treatmentofanincidentallydetected Appendicularcarcinoidmeasuring2.5cmis:September2002 a)Righthemicolectomy b)Limitedresectionoftherightcolon c)Totalcolectomy d)Appendicectomy CorrectAnswer-AAns.Ai.e.Righthemicolectomy
136.Heller'smyotomyisdonefor: September2007,2009,2010 a)Esophagealcarcinoma b)Pylorichypertrophy c)Achalasiacardia d)Inguinalhernia CorrectAnswer-CAns.C:AchalasiacardiaAchalasiaisassociatedwithlossofganglioncellsintheesophagealmyentericplexus.TheseimportantinhibitoryneuronsinduceLESrelaxationandcoordinateproximal-to-distalperistalticcontractionoftheesophagusAchalasiaisanesophagealmotordisordercharacterizedbyincreasedloweresophagealsphincter(LES)pressure,diminished-to-absentperistalsisinthedistalportionoftheesophaguscomposedofsmoothmuscle,andlackofacoordinatedLESrelaxationinresponsetoswallowing.Bariumradiologymayshow'bird'sbeak'appearance.Esophageal(Heller)myotomyisasurgicalprocedurethatisperformedwithminimallyinvasivetechniques.Thelaparoscopicapproachappearstobemostappropriate.
137.Allofthefollowingtesticulartumours aregermcelltumoursEXCEPT:March2013(b) a)Seminoma b)Teratoma c)Choriocarcinoma d)Sertolicelltumour CorrectAnswer-DAns.Di.e.SertolicelltumourTesticularcarcinomaBilateralin10%ofcases,AbdominalcryptorchidtestesareathigherriskascomparedtoinguinalcryptorchidtestesTesticularfeminizationsyndromeincreasestheriskoftesticulargermcelltumourMCCaoftestesinyoung:SeminomaMCCainelderly:LymphomaMCCaininfants:YolksactumourSeminomaRadiosensitive,Correspondstodysgerminomaofovary&Treatmentofchoiceis:SurgeryGermCellTumorsPrecursorlesionIntratubulargermcellneoplasm,unclassifiedIntratubulargermcellneoplasm,specifictypeTumorsof1histologictype
SeminomaVariant:SeminomawithsyncytiotrophoblasticcellsPartiallyregressedtumorshowingseminomawithscarSpermatocyticseminomaEmbryonalcarcinomaVariant:Spermatocyticseminomawithasarcomatouscomponent Yolksactumor ChoriocarcinomaPlacentalsitetrophoblastictumorVariant:"Monophasic"type Trophoblastictumor,unclassified Teratoma Withasecondarysomatictypemalignantcomponent Monodermalvariants Carcinoid Primitiveneuroectodermaltumor Others
138.Cobraheadappearanceonexcretory urographyissuggestiveof:March2010 a)Horseshoekidney b)Duplicationofrenalpelvis c)Simplecystofkidney d)Ureterocele CorrectAnswer-DAns.D:UreteroceleThetermureteroceledenotesacysticballooningofthedistalendoftheureter.Thistypeofureteroceleisalsotermedorthotopic,sinceitarisesfromaureterwithanormalinsertionintothetrigone.Anintravesicalureteroceleresultsfromtheprolapseofthemucosaoftheterminalsegmentoftheureterthroughtheureterovesicalorificeintothebladder.Thisprolapsedureteralmucosacarrieswithitaportionofthecontinuoussheetofthebladdermucosaaroundtheorifice.Theprolapsedsegmentthushasawallthatconsistsofathinlayerofmuscleandcollageninterposedbetweenthebladderuroepitheliumandtheureteruroepithelium.Sincetheterminalureteralorificeisusuallynarrowedandpartiallyobstructed,andsincethereisnomusclesupportforthedoublemucosalwallsoftheprolapsedsegment,itdilates.Thisdilatedsegmentfillswithurineandprotrudesintothebladder.Onexcretoryurography,cobraheadsignisclassicallyseenwithanintravesicalureterocele.
139.Orchidopexyforincompletelydescended testisisdoneaftertheageof:September2011 a)Atbirth b)1year c)2years d)5years CorrectAnswer-BAns.B:1yearOrchidopexyisusuallyperformedaftertheageof1yeartoavoidtheriskofoperatingonatinypatientRemember:IncompletedescentoftestesmayincreasedliabilitytomalignantchangeAlltypesofmalignanttesticulartumoursaremorecommoninincompletelydescendedtesteseveniftheyhavebeenbroughtdownsurgically
140.MCsiteforthyroglossalcystis: a)Beneaththeforamencaecum b)Floorofmouth c)Abovehyoid d)Subhyoid CorrectAnswer-DAns.Di.e.SubhyoidSitesofthyroglossalcyst 1. Subhyoid:Themostcommontype2. Atthelevelofthyroidcartilage:2ndcommonsite3. Suprahyoid:Doublechinappearance4. Attheforamencaecum:Rare5. Atthelevelofcricoidcartilage:Rare6. Inthefloorofthemouth
141.TrueaboutBranchialcystis: March2013(h) a)Cystsaremorecommonthansinuses b)Mostlyarisesfrom2ndbranchialsystem c)Causesdysphagiaandhoarseness d)Sinusshouldalwaysbeoperated CorrectAnswer-BAns.Bi.e.Mostlyarisesfrom2ndbranchialsystem
142.AllofthefollowingarefalseforGall Bladdercarcinomaexcept:March2005 a)Carriesagoodprognosis b)Gallstonesmaybeapredisposingfactor c)Commonlysquamouscellcarcinoma d)Jaundiceisrare CorrectAnswer-BAns.B:GallstonesmaybeapredisposingfactorPrimarycarcinomaofthegallbladderisanuncommon,aggressivemalignancythataffectswomenmorefrequentlythanmen.Olderagegroupsaremostoftenaffected,andcoexistinggallstonesarepresentinthevastmajorityofcases.Thesymptomsatpresentationarevagueandaremostoftenrelatedtoadjacentorganinvasion.Imagingstudiesmayrevealamassreplacingthenormalgallbladder,diffuseorfocalthickeningofthegallbladderwall,orapolypoidmasswithinthegallbladderlumen.Adjacentorganinvasion,mostcommonlyinvolvingtheliver,istypicallypresentatdiagnosis,asisbiliaryobstruction.Periportalandperipancreaticlymphadenopathy,hematogenousmetastases,andperitonealmetastasesmayalsobeseen.Thevastmajorityofgallbladdercarcinomasareadenocarcinomas.Becausemostpatientspresentwithadvanceddisease,theprognosisispoor.Theradiologicdifferentialdiagnosisincludesthemorefrequentlyencounteredinflammatoryconditionsofthegallbladder,xanthogranulomatouscholecystitis,adenomyomatosis,otherhepatobiliarymalignancies,andmetastaticdisease.
hepatobiliarymalignancies,andmetastaticdisease.Theprevalenceoflymphaticspreadishighingallbladdercarcinoma.Lymphaticmetastasesprogressfromthegallbladderfossathroughthehepatoduodenalligamenttonodalstationsneartheheadofthepancreas.Threepathwaysoflymphaticdrainagehavebeensuggested:thecholecystoretropancreaticpathway,thecholecystoceliacpathway,andthecholecystomesentericpathway.Thecysticandpericholedochallymphnodesarethemostcommonlyinvolvedatsurgeryandareacriticalpathwaytoinvolvementoftheceliac,superiormesenteric,andpara-aorticlymphnodes.
143.Cryoprecipitatecontains: March2009 a)FactorII b)FactorV c)FactorVIII d)FactorIX CorrectAnswer-CAns.C:FactorVIIICryoprecipitateispreparedfromplasmaandcontainsfibrinogen,vonWillebrandfactor,factorVIII,factorXIIIandfibronectin.Cryoprecipitateistheonlyadequatefibrinogenconcentrateavailableforintravenoususe.Cryoprecipitateisindicatedforbleedingorimmediatelypriortoaninvasiveprocedureinpatientswithsignificanthypofibrinogenemia(CryoprecipitateshouldnotbeusedforpatientswithvonWillebranddiseaseorHemophiliaA(FactorVIIIdeficiency).ItisnotusuallygivenforFactorXIIIdeficiency,astherearevirus-inactivatedconcentratesofthisproteinavailable.Cryoprecipitateissometimesusefulifplateletdysfunctionassociatedwithrenalfailuredoesnotrespondtodialysis.
144.Adsontestispositivein: September2007 a)Cervicalspondylosis b)Cervicalrib c)Cervicalvertebrafracture d)Superiorvenacavasyndrome CorrectAnswer-BAns.B:CervicalribAdson'stestisusedtoassessforthepresenceofThoracicOutletSyndrome(cervicalrib)atthescalenetriangle.
ProcessThepatientisplacedinasittingposition,handsrestingonthighs.TheexaminerpalpatesradialpulseonsidebeingtestedPatientactivelyrotatesheadtoipsilateralsidebeingtestedwhiletheexaminerlaterallyrotatesandextendsthepatient'sshoulderPatienttakesadeepbreathandisinstructedtoholditTheexaminerpalpatestheradialpulsewhilemovingtheupperextremityinabduction,extension,andexternalrotation.Thepatientthenisaskedtorotateherheadtowardtheinvolvedsidewhiletakingadeepbreathandholdingit.Apositiveexamwillresultinadiminishedorabsentradialpulse.
145.
Drugsforparalyticileusforbowelresectionsurgeryareallexcept? a)Alvinopam b)Dihydroergotamine c)Naloxone d)Methylnaltrexone CorrectAnswer-CAns.is'c'i.e.,NaloxonePharmacologicManagementofPostopparalyticileus(P01):Minimizingthesympatheticinhibitionofgastrointestinalmotility,decreasinginflammationandstimulationofgastrointestinal11-opioidreceptorsaretheultimategoalsofpharmacologicmanagement.A)MinimizingsympatheticinhibitionBothpropranolol,anonspecific13-receptorantagonist,anddihydroergotamine,ana-receptorantagonist,havebeeninvestigatedfortreatmentofPOI.Neostigmineisanacetylcholinsteraseinhibitorthatcausesanincreaseincholinergic(parasympathetic)activityinthegutwall,whichisbelievedtotherebystimulatecolonicmotility.Useofedrophoniumchlorideandbethanecholchloride,whichcompetitivelyinhibitacetylcholineonthebindingsiteofacetylcholinesterase,hasbeenreportedtoshowimprovementofPOI.Cisaprideisaserotonin(5-HT)4receptorantagonistthatpromotesacetylcholinereleasefrompostganglionicnerveendingsinthemyentericplexusandisthoughttoindirectlyimprovegastrointestinalmotility.Metocloprimideissuspectedtoenhancegastrointestinalmotility
withoutstimulatinggastricsecretion,butitsusehasnotbeensubstantiatedforPOI.B)DecreasinginflammationDecreasinginflammationmaybeindicatedinpatientswhoareabouttoundergomajorintestinalsurgery,asthisisthoughttobeanimportantcontributingfactortoPOI.Nonsteroidalanti-inflammatory(NSAIDs)agentscanbeusedinconjunctionwithopioidanalgesicsfortheirdualeffectsonpaincontrolandinflammatoryinhibition.C)Stimulationofgastrointestinaliii-opioidreceptorsStimulationofgastrointestinal.1-opioidreceptorscantheoreticallyinfluencegastrointestinalmotilitydirectly;therefore,blockingtheperipheralgastrointestinaleffectsofcentrallyactingopioidsusedforanalgesiamayhelppreventPOI.Twonoveldrugsarebeinginvestigatedforthisreason:alvimopanandmethylnaltrexone.BothdrugsareIf-opioidreceptorantagonists,andbothappeartoofferpromisingresultsforpreventingprolongedPOI.Opioidtherapyforpostoperativeorchronicpainisfrequentlyassociatedwithadverseeffects,themostcommonbeingdose-limitinganddebilitatingboweldysfunction,soalvimopanandmethylnaltrexonemayalsobeusefulinthetreatmentofchronicopioidboweldysfunction.Thecurrentlyavailableopioidantagonistssuchasnaloxoneareoflimitedusebecausetheyalsoactatcentralopioidreceptorstoreverseanalgesiaandelicitopioidwithdrawal.Alvimopanandmethylnaltrexoneareperipherallyactingif-opioidreceptorantagoniststhathavebeenstudiedinpatientsundergoingabdominalandpelvicsurgeryandhavebeenshowninseveralstudiestosignificantlyaccelerategastrointestinalrecovery.AlvimopanreceivedFDAapprovalforthetreatmentofPOIonMay20,2008.D)AlternativemedicationsBisacodyladministrationversusplacebotwicedailystartingonpostoperativeday1,patientswhoreceivedbisacodylhadsignificantlyearlierbowelmovementsthanthosewhoreceivedplacebo(25hv.56h),butfurtherstudiesareneededtoassessthe
effectoflaxativesonPOI.
146.Anadulthypertensivemalepresented withsuddenonsetsevereheadacheandvomiting.Onexamination,thereismarkedneckrigidityandnofocalneurologicaldeficitwasfound.Thesymptomsaremostlikelydueto: a)Intracerebralparenchymalhemorrhage b)Ischemicstroke c)Meningitis d)Subarachnoidhemorrhage CorrectAnswer-DAns.d.SubarachnoidhemorrhageThemostlikelydiagnosishereisasubarachnoidbleed(SAH).Themostcommoncauseistraumaandismanagedconservatively.Thesecondmostcommoncauseisaruptureofaberryaneurysm.Thetypicalpresentationofasubarachnoidhemorrhageincludesa`thunderclap'headache.Meningiticfeaturesofneckstiffnessandphotophobiaoftendevelopoverhours.Computedtomography(CT)istheinvestigationofchoice.LumbarpunctureshouldbeperformediftheCTscanfailstoestablishthediagnosisofSAH;itshowsaxanthochromicbloodpicture.Delayedischemicneurologicaldeficit(DIND)isattributedtovasospasmofthecerebralvasculaturetypicallydeveloping3?10daysfollowingictus.Itisthemaincauseofapooroutcome.
Endovasculartreatment("coiling")isgenerallypreferredovercraniotomyandclippingforaneurysmsamenabletothisapproach.
147.Gaslessabdomenseenin- a)Ulcerativecolitis b)Acutepancreatitis c)Intussusception d)Necrotisingenterocolitis CorrectAnswer-BAnswer-B(acutepancreatitis)Commoncausesinclude:Inadult: -Highobstruction. -Ascites. -Acutepancreatitisduetoexcessvomiting. -Fluidfilledintestine. -Largeabdominalmass.ACUTEPANCREATITIS-radiologysignsrenalhalosigngaslessabdomengroundglassappearancecoloncutoffsignsentinelloop
148.Dohlmanprocedurefor- a)Meckel'sdiverticulum b)Zenker'sdiverticulum c)Dermatomyositis d)Menetrier'sdisease CorrectAnswer-BAnswer-B(Zenker'sdiverticulum)Thetreatmentofpharyngealpouches(Zenker'sDiverticulum)maybebyeitheropensurgicalorendoscopictechniques.TheendoscopicDohlman'sprocedureisanidealtechniqueintheelderly.
149.Ifamotherisdonatingkidneytoherson isanexample- a)Isograft b)Allograft c)Autograft d)Xenograft CorrectAnswer-BAnswer-B(Allograft)Allograft?Graftingbetweentwonon-identicalmemberofsamespeciesbutnotsamegenotypes.Itincludesthetransplantationofheart,kidney,lungetc,fromamemberswhodonatetheirorgans.Anti?rejectiondrugsorimmunosuppressantneedtobetakentopreventthebodyfromrejectingatransplantedorgan.Mosthumantissueandorgantransplantsareallografts.
150.Uvulavesicaeisproducedbywhich prostatelobe? a)Anteriorlobe b)Postlobe c)Medianlobe d)Laterallobe CorrectAnswer-CAnswer-C(medianlobe)Themucousmembraneimmediatelybehindtheinternalurethralorificepresentsaslightelevation,theuvulaofurinarybladder,causedbythemedianlobeoftheprostate.Itisoftenenlargedinbenignprostatichypertrophy.
151.TrueaboutBarrett'sesophagusareallof thefollowingexcept: a)Causesadenocarcinoma b)Patientisusuallyasymptomatic c)Histologyofthelesionshowsmucussecretinggobletcells d)Chronicgastroesophagealrefluxisapredisposingfactor CorrectAnswer-AAnswer-Barrettesophagusisapremalignantconditionthatinvolvesthedistalesophagusandappearstoberelatedtochronicgastroesophagealreflux(GORD).Thereismetaplasticchangeintheliningmucosaofesophagus.PatientisusuallyasymptomaticinacaseofBarrett'sEsophagus.HistologyofthelesioninacaseofBarrett'sesophagusshowsmucussecretinggobletcells.DiagnosisofBarrett'sesophagusismadebydemonstrationofcolumnarmucosa,whichonhistopathologyshowsIntestinaltypeofmetaplasiaChronicrefluxisacasueofBarrett'sEsophagus.
152.Parathyroidautoimplantationtakesplace inwhichofthemuscle? a)Biceps b)Triceps c)Brachioradialis d)Sartorius CorrectAnswer-CAnswer-C(Brachioradialis)Atotalparathyroidectomywithaforearmautograftinvolvesremovalofallparathyroidtissueintheneck,withreimplantationofasmallamountofmorcellatedtissuewithinapocketformedinthebrachioradialismuscle.
153.Bell'spalsyisassociatedwiththelesion inwhichofthenerve? a)11thcranialnerve b)7thcranialnerve c)9thcranialnerve d)3rdcranialnerve CorrectAnswer-BAnswer-B(7thcranialnerve)Bell'spalsyoccursduetoamalfunctionofthefacialnerve(VIIcranialnerve),whichcontrolsthemusclesoftheface.Facialpalsyistypifiedbyinabilitytocontrolmovementinthemusclesoffacialexpression.
154.Cushingulcerisseenincaseof- a)Burns b)Headinjury c)Cellnecrosis d)Stress CorrectAnswer-BAnswer-B(Headinjury)Cushingulcersareassociatedwithabraintumororheadinjuryandtypicallyaresingle,deepulcersthatarepronetoperforation.Itisagastriculcerassociatedwithelevatedintracranialpressure.ItisalsocalledvonRokitansky?Cushingsyndrome.ThemechanismofdevelopmentofCushingulcersisthoughttobeduetodirectstimulationofvagalnucleiasaresultofincreasedintracranialpressure.Braintumors,traumaticheadinjury,andotherintracranialprocessesincludinginfections,cancauseincreasedintracranialpressureandleadtooverstimulationofthevagusnerve.
155.Mostcommonindicationforliver transplantinchildren- a)Biliaryatresia b)Cirrhosis c)Hepatitis d)Drugreactions CorrectAnswer-AAnswer-A(Biliaryatresia)Indicationsforlivertransplantationininfantsandchildrenincludeacuteliverfailure(ALF),chronicliverfailurewithpruritus,complicationsofcholestasisandfailuretothrive.Inyoungchildren,themostcommonliverdiseaseleadingtotransplantationisbiliaryatresia
156.Mostcommontypeofgallstoneis- a)Mixedstones b)Purecholesterolstones c)Pigmentstones d)Calciumbilirubinate CorrectAnswer-AAnswer-AMixed-Between4%and20%ofstonesaremixed.Pigment-Between2%and30%ofstonesarebilirubinstones.Cholesterol-Between35%and90%ofstonesarecholesterolstones.Themostcommontypeofgallstoneswasmixedcholesteroltypegallstoneswith67.5%followedbyblackpigmentandbrownpigmenttypesas23.83%and5.89%,respectively.Mixedstonesare90%common.Itcontainscholesterol,calciumsaltsofphosphatecarbonate,palmitate,proteins,andaremultiplefaceted.
157.Serpiginousulcerdistalesophagus- a)CMV b)Herpes c)Pill d)Corrosive CorrectAnswer-AAnswer-A(CMV)CMVclassicallycausesserpiginousulcersinthedistalesophagusthatmaycoalescetoformgiantulcers.
158.Omphaloceleiscausedby? a)Duplicationsofintestinalloops b)Abnormalrotationoftheintestinalloop c)Failureofguttoreturntothebodycavityfromitsphysiological herniation d)Reversedrotationoftheintestinalloop CorrectAnswer-CAnswer-C(Failureofguttoreturntothebodycavityfromitsphysiologicalherniation)Failureofguttoreturntothebodycavityfromitsphysiologicalherniation.Exomphalos(omphalocele)isherniationofabdominalviscerathroughanenlargedumbilicalring.Theviscera,whichmayincludeliver,smallandlargeintestines,stomach,spleen,orbladder,arecoveredbyamnion.Theoriginofomphaloceleisafailureoftheboweltoreturntothebodycavityfromitsphysiologicalherniationduringthe6thto10thweeks.
159.Whichofthefollowingistrueabout coeliacplexusblock? a)LocatedretroperitoneallyatthelevelofL3 b)Usuallydoneunilaterally c)Usefulforthepainfulconditionsoflowerabdomen d)Mostcommonsideeffectisdiarrheaandhypotension CorrectAnswer-DAnswer-D(Mostcommonsideeffectisdiarrheaandhypotension)CeliacPlexusBlock:LocatedretroperitoneallyatthelevelofLlUsuallydonebilaterallvUsefulforthepainfulconditionsofupperabdomenMostcommonsideeffectisdiarrheaandhvpotension
160.Mostcommonlyperformedand acceptablemethodofbariatricsurgeryis: a)Biliopancreaticdiversion b)Biliopancreaticdiversionwithilcostomy c)Laparoscopicgastricbanding d)Roux-en-Ygastricbypass. CorrectAnswer-DAns:D.Roux-en-Ygastricbypass.(Ref.Sabiston20/ep1187,19/ep363;Schwartz10/ep1112,9/952;Harrison19/p2398).Roux-en-Ygastricbypass:Mostcommonlyperformed&acceptablemethodofbariatricsurgery.3restrictive-malabsorptivebypassprocedures:Combineelementsofgastricrestriction&selectivemalabsorption.ProceduresincludeRoux-en-Ygastricbypass,biliopancreaticdiversion&biliopancreaticdiversionwithduodenalswitch.Roux-en-Y:Mostcommonlyundertaken&mostacceptedbypassprocedure.Performedwithanopenincisionorbylaparoscopy.

161.Apatientafterroadtrafficaccident presentedwithtensionpneumothorax.Whatisthefirstlineofmanagement? a)Insertwideboreneedlein2ndintercostalspace b)ImmediatechestX-ray c)CTscanEmergencythoracotomy d)Emergencythoracotomy CorrectAnswer-AAnswer-A.Insertwideboreneedlein2ndintercostalspaceFirstlineofmanagementintensionpneumothorax:Insertwideboreneedlein2ndintercostalspace.
162.Prehnsignispositivein a)Acuteepidydimoorchitis b)Chronicorchitis c)Testiculartorsion d)None CorrectAnswer-AAnswer-A.AcuteepidydimoorchitisOnelevationoftestisthe-Painisnotrelievedintorsion(testisnegative)Painrelievedinepidydimoorchitis(testispositive)
163.
Definitivesurgeryisapartofwhichstageofdamagecontrolsurgery? a)I b)II c)III d)IV CorrectAnswer-DAnswer-D.IVFollowingmajorinjury,protractedsurgeryinthephysiologicallyunstablepatientwiththe'deadlytriad'-thecombinationofhypothermia,acidosisandcoagulopathy'Damagecontrol'or'damagelimitationsurgery'isaconceptthatoriginatedfromnavalarchitecture,wherebyashipwasdesignedtohaveareassealedoffinthecaseofdamage,tolimitflooding.Stages- 1. Patientselection2. Controlofhaemorrhageandcontrolofcontamination3. Resuscitationcontinuedintheintensivecareunit4. Definitivesurgery5. Abdominalclosure
164.Whichofthefollowingisnottrueabout Boerhaavesyndrome? a)Perforationoftheesophagusduetobarotrauma b)Mostcommonsiteisleftposteromedialaspect3-5cmsabove thegastroesophagealjunction c)Painistheearlymanifestation d)Mostcasesfollowaboutofheavyeatingordrinking CorrectAnswer-BAnswer-B.Mostcommonsiteisleftposteromedialaspect3-5cmsabovethegastroesophagealjunctionVomitingisthoughttobethemostcommoncause,othercausesincludeweightlifting,defecation,epilepticseizures,abdominaltrauma,compressedairinjury,andchildbirth,allofwhichcanincreasethepressureintheesophagusandcauseabarogenicesophagealrupture.Twocommonriskfactorsincludealcoholismandexcessiveindulgenceinfood.Ruptureoccursmostcommonlyintheleftposterolateralwallofthedistalthirdoftheesophaguswithextensionintotheleftpleuralcavity.Symptomsconsistofvomiting,lowerthoracicpain,andsubcutaneousemphysema.Typically,thepatientwillpresentwithpainatthesiteofperforation,usuallyintheneck,chest,epigastricregion,orupperabdomen.
165.Allofthefollowingareindicationsfor bariatricsurgeryexcept- a)BMI>40kg/m2 b)BMI>35kg/m2withatleastonecomorbidity c)BMI>30withlongstandingdiabetes d)Failureofothermethodsofweightloss CorrectAnswer-CAnswer-C.BMI>30withlongstandingdiabetesIndicationsforbariatricswgeryBMI>40kg/m2BMI>35kg/m2withatleastonecomorbidityPatientathighriskofobesityassociatedmorbidityandmortalityFailureofothermethodsofwightloss
166.Indicationsforemergencythoracotomy areallofthefollowingexcept a)Majortracheobronchialinjuries b)Cardiactamponade c)Penetratinginjuriestoanteriorchest d)TensionPneumothorax CorrectAnswer-DAnswer-D.TensionPneumothoraxEmergencythoracotomyisindicatedafterchesttraumainfollowingconditions:Cardiacarrest(resuscitativethoractomy)Massivehemoththorax(>1500mLofbloodthroughthechesttubeacutelyor>200-300ml/hrafterinitialdrainage.Penetratinginjuriesoftheanterioraspectofthechestwithcardiactamponade.Largeopenwoundsofthethoraciccage.Majorthoracicvascularinjuriesinthepresenceofhemodynamicinstability
167.Afemaleundergonesurgeryforleft breastcancer3yrsbacknowdevelopedbluenoduleonsameside a)Lymphangiosarcoma b)Recurrence c)Hemangioma d)Cellulitis CorrectAnswer-AAnswer-A.LymphangiosarcomaAngiosarcomaisavasculartumorwhichmayarisedenovointhebreastorasacomplicationoftheradiationtherapy.
168.BloodlossinclassIIIhemorrhagicshock - a)<750ml b)750-1500ml c)1500-2000ml d)>2000ml CorrectAnswer-CAns.is'c'i.e.,1500-2000ml Parameters ClassI ClassII ClassIII ClassIV BloodLoss(mL) Upto750 750?1500 1500?2000 >2000 BloodLoss(%BV) Upto15% 15?30% 30?40% >40% Pulserate <100 >100 >120 >140 (beats/min) BloodPressure Normal Minimal Decreased Significantlydecreased decrease PulsePressure Normal Narrowed Narrowed Unobtainableorverynarrow Hourlyurine 0.5mL/kg 0.5mL/kg <0.5cc/kg Minimal output CNS/Mental Slightly Mildlyanxious Anxiousand Confusedorlethargic status anxious confused
169.Mostcommoncancerinmenis a)Bladdercancer b)Colorectalcancer c)Prostatecancer d)Oralcancer CorrectAnswer-CAnswer-C.ProstatecancerCancersinmalesinIndia:Lip/oralcavity>Prostate>Colorectum>Pharynx(otherthannssopharynx)>LarynxLungcancerwasthemostcommoncancerinmenworldwide,contributing15.5%ofthetotalnumberofnewcasesdiagnosedin2018.Thetopthree?lung,prostateandcolorectalcancers?contributed44.4%ofallcancers(excludingnon-melanomaskincancer).
170.
Anerveinjuredinradicalneckdissectionleadstolossofsensationinmedialsideofthearm,nerveinjuredis? a)Longthoracicnerve b)Thoracodorsalnerve c)Dorsalscapularnerve d)Medialcutaneousnerveofarm CorrectAnswer-DAnswer-D.MedialcutaneousnerveofarmThemedialcutaneousnerveofthearmisthesmallestandmostmedialbranchofhebrachialplexus,andarisesfromthemedialcord.Itpiercesthedeepfasciaatthemidpointoftheupperarmtosupplytheskinoverthemedialaspectofthedistalthirdoftheupperarm.
171.Goldstandardinvestigationforchronic pancreatitis? a)MRI b)ERCP c)Pancreaticfunctiontests d)Fecalfatestimation CorrectAnswer-BAnswer-B.ERCPERCPhasbeenconsideredthemostsensitiveradiologictestforthediagnosisofchronicpancreatitis,withspecificERCPfindingsthatarehighlycorrelativewiththedegreeorstageofchronicdisease.
172.Mostaccuratemethodforthediagnosis GastroesophagealRefluxDisease(GERD)is a)Histologicalstudy b)Manometry c)24-hourpHrecordingandelectricalimpedancemeasurement d)Bariumswallowstudies e)UpperGIendoscopy CorrectAnswer-CAnswer-C.24-hourpHrecordingandelectricalimpedancemeasurementThemostsensitivetestfordiagnosisofGERDis24-hambulatorypHmonitoring.Endoscopyisindicatedinpatientswithrefluxsymptomsrefractorytoantisecretorytherapy;inthosewithalarmingsymptomssuchasdysphagia,weightloss,orgastrointestinalbleeding;andinthosewithrecurrentdyspepsiaaftertreatmentthatisnotclearlyduetorefluxonclinicalgroundsalonepHwaslessthan4
173.AllarefeaturesofSIRSexcept- a)RR>24&Paco2<22mmhg b)WBC>11or<4 c)Temperature<36and>38 d)PR>90 CorrectAnswer-AAnswer-A.RR>24&Paco2<22mmhg
174.Whichofthefollowingprimarilygoverns theuptakeofTc-99mMDPinbody? a)Amountofosteogenicactivity b)Amountofiodineuptake c)Amountofcalciumuptake d)Amountofcatecholamineactivity CorrectAnswer-AAnswer-A.AmountofosteogenicactivityTechnetium99-misacommonlyusedradiopharmaceutical.Technetium99-mmethylenediphosphonate(Tc-99mMDP),desirableforthegammacarmeraimagingisthecommonlyusedform.Tc-99mMDPcanbepreparedfromakitcontaining,sodiumpertechnitate(NaTcO4)vial,MDP,stabilizersandstannousion.
175.Technitium-99mpertechnetatelabelled methylenediphosphonateisstructurallysimilarto a)Calciumphosphate b)Phosphorus c)Sodiumbicarbonate d)Magnesiumsulfate CorrectAnswer-AAnswer-A.CalciumphosphateTheradionucletideadministeredisTechnitium-99mpertechnetatelabeledmethylenediphosphonateisananalogofcalciumphosphate.
176.Neuroimagingfeaturesof neurocysticercosisareconsideredascriteriafordefinitivediagnosis- a)Major b)Minor c)Probable d)Absolute CorrectAnswer-AAnswer-A.MajorEvidenceoflesionshighlysuggestiveofneurocysticercosisonneuroimagingstudiesPositiveserumimmunoblotforthedetectionofanticysticercalantibodiesResolutionofintracranialcysticlesionsaftertherapywithalbendazoleorpraziquantelSpontaneousresolutionofsmallsingleenhancinglesions.
177.Laproscopicprocedurepatientdevelops shoulderpaindueto a)Subphrenicabscess b)Positionalpainduringsurgery c)Subdiaphragmaticmigrationofgas d)Injurytoliver CorrectAnswer-CAnswer-C.SubdiaphragmaticmigrationofgasOnetypeofpainthatisuniquetolaparoscopyisthepostlaparoscopyshoulderpainduetothephrenicnerveirritationtothediaphragmcausedbytheCO,gasthatremainsintheabdomenattheendoftheprocedure.Whenthepatientsitsup,thegasmovesupwardstothediaphragmandirritatesitleadingtoreferredpainC3-C%.
178.Apatientcomeswithacomplaintof shoulderpainafterlaparoscopicsurgery.Whatshouldbethenextstepinmanagement? a)Oralparacetamolfor2-3days b)USGofshoulderregion c)Diagnosticshoulderarthroscopy d)Intraarticularlignocaineinjection CorrectAnswer-AAnswer-A.Oralparacetamolfor2-3daysShouldertippainThepatientshouldbewarnedaboutthispreoperativelyandtoldthatthepainisreferredfromthediaphragmandnotduetoalocalproblemintheshoulders.Itcanbeatitsworst24hoursaftertheoperation.Itusuallysettleswithin2-3daysandisrelievedbysimpleanalgesics,suchasparacetamol.
179.Whichofthefollowingshouldbedone foranacuteonsetpainfulscrotalswellingin12yearsoldmale? a)Dopplerstethoscopeevaluation b)Administeranalgesics c)Advisebedrest d)Administerantibiotics CorrectAnswer-AAnswer-A.DopplerstethoscopeevaluationAcuteonsetpainfulscrotalswellingin12yearsoldpreadolescentmaleismostprobablyduetotorsionoftestis.Insuspectedcasesoftorsionoftestisortesticularappendageadopplerstethoscopeshouldbeusedtoevaluatebloodflowtothetesticles.
180.Whichofthefollowingelectrolyte abnormalitiescanbeseenafterbraininjury? a)Hyponatremia b)Hyperkalemia c)Hypomagnesemia d)Alltheabove CorrectAnswer-AAnswer-AElectrolyteabnormalitiesoccurin60%ofthepatientswithheadinjury.Patientswithbraininjuryareatahighriskforthedevelopmentofelectrolyteimbalanceincludinghyponatremia,hypocalemia,hypophosphatemiaaswellashypokalemiaand(toalesserdegree)Hypomagnesemia.Sotheappropriateanswerwillbehyponatremia.
181.Earliesthematologicalchangefollowing spleenectomyis a)Leukocytosisandthrombocytosis b)PresenceofHeinzbodies c)EvidenceofHowellJollybodies d)Poikilocytosis CorrectAnswer-AAnswer-A.LeukocytosisandthrombocytosisIntheimmediatepostsplenectomyperiod,leukocytosis(upto25,000/4)andthrombocytosis(upto1.106/4)develop,butwithin2-3weeks,bloodcellcountsandsurvivalofeachcelllineageareusuallynormal.
182.Percuteneouschemicallumbar sympathectomyispractisedusing- a)Phenol b)Ethanol c)Formalin d)Aceticacid CorrectAnswer-AAnswer-A.PhenolChemicalsympathectomyrequirestheinjectionofsmallquantitiesofdiluteaqueousphenolintothelumbarsympatheticchainunderradiographiccontrol.
183.Typicalofrectussheathhematomais- a)Severetenderness b)Bluishdiscoloration c)Firmpainfulmass d)Ecchymosis CorrectAnswer-CAnswer-C.FirmpainfulmassClinicalfeaturesofrectussheathhematomaA)SymptomsCommonhistoricalfeaturesofrectussheathhematoma(RSH)includeacuteabdominalpain,fever,nausea,andvomiting.B)Signsi)VitalsignsAlow-gradefeveriscommoninrectussheathhematoma.Thehematomacanbelargeenoughtocompromiseintravascularvolume,withresultantsignsofhypovolemicshockincludinghypotension,tachycardia,andtachypnea.ii)AbdominalexaminationTypically,theabdominalexaminationrevealsapalpable,painful,firm,nonpulsatileabdominalmasscorrespondingtotherectussheath.Themassmaybebilobarwithacentralgroove.Themassdoesnotmovewithrespiration.Becausethehematomaisdeeptothesubcutaneoustissueandrectusmuscles,themassisnotalwayspalpable,particularlyinobesepatients.
184.Burrholeisdonefor- a)ChronicSDH b)EDH c)SAH d)Contusion CorrectAnswer-AAnswer-A.ChronicSDHLiquefiedSDHsarecommonlytreatedwithdrainagethroughoneortwoburrholesplacedoverthethickestaspectsofthehematoma.Manysurgeonsplacefrontalandparietalburrholesthatlatercanbeincorporatedintoafrontotemporoparietalcraniotomy,ifneeded.
185.
Followingroadtrafficaccidentpatientsufferspolytraumaandisevaluatedintheemergencysectionofthehospital.Hispulserateis116,respiratoryrateis24,bloodpressureof122/78mmofHgandpatientismildlyanxious.Whatistheapproximatebloodlosspatienthasfollowingtrauma? a)<750ml b)750-1500ml c)1500-2000ml d)>2000ml CorrectAnswer-BAnswer-B.750-1500ml
186.Sebaceouscystoccursoccurinallthe followinglocationsinbodyexcept a)Palmsandsoles b)Axilla c)Back d)Pubicarea CorrectAnswer-AAnswer-A.PalmsandsolesThescalp,scrotum,shoulders,neck,andbackarethecommonsites,buttheycanoccurwherevertherearesebaceousglands.Therearenosebaceousglandsonthepalmsofthehandsandsolesofthefeet,thussebaceouscystsarenotfoundintheselocations.
187.Cortisollevelsremainelevatedforhow manyweek/sfollowinghemorrhage? a)1 b)2 c)3 d)4 CorrectAnswer-AAnswer-A.1Burnpatientshavedemonstratedelevatedcirculatingcortisollevelsforupto4weeks,whilesofttissueinjuryandhemorrhagemaysustainelevatedcortisollevelsforaslongas1week.
188.Achildswallowedawatchbattery containingalkalinecontent.Whatnext- a)ImmediateX-raymeasurements b)Removesurgicallyimmediately c)CTabdomen d)Laxatives CorrectAnswer-AAnswer-A.ImmediateX-raymeasurementsAlkalinedrycellbatteriescontain:SodiumhydroxidePotassiumhydroxide
189.Surgicaltreatmentofcongenital hydrocelerecommendedifitfailstoresolvebywhatage? a)1year b)2years c)3years d)4years CorrectAnswer-BAnswer-B.2yearsApatentprocessusvaginalisthatistoonarrowtopreventthedevelopmentofaninguinalherniamayneverthelessallowperitonealfluidtotrackdownaroundthetestistoformacongenitalhydrocele.Themajorityresolvespontaneouslyastheprocessuscontinuestoobliteratebutsurgicalligationisrecommendedinboysolderthan2yearsiftheydonotresolvespontaneously.
190.Whatpercentageofgallstonesareradio opaque? a)10-20 b)30-40 c)50-60 d)70-80 CorrectAnswer-AAnswer-A.10-20Cholelithiasisisacommoncauseofepigastricorrightupperquadrantpaininmiddleagedobesefemalepatients.Only15-20%ofthegallstonesareradio-opaque.Gallstonesarethemostcommonbiliarypathology.Itisestimatedthatgallstonesarepresentin10-15%oftheadultpopulationintheUSA.
191.Hemorrhagecommonlyseenwithtrivial traumainelderly- a)Subduralhemorrhage b)Extraduralhemorrhage c)Subarachnoidhemorrhage d)Intraparenchymalhemorrhage CorrectAnswer-AAnswer-A.SubduralhemorrhageSubduralhemorrhagefrequentlyoccursinolderadults,afterapparentlytrivialtraumaandisoftenrelatedtoafallinwhichthereisnodirecttraumatothehead.Recurrenttrivialtraumainelderlyisthemostcommoncauseofsubduralhemorrhagedeveloping.
192.Whichofthefollowingprovidesexcellent detailsaboutthechemodectomas? a)Xray b)CTangiography c)MRI d)PETSCAN CorrectAnswer-CAnswer-C.MRIMRIscanningprovidesexcellentdetailinmostcasesofchemodectomas.
193.Acrallentiginestypeofmalignant melanomaoccursin- a)Face b)Napeofneck c)Mucosa d)Sunexposedareas CorrectAnswer-CAnswer-C.MucosaAcrallentiginous-LeastcommonwithworstprognosisMCsite-sole,mucosa
194.Hoarsenessofvoiceinlungcarcinomais duetoinvasionofwhichstructure? a)Recurrentlaryngealnerve b)Internallaryngealnerve c)Glossopharyngealnerve d)Vagusnerve CorrectAnswer-AAnswer-A.RecurrentlaryngealnerveHoarseness-Recurrentlaryngealnerveinvasion
195.Percentageofrenalstonesthatare radio-opaque a)20 b)40 c)60 d)80 CorrectAnswer-DAnswer-D.80RoutineuseofnoncontastCTScanhascompletelyrevolutionizedtoimagingevaluationofrenalstonedisease,nearlycompletelyreplacingplainradigrpahsandXurographyfordiagnosisofacuteureteralobstructionbyrenalstones.Nephrolithiasisreferstothepresenceofcalculiintherenalcollectingsystem.Nearly10%ofthepopulationwillformarenalstoneintheirlifetime.Sufficientcalciumoxalateandphosphateispresentin80%oftherenalcalculiforthemtoberadio-opaqueontheplainradiographs.
196.Indicationsofthoracotomyinbluntchest traumaincludeallexcept a)Initialdrainageof>500mloffreshblood b)Ruptureofbronchous c)Continuedbleedingof>200ml/hrfor>/=3hrs d)Unsuccesfulattemptatdrainageofcardiactamponade CorrectAnswer-AAnswer-A.Initialdrainageof>500mloffreshbloodContinuedbleedingof>200ml/hrfor>3hrsRuptureofbronchus,aorta,esophagusordiaphragm.Cardiactamponade(ifneedleaspirationunsuccessful).
197.Visualexaminationisusedasscreening testfor a)Melanoma b)Breastcancer c)Thyroidcancer d)Testicularcancer CorrectAnswer-AAnswer-A.MelanomaVisualexaminationistheonlyscreeningtestformelanomaorskincancer.Forlesionswithsuspicionofheadandneckmelanoma,dermatologistsvisualexaminationscreeningis89-97%sensitivewith35-75%positivepredictivevalue.
198.Embolisationoftumorsisdoneusing a)Polyvinylalcohol b)Autologousbloodclots c)Absoluteethanol d)Alltheabove CorrectAnswer-DAnswer-D.AlltheaboveThefollowingaretheembolicmaterialscommonlyusedfortrans-arterialembolization:GelatinspongeparticlesMicrospheresAutologousbloodclotsPolyvinylalcoholn-butylcyanoacrylateglueAbsoluteethanol
199.
UptoLevelIIIlymphnodedissectionisdoneforwhichnodalstatusoforopharyngealcancer? a)N1 b)N2 c)N3 d)N4 CorrectAnswer-AAnswer-A.N1 ManagementofneckNOSelectiveneckdissectionOralCavity atleastlevelsl-lllOropharynx atleastlevelsll-lVN1-N2a-c Selectiveorcomprehensiveneck LevelVl Subglotticlaryngealcancers
200.Breastsurgeryisconsideredunderwhat categoryofcardiacrisk? a)>10% b)5-10% c)1-5% d)<1% CorrectAnswer-DAnswer-D.<1%Low(reportedcardiarisk<1%)-EndoscopicproceduresSuperficialproccdureCataractsurgeryBreastsurgeryAmbulatorysurgery
201.Whatistheorderofdisorientation,which apersongoesthroughafteraneventoftraumatohead? a)Firsttime,thenplacefollowedbyperson b)Firstplace,thentimefollowedbyperson c)Firstperson,thentimefollowedbyplace d)Firsttime,thenpersonfollowedbyplace CorrectAnswer-AAnswer-A.Firsttime,thenplacefollowedbypersonOrientationreferstothecleintsrecognitionofperson,placeandtime-thatisknowingwhoandwhereeorsheisandthecorrectday,dateandyear.Thisiscommonlydocumentedas"orientedX3".Absenceofcorrectinformationaboutperson,placeandtimeisreferredtoasdisortientation.
202.Whichofthefollowingisthepreferred routeofaccessfortotalparenteralnutritioninapatientwhorequiresthesamefor<14daysandthereisotherwisenoindicationforuseofcentralcatheter? a)Internaljugularvein b)Externaljugularvein c)Periphaeralvein d)PICCline CorrectAnswer-CAnswer-C.PeriphaeralveinAdministrationofparenteralnutritionviaaperipheralvenouscathetershouldbeconsideredforpatientswhoarelikelytoneedshort-termparenteralnutrition(lessthan14days)whohavenoneedforcentralaccessforotherreasons.Careshouldbetakenincatheterchoice,andinattentiontopH,tonicityandlong-termcompatibilityoftheparenteralnutritionformulationsinordertoavoidadministrationorstabilityproblems.
203.Whichofthefollowingisnottrueabout useofgraftinvascularsurgery a)Autologussaphenousveinachievessuperiorpatencyratesto prostheticmaterialsespeciallyinfemorodistalbypass b)ThepatencyofthePTFEgraftsmaybeimprovedby interpositionofaveincuffatproximalanastomosis c)Dopplerultrasoundassessmentisthemethodofchoicefor qualityassuaranceoncompletionofoperativeprocedure d)Intheabsenceofspecificcontraindicationsaspirinshouldbe prescribedforallthepatientsofperipheralvasculardisease CorrectAnswer-BAnswer-B.ThepatencyofthePTFEgraftsmaybeimprovedbyinterpositionofaveincuffatproximalanastomosisAutologoussaphenousveinachievessuperiorpatencyratestoprostheticmaterials,especiallyinfemorodistalbypass.Insituandreversedveingraftbypassesperformequallywellandthechoiceoftechniqueshouldbebaseduponanatomicalconsiderationsthatarespecifictoindividualpatients.Intheabsenceofsaphenousvein,noconvencingstudiesexisttosuggestthepreferredprostheticalternative.ThepatencyofPTFEgraftsmaybeimprovedbyinterpositionofaveincuffatthedistalanastomosis.Patencyratesassociatedwithpre-cuffedPTFE,graftsarecomparabletothatobtainedfromstandardPTFEgraftswithaninterpositionveincuff.Thereisnojustificationfortheroutineuseofadjuvantdistalarterivenousfistulae.
204.Forlowerlipcarcinomaof<1cminsize. Thetreatmentofchoicewillbe- a)Radiation b)Chemotherapy c)Excision d)Radiationandchemotherapy CorrectAnswer-CAnswer-C.ExcisionSmalltumors(T1&T2)(<2cm)-If1/3rdorlessthan1/3rdlipinvolved-VorW-shapedexcision+primaryclosure.Excisionoflowerlipupto1/3rdcanbesuturedprimarilyin3layers?mucosa,muscle,andskinkeepingvermillionborderinproperapposition.
205.Preferredtreatmentfororaltongue carcinomawhichinfiltratesthelocalcorticalboneis- a)Subtotalglossectomy b)Subtotalglossectomy+selectiveneckdissection c)Subtotalglossectomy+selectiveneckdissection+ mandibulectomy d)Totalglossectomy+selectiveneckdissection+ mandibulectomy CorrectAnswer-CAnswer-C.Subtotalglossectomy+selectiveneckdissection+mandibulectomyThemanagementplanforlocallyadvancedtonguecarcinomasincludessubtotalglossectomy+selectiveneckdissection+mandibulectomy.Advancedtumors(T3andT4)oftenencroachuponthefloorofthemouthand,occasionally,themandible.Inthesecircumstances,aresectionofthetongueandfloorofthemouthandmandibleisrequired.
206.Intracranialpressureisnotraised during a)Hyperventilation b)Statusepilepticus c)Headinjury d)Subduralhematoma CorrectAnswer-AAnswer-A.HyperventilationHyperventilationisusedasatreatmentwithraisedintracranialpressure.HyperventilationcausesdecreasedPaCO2whichsubsequentlyleadstoarterialvasoconstrictionthusloweringcerebralbloodflow(CBF),cerebralbloodvolume,andintracranialpressure
207.Outcomeofburnsdependson- a)Extentofburns b)Typeofresuscitationfluid c)Maintenanceofairway d)Skingrafting CorrectAnswer-AAnswer-A.ExtentofburnsThevariousfactorsinfluencingtheoutcomeofburnsare:ExtentofburnsDepthofburnsTimingoffirstEscharectomy(removesdevitalizedtissueandhencesourceofinfection)Ageandmedicalcomorbidities
208.Immediatephysiologicalresponseto suddendecreaseinbloodvolumeis a)Releaseofepinephrine b)Shiftoffluidfromintracellulartointerstitialcompartment c)Releaseofangiotensin d)Releaseofthyroxine CorrectAnswer-AAnswer-A.ReleaseofepinephrineThemajorhemodynamicabnormalityinhypovolemicshockisdecreaseinpreload.Theimmediatephysiologicalresponseofthebodytothesuddendecreaseinvolume(preload),isareleaseofcatecholamines(epinephrine,norepinephrine).Thesubsequentincreaseinheartrateandcontractilityhelpmaintaincardiacoutput.
209.Cardinalrulefordressingofpressure ulceristo- a)Keepulcertissuedryandsurroundingintacttissuedry b)Keepulcertissuedryandsurroundingintacttissuemoist c)Keepulcertissuemoistandsurroundingintacttissuedry d)Keeptheulcertissuemoistandsurroundingintacttissuemoist CorrectAnswer-CAnswer-C.KeepulcertissuemoistandsurroundingintacttissuedryAnidealdressingshouldprotectthewound,bebiocompatibleandprovideidealhydration.Thetypeoftheulcerbedandthedesireddressingfunctiondeterminethetypeofdressingneeded.Thecardinalruleistokeeptheulcertissuemoistandthesurroundingintacttissuedry.
210.WhatisthemeaningoftheFrenchword "Debridement" a)UnleashorCutOpen b)Debulk c)Sanitize d)Rehydration CorrectAnswer-AAnswer-A.UnleashorCutOpenTakenfromtheFrenchmeaningto'unleashorcutopen',debridementhascometomeanmorethansimplythelayingopenoftissues.Itplaysacrucialpartinthemanagementoftrauma.
211.Slipsignisseenin a)Lipoma b)Desmoidtumor c)Sebaceouscyst d)Hernia CorrectAnswer-AAnswer-A.LipomaAcharacteristic"slippagesign"maybeelicitedbygentlyslidingthefingersofftheedgeofthetumor.Thetumorwillbefelttoslipoutfromunder,asopposedtoasebaceouscystoranabscessthatistetheredbysurroundinginduration.
212.Forreimplantationdigitsarestoredin a)Icepacks b)Deepfreeze c)Coldsaline d)Plasticbagswithice CorrectAnswer-DAnswer-D.PlasticbagswithiceTheamputatedpartshouldbewrappedinmoistenedgauzeandplacedinasealedplasticbag.Thisbagshouldthenbeplacedinanicewaterbath.Donotusedryiceanddonotallowtheparttocontacticedirectly;frostbitecanoccurintheamputatedpart,whichwilldecreaseitschanceofsurvivalafterreplantation.Bleedingshouldbecontrolledintheproximalstumpbyasminimalameansasnecessary,andthestumpdressedwithanonadherentgauzeandbulkydressing."
213.Characteristicsitefordevelopmentof venouslegulcersis a)Skinofgaiterregion b)Behindlateralmalleolus c)Shinoftibia d)Medialaspectofknee CorrectAnswer-AAnswer-A.SkinofgaiterregionThevenousulcerofthelegcharacteristicallydevelopsintheskinofthegaiterregion,theareabetweenthemusclesofthecalfandtheankle.ThisistheregionwheremanyoftheCockettperforatorsjointheposteriortibialveintothesurfacevein,knownastheposteriorarchveinThemajorityofulcersdeveloponthemedialsideofthecalfbutulcersassociatedwithlessersaphenousincompetenceoftendeveloponthelateralsideoftheleg.Ulcerscandeveloponanypartofthecalfskininpatientswithpost-thromboticlegs;however,venousulcersrarelyextendontothefootorintotheuppercalfand,ifthereisulcerationatthesesites,otherdiagnosesshouldbeseriouslyconsidered.
214.Whichofthefollowingistrueaboutthe managementofvenousulceroflowerlimb? a)Initialtreatmentisdebridementandsurgery b)Compressiondressingsshouldideallybeappliedontwice weeklybasis c)Antibioticsdonotspeeduptheulcerhealing d)Biologicaldressingsdonothavepotentialtoimprovehealing CorrectAnswer-CAnswer-C.AntibioticsdonotspeeduptheulcerhealingPatientsareinitiallytreatedbyacompressionbandagingregimen.Alternativetothesebandagingregimensistoapplyablandabsorbentleak-proofdressingbeneathagraduatedelasticcompressionstocking(classII).BiopsiesareindicatedifmalignancyissuspectedanditisimportanttorememberthataMarjolin'stypeofulcer(asquamouscellorbasalcellcarcinoma)candevelopinachroniclongstandingvenousulcer.Considerationmustbegiventohealingtheulcerbyexcisionandgrafting.Biologicaldressingshavebeendeveloped,includingfetalkeratinocytesandcollagenmeshesPinchgraftsandulcerexcisionwithmeshgraftinghavebeenshowntoprovidegoodearlyhealingwithmoderatelong-termresults
215.Whichofthefollowingsuturehasmax tensilestrengthandminimumtissuereaction a)Poliglecaprone b)Polypropylene c)Polygalctine d)Polydioxanone CorrectAnswer-BAnswer-B.PolypropylenePolypropylenesuturesarenon-absorbableandprovidepermanentwoundsupport.Polypropylenesuturesarebluecoloredforeasyidentificationduringsurgery.Polypropylenesutureshaveexcellenttensilestrengthandareusedfororthopaedic,plasticandmicrosurgeries,generalclosureandcardiovascularsurgeries
216.Blunttraumaexploratorylaparotomy donenonexpansileswellingfoundonmesentericborderofintestine,management a)Resectionandanastomosis b)Ligation c)Excisionofswelling d)None CorrectAnswer-DAnswer-D.NoneNonExpansileswellingonmesentericborderofintestineisalmostalwaysduetomesentericadenitisanditisanincidentalfinding,hencewarrantingnosurgicalintervention.
217.InSVCsyndromefornonsmallcell carcinomaoflungmanagementdone a)Radiotherapy b)Immunotherapy c)Chemotherapy d)Surgery CorrectAnswer-AAnswer-A.RadiotherapyInpatientswithSVCSsecondarytonon?small-cellcarcinomaofthelung,radiotherapyistheprimarytreatment.Thelikelihoodofpatientsbenefitingfromsuchtherapyishigh,buttheoverallprognosisofthesepatientsispoorSmallcellcarcinomaoflungpresentingasSVCsyndrome-Combinationofchemotherapyandradiotherapy.NonSmallcellcarcinomaoflungpresentingasSVCsyndrome-Radiotherpyalone.
218.Hadfieldsoperationisperformedfor whichofthefollowingpathology? a)Ductectasia b)Fibroadenoma c)Mondorsdisease d)Inflammatorybreastcarcinoma CorrectAnswer-AAnswer-A.DuctectasiaTREATMENT-StopsmokingHadfield'soperation-excisionofallmajorductsAntibiotics-amoxiclavandmetronidazole
219.Microdochotomyistreatmentfor a)Ductectasia b)Breastabscess c)Ductpapilloma d)DCIS CorrectAnswer-CAnswer-C.DuctpapillomaThefinaldiagnosisismadebyexcisingtheinvolvedduct(Microdochotomy)andanyunderlyingmassifpresentandsubjectingthemforahistopathologicaldiagnosis
220.Previousraditiontherapyforwhich diseaseparticularyincreasestheriskofbreastcarcinomadevelopment? a)Hodgkinslymphoma b)Mantlecelllymphoma c)Nasopharyngealcarcinoma d)Lungcarcinoma CorrectAnswer-AAnswer-A.HodgkinslymphomaArealprobleminwomenwhohavebeentreatedwithmantleradiotherapyaspartofthemanagementofHodgkin'sdisease,inwhichsignificantdosesofradiationtothebreastarereceived.
221.LAHSALcodeisusedtorepresent congenitalmalformationof- a)Lip b)Alveolus c)Hardandsoftpalate d)Alltheabove CorrectAnswer-DAnswer-D.Alltheabove
222.Whichofthefollowinghistologicaltype ofbreastcarcinomaworstprognosis? a)Tubular b)Colloid c)Papillary d)Scirrhous CorrectAnswer-DAnswer-D.ScirrhousMCtypeofbreastcarcinomaisinvasiveductalcarcinoma(schirrhous).MCformseen(60-75%)Hardlump,whitishyellow,noncapsulated,irregularwithcartilaginousconsistencyRetractionofnipple
223.Kernahen'sstriped'Y'classification, Mainreferencepointis- a)Incisiveforamen b)Softpalate c)Hardpalate d)Thirdmolar CorrectAnswer-AAnswer-A.IncisiveforamenKernahen'sstriped'Y'classificationItisusedtoclassifycleftlipandcleftpalateTheincisiveforamenistakenasreferencepoint
224.Themostcommonlyusedmyocutaneous pediclegraftforpelvissurgeriescontainsmusclesegmentsfrom- a)Rectusabdominismuscle b)Externalobliquemuscle c)Internalobliquemuscle d)Transversusabdominismuscle CorrectAnswer-AAnswer-A.RectusabdominismuscleThemostfrequentlyusedmyocutneouspediclegraftscontain,musclesegmentsfromtherectusabdominismuscleoftheanteriorabdominalwall,gracialismuscleoftheinnerthigh,bulbocavernosusmuscleofthevulva,thetensorfascialatamuscleofthelateralthigh,andgluteusmaximusmuscle.
225.52yearsfemalepatientpresentswith4 cmdiameterdiagnosedbreastcancerlesionwithipsilateralaxillaryandcontralateralsupraclavicularlymphadenopathy.AsperAJCCsystem,patientbelongstowhichstageofbreastcancer? a)Ilia b)None c)IIIc d)IV CorrectAnswer-DAnswer-D.IV

226.Revascularizationandangiogenesis processafterskingraftingisseenafterhowmanydaysaftertheprocedure? a)4 b)5 c)6 d)7 CorrectAnswer-BAnswer-B.5RevascularizationorangiogenesisAfterapprox5days,revascularizationoccursandthegraftdemonstratesbotharterialinflowandvenousoutflow.
227.Bestprognosisforcarcinomabreastis seenwithwhichofthefollowing? a)<1cmsize,nodes-ye,ER/PR+ve,her2/neu-ye b)<1cmsize,nodes-ye,ER/PR-ye,her2/neu+ve c)<2cmsize,nodes-ye,ER/PR+ve,her2/neu-ye d)<2cmsize,nodes-ye,ER/PR-ye,her2/neu+ve CorrectAnswer-AAnswer-A.<1cmsize,nodes-ye,ER/PR+ve,her2/neu-yeMolecularchangesinthetumorarealsouseful.TumorsthatoverexpresserbB2(HER2/neu)orhaveamutatedp53genehaveaworseprognosis.TumorsthatoverexpresserbB2aremorelikelytorespondtohigherdosesofdoxorubicin-containingregimensandpredictthosetumorsthatwillrespondtoHER2/neuantibodies(trastuzumab)(herceptin)andHER2/neukinaseinhibitors.
228.Favorableprognosiswith>90%5year survivalrateforcarcinomabreastisseeninwhichofthefollowing? a)Screendetectedductalcarcinomainsitu b)Screendetectedlobularcarcinomainsitu c)Nodenegativetumorwithfavourablehistology d)Noneoftheabove CorrectAnswer-AAnswer-A.ScreendetectedductalcarcinomainsituIntraductalcarcinoma(malignantmammaryductalepithelialcells)withoutanyinvasionintobasementmembrane.
229.Whatpercentageofreductionindying frombreastcancercanbeachievedwithannualscreeningafterageof50yrswithmammography? a)15-20% b)20-25% c)25-30% d)30-35% CorrectAnswer-CAnswer-C.25-30%Meta-analysisexaminingoutcomesfromeveryrandomizedtrialofmammographyconclusivelyshowsa25-30%reductioninthechanceofdyingfrombreastcancerwithannualscreeningafterage50years.
230.Whichofthefollowinghormonallevels influencethedevelopmentofbenignbreastdisease? a)Estrogen b)Progesterone c)Lutenizinghormone d)Testosterone CorrectAnswer-A:BAnswer-A.Estrogen&B.ProgesteroneEtiology/PathogenesisHormonal:Responsivenessofbreasttissuetomonthlychangesofestrogenandprogesteroneplayanimportantroleinpathogenesisofbenignbreastdisease.Theymayberelatedtoexcesshormonalstimulationorhypersensitivityofbreasttissue.
231.Whichofthefollowingistrueabout breastreconstructionsurgery a)Easiestreconstructionisdoneusingsilicongelimplant b)TRAMflapgivesbettercosmeticresultsthanLDflap c)Radiotherapyinpostopperioddoesnotinfluencetheoutcome afterbreastreconstruction d)Nipplereconstructioncannotbeperformedunderlocal anesthesia CorrectAnswer-AAnswer-A.EasiestreconstructionisdoneusingsilicongelimplantTheeasiesttypeofreconstructionisusingasiliconegelimplantunderthepectoralismajormuscle.Thismaybecombinedwithpriortissueexpansionusinganexpandablesalineprosthesisfirst(oracombineddevice),whichcreatessomeptosisofthenewbreast.Iftheskinatthemastectomysiteispoor(e.g.followingradiotherapy)orifalargervolumeoftissueisrequired,amusculocutaneousflapcanbeconstructedeitherfromthelatissimusdorsimuscle(anLDflap)orusingthetransversusabdominismuscle(aTRAMflapas).Thelattergivesanexcellentcosmeticresultinexperiencedhandsbutisalengthyprocedureandrequirescarefulpatientselection.Itisnowusuallyperformedasafreetransferusingmicrovascularanastomosis,althoughthepedicledTRAMfromthecontralateralsideisstillused.VariationsontheTRAMflaprequiringlessmuscleharvesting,suchastheDIEPflap(basedondeepinferiorepigastricvessels),areincreasinglybeingused.
increasinglybeingused.Impedimentstoimmediatereconstructionincludeinsufficienttheatretimeandalackofexperiencedreconstructivesurgeons.Inaddition,ifapatientislikelytoneedpostoperativeradiotherapythenadelayedreconstructionusingaflapoftengivesabetterresult.Radiotherapyontoaprosthesisoftenleadstoahighincidenceofcapsularcontractureandunacceptableresults.Nipplereconstructionisarelativelysimpleprocedurethatcanbeperformedunderalocalanaesthetic.
232.Treatmentoflargeomphaloceledefect canbedoneby a)Primaryclosure b)StagedclosureusingPTFEmesh c)Paintingintactsacdailywithantisepticsolution d)Alltheabove CorrectAnswer-DAnswer-D.AlltheaboveLargedefectspresentamoresubstantialproblemandfourtechniqueshavebeendescribed:non-operativetherapy,skinflapclosure,stagedclosureandprimaryclosure
233.Whichofthefollowingisnottrueabout Pateysmastectomy a)Itisalsocalledmodifiedradicalmastectomy b)Intercostobrachialnervesareusuallypreserved c)Alllymphnodesofaxillaareremoved d)Pectoralismuscleiseitherdividedorretracted CorrectAnswer-BAnswer-B.IntercostobrachialnervesareusuallypreservedItisalsocalledmodifiedradicalmastectomyandisacommonlyperformedprocedure.Theintercostalbrachialnervesareusuallydividedinthisoperationandthepatientshouldbewarnedaboutsensationchangespostoperatively.Thewoundisdrainedusingawide-boresuctiontube.Earlymobilisationofthearmisencouragedandphysiotherapyhelpsnormalfunctiontoreturnveryquickly
234.Dermoepidermalburniswhatdegreeof burn- a)I b)II c)III d)IV CorrectAnswer-BAnswer-B.IIDermoepidermalburnsareSuperficialIIdegreeburns.
235.Macroprolactinomaidealtreatmentis a)Excision b)Bromocriptine c)Stereotacticradiosurgery d)Observation CorrectAnswer-BAnswer-BBromocriptine(BEC)isgenerallyconsideredtobetheagentofchoiceinthetreatmentofprolactinomabecauseofitslongtrackrecordandsafety.
236.Initialtreatmentformostpatientof growthhormonesecretingpituitaryadenomais a)Transphenoidalsurgicalresection b)Somatostatinanalogs c)GHrectorantagonists d)Dopamineagonists CorrectAnswer-AAnswer-A.TransphenoidalsurgicalresectionTranssphenoidalsurgicalresectionbyanexperiencedsurgeonisthepreferredprimarytreatmentforbothmicroadenomas(curerate-70%)andmacroadenomas(<50%cured).
237.Radioiodinepreferredintreatmentin a)Youngpatients b)Pregnancy c)Recentonsetoftoxicgoiter d)Postsurgeryforpapillarythyroidcancer CorrectAnswer-CAnswer-C.RecentonsetoftoxicgoiterThemainindicationsforRAItherapyincludethefollowingconditions 1. Hyperthyroidismdueto: Grave'sdiseaseToxicmultinodulargiotreorHyperfunctioningthyroidnodules 2. Non-toxicmultinodulargoitre3. Thyroidcancer.
238.Parathyroidadenomasaccountforhow muchpercentageofpatientswithprimaryhyperpara-thyroidism? a)50% b)60% c)70% d)80% CorrectAnswer-DAnswer-D.80%Solitaryadenomas,Asingleabnormalgland,isthecausein-80%ofpatients;theabnormalityintheglandisusuallyabenignneoplasmoradenomaandrarelyaparathyroidcarcinoma.
239.Investigationofchoiceforhepatic metastasisformstomachcanceris a)MRI b)CECT c)USG d)HIDA CorrectAnswer-BAnswer-B.CECTCTistheimagingmodalityofchoiceforevaluatinglivermetastases.Thispreferenceislargelyattributabletotheeffectsofthedualbloodsupplyontheenhancementcharacteristicsofmetastases,ascomparedwithnormalliverparenchyma.
240.Normalurineflowrateinhealthyadults inurodynamicstudyis- a)10ml/sec b)20ml/sec c)25ml/sec d)5ml/sec CorrectAnswer-BAnswer-B.20ml/secAges14to45--Theaverageflowrateformalesis21mL/sec.Theaverageflowrateforfemalesis18mL/sec.Ages46to65--Theaverageflowrateformalesis12mL/sec.Theaverageflowrateforfemalesis18mL/sec.Ages66to80--Theaverageflowrateformalesis9mL/sec.Theaverageflowrateforfemalesis18mL/sec.
241.
Radicalnephrectomyinvolvesresectionofthefollowingexcept- a)Gerotasfascia b)Ipsilateraladrenalgland c)Surroundinghilarlymphnodes d)Proximalpara-aorticlymphnodes CorrectAnswer-DAnswer-D.Proximalpara-aorticlymphnodesThestandardmanagementforstageIorIItumorsandselectedcasesofstageIIIdiseaseisradicalnephrectomy.ThisprocedureinvolvesenblocremovalofGerota'sfasciaanditscontents,includingthekidney,theipsilateraladrenalgland,andadjacenthilarlymphnodes.
242.Prognosisofsurgeryforliver secondariesisbestforwhichcancer? a)Colorectal b)Neuroendocrine c)Genitourinary d)Esophageal CorrectAnswer-CAnswer-C.GenitourinaryFornoncolorectal,nonneuroendocrinetumors,metastasesfromgenitourinaryprimarieshavethebestprognosisfollowinghepaticmetastatectomy.
243.IndicatonsoflivertransplantinPCM poisoningareallexcept a)SGPTincrease b)PT/INR c)Highcreatinine d)Encephalopathy CorrectAnswer-AAnswer-A.SGPTincreaseIndicationpoorprognosisandhencenecessitateatransplantinsuchpatients- 1. ArterialpH<7.3(takenbysamplingofbloodfromanartery)2. Allthreeofaninternationalnormalizedratio(INR)ofgreaterthan 6.5,serumcreatinineofgreaterthan300micromolesperlitreandthepresenceofencephalopathy(ofgradeIIIorIV).Thesethreearemarkersofcoagulopathy,kidneyfunctionandmentalstatus.
244.Harderrenalstonehavingless satisfactoryresultswithECWLis- a)Oxalatestone b)Phosphatestone c)Uratestone d)Cystinestone CorrectAnswer-DAnswer-D.CystinestoneExtracorporealShockwaveLithotripsy(ECWL):Aurinarycalculushasacrystallinestructure.Bombardedwithshockwavesofsufficientenergyitdisintegratesintofragments.TheclearanceofstonefromthekidneyusingESWLwilldependupontheconsistencyofthestoneanditssite.Mostoxalateandphosphatestonesfragmentwell.
245.Milancriteriaisfor a)SelectingpatientsforLivertransplantation b)SelectingpatientsforLungtransplantation c)SelectingpatientsforKidneytransplantation d)Selectingpatientsforhearttrnasplantation CorrectAnswer-AAnswer-A.SelectingpatientsforLivertransplantationTheMilancriteriastatethatapatientisselectedfortransplantationwhenheorshehas: 1. Onelesionsmallerthan5cm2. Upto3lesionssmallerthan3cm3. Noextrahepaticmanifestations4. Novascularinvasion
246.Whatshouldbetheplanofmanagement forapatientforwhomwhileundergoingsimplecholecystectomyT2gallbladdercarcinomaisdiscovered? a)Resectionof4b-5segement,dissectionoftheN1-2nodesand excisionofportsites b)Postoperativeadjuvantchemotherapy c)Radicalcholecystectomy d)Whipplesprocedure CorrectAnswer-AAnswer-A.Resectionof4b-5segement,dissectionoftheN1-2nodesandexcisionofportsitesStageIandII-simplecholecystectomyStageIII?cholecystectomy+adjacenthepaticresection(atleast2cmdepth)+regionallymphadenectomyPoorprognosis
247.Followingarethesign/ssuggestiveof obstructionofurinarytractonCTScan a)Hyroureter b)Perinephricstranding c)Thickeningofthelateroconalfascia d)Alltheabove CorrectAnswer-DAnswer-D.AlltheaboveNonenhancedCThasbeenacceptedastheimagingmodalityofchoiceintheradiologicevaluationofpatientssuspectedofhavingurolithiasis.CTissuperiortootherimagingmodalitiesinthediagnosticaccuracyandassessmentoftheircharacteristics.Themostdirectsignofureterolithiasisisthestonewithintheureterallumen,withproximalureteraldilatationandnormaldistalcaliber.OtherfavorablesecondarysignsatCTare:Hydroureter,hydronephrosis,perinephricstranding,periureteraledemaandunilateralrenalenlargement.
248.Treatmentofchoiceforbladderstone a)Transurethrallitholapaxy b)Percutaneoussuprapubiclitholapaxy c)Vesicotomyandstoneretrieval d)Intravenousantibiotics CorrectAnswer-AAnswer-A.TransurethrallitholapaxyTREATMENT-Atransurethralcystolitholapaxyisthemostcommonprocedureusedtotreatadultswithbladderstones.Smallstone-UltrasoundlithotripsyLargestone-laserlithotripsyLitholapaxy-cystoscopiclithotriteSuprapubiccystolithotomy
249.3cmstoneincysticductnearthe ampullaofvater,TheMethodofremovalisa a)Transduodenalapproach b)Supraduodenalapproach c)Lithotripsy d)Chemicaldissolution CorrectAnswer-AAnswer-A.TransduodenalapproachAccordingtoMaingot's"Themethodsofsurgicaldrainageincludetransduodenalsphincterotomy,choledochoduodenotomy,andcholedochojejunostomy."
250.40yearsoldmalecomplainsofloinpain since1month.Patient'scomplaintofpainhasseverelyincreasedoverlast2hoursandpainnowradiatesfromloinandtogroinandanteriorthighandpatientiswrithinginbedforcomfort.Whatisthemostprobableetiology? a)Bladdercalculus b)Uretericcalculus c)Vesicouretericreflux d)Hydronephrosis CorrectAnswer-BAnswer-B.UretericcalculusThereisapatternofsevereexacerbationonabackgroundofcontinuingpainRadiatestothegroin,penis,scrotumorlabiumasthestoneprogressesdowntheureterTheseverityofpainisnotrelatedtothesizeofthestoneThepainisalmostinvariablyassociatedwithhaematuriaTheremaybefewphysicalsigns
251.Whatisthemainhazardofectopictestis - a)Impotence b)Carcinomadevelopment c)Liabletoinjury d)Herniadevelopment CorrectAnswer-CAnswer-C.LiabletoinjuryThemainhazardisliabilitytoinjury.
252.Traditionallyscrotalcarcinomais associatedwithwhichofthefollowingoccupations? a)Chimneysweeps b)Mulespinners c)Coalworker d)Woodworkers CorrectAnswer-AAnswer-(A)ChimneysweepsCarcinomaofthescrotum.Itistraditionallyrecognizedasanoccupationalhazardforchimneysweepsandmulespinners.ItwasdescribedbyPotts.Itwasthefirstcancerlinkedtooccupationalexposurewhen,in1775,PerivallPottdescribeditinchimneysweepsinEngland.Otheroccupationsthathadapreponderanceofthediseaseincludedpeoplewhoworkedwiththedistillatesofcoalandmenexposedtomineraloil.
253.Whichofthefollowingarethetreatment optionsforcystocele? a)Anteriorcolporrhaphy b)Transvaginaltape c)Transobturatortape d)Alltheabove CorrectAnswer-DAnswer-D.AlltheaboveTraditionally;ananteriorvaginalwallrepair(anteriorcolporrhaphy)wasperformedvaginally;nowreplacedbyvaginallyinsertedtape[transvaginaltape(TVT)ortransobturatortape(TOT)]ormeshslings.
254.Mostcommonsiteofpenilecarcinomais - a)Glans b)Prepuce c)Shaft d)Coronalsulcus CorrectAnswer-AAnswer-A.GlansWhenitoccursontheglanspenis,itisknownaserythroplasiaofQueyratandwhenitoccursontheshaftofthepenisitiscalledBowen'sdisease.Carcinomaofthepenisismosttypicallyasquamouscellcarcinomaarisingintheskinoftheglanspenisortheprepuce.MCorginatesfromglans>sulcus>prepuce>shaft
255.Besttimeforsurgeryofhypospadiasis atwhatage- a)0-4months b)4-6months c)6-10months d)>2years CorrectAnswer-CAnswer-C.6-10monthsOperationsforhypospadiasisareroutinelyperformedwhenthepatientisbetween6and18monthsofage.Theinfanthasgoodtolerancetosurgeryandanesthesiabytheageof6months.Thechildiswellawareofhisgenitaliaandtoilettrainingbytheageof18months.So,themostsuitableagefortheoperationofhypospadiasisbetween6and18months.Thedegreeofhypospadiasdictatestheneedforrepair.Iftheopeningisglanularorcoronal(85%ofpatients),thepenisisusuallyfunctionalbothformicturitionandprocreationandrepairisdoneprimarilyforcosmeticreasons.Openingsthataremoreproximalontheshaftrequirecorrectiontoallowvoidingwhilestanding,normalerection,andproperspermdepositionduringintercourse.Thecommonestprocedurefordistalhypospadiasisthe`tubularizedincisedplate'urethroplasty,whiletechniquesthatutilizetheforeskinarecommonlyusedformoreproximalhypospadias.Complicationsofhypospadiassurgery:urethrocutaneousfistula(mostcommon)RecurrenceofchordeeUrethralstricture
MeatalstenosisInfection
256.Commonestsiteofhypospadiasis- a)Justproximaltoglans b)Inthemeidofpenis c)Scrotum d)Perineum CorrectAnswer-AAnswer-A.JustproximaltoglansHypospadiasisaconditioninwhichtheurethralmeatusopensontheundersideofpenisortheperineum(i.e.ventralsurfaceofpenis)proximaltothetipoftheglanspenis.
257.Whichenzymeisofdiagnostic importanceinchronicpancreatitis a)Amylase b)Pancreaticpolypeptide c)Lipase d)SerumInterleukin6levels CorrectAnswer-CAnswer-C.LipaseElevatedlipaselevelsaremorespecifictothepancreasthanelevatedamylaselevels.Lipaselevelsremainhighfor12days.Inpatientswithchronicpancreatitis(usuallycausedbyalcoholabuse),lipaselevelsmaybeelevatedinthepresenceofanormalserumamylaselevel.
258.Mostcommonscreeningtestforacute pancreatitis- a)Serumamylase b)Serumlipase c)Urinetrypsinogen d)Insulin CorrectAnswer-CAnswer-C.UrinetrypsinogenRapidmeasurementofurinarytrypsinogen-2levelisusefulintheemergencydepartmentasascreeningtestforacutepancreatitis.
259.Physiologicaladhesionsbetween foreskinandglanspenispersistuntilyearsofage- a)4 b)5 c)6 d)7 CorrectAnswer-CAnswer-C.6Phimosisisaconditioninwhichtheforeskinofthepeniscannotbepulledbackpasttheglans.Thephysiologicaladhesionsbetweentheforeskinandtheglanspenismaypersistuntil6yearsofageormore,givingthefalseimpressionthattheprepucewillnotretract.
260.Whichofthefollowingis/arethe imagingcriteriaforunresectablecarcinomaofpancreas? a)Metastaticspreadtovertebrae b)Invasioninduodenalwall c)Irregularincreaseindensityofomentalfat d)Alltheabove CorrectAnswer-DAnswer-D.AlltheaboveMetastaticspredoutsidethepancreasmakestumorirresectable.Tumorinvasionintoadjacentorganssignificsunresectability.
261.A10yearsoldmalepresentsasmooth swellingnearsuperficialinguinalring,whichmovesdownwardswhenthetesticleispulleddownwards.Diagnosis- a)Inguinalhernia b)Congenitalhydrocele c)Encystedhydroceleofthecord d)Varicocele CorrectAnswer-CAnswer-C.EncystedhydroceleofthecordAhydrocoeledevelopsinaremnantoftheprocessusvaginalissomewherealongthecourseofthespermaticcord.Thishydrocoelealsotransilluminates,andisknownasanencystedhydrocoeleofthecord.Theswellingmovesdownwardsandbecomeslessmobileifthetestisispulledgentlydownwards.Infemales,amulticystichydrocoeleofthecanalofNucksometimespresentsasaswellinginthegroin.
262.Drugofchoiceforpalliativetreatmentof pancreaticcarcinoma a)Erlotinib b)Gemcitabine c)Paclitaxel d)Cyclophosphomide CorrectAnswer-BAnswer-B.GemcitabineA)Inoperablelocallyadvanceddisease-Gemcitabineisusedathetreatmentofchoiceforthesepatients.B)MetastaticDiseaseGemcitabineisthestandardtreatmentwithamediansurvivalof6monthsanda1-yearsurvivalrateofonly20%.Capecitabine,anoralfluoropyrimidine,hasbeencombinedwithgemcitabine(GEM-CAP)inaphaseIIItrialthatshowedanimprovementinresponserateandprogression-freesurvivaloversingle-agentgemcitabine,butnosurvivalbenefit.
263.
Modified-KaushWhippleoperationisdevisedtopreservewhichpartofstomach? a)Fundus b)Body c)Lessercurvature d)Pyloricantrum CorrectAnswer-DAnswer-D.PyloricantrumThisprocedureisalsocalledpyloruspreservingpancreaticoduodenectomy(PPPDorpp-KaushWhippleprocedure).Itistheprocedureofchoiceformostadenocarcinomasoftheheadofpancreas.Theoriginalpancreato-duodenectomyasproposedbyWhippleincludedresectionofthegastricantrum.TheWhippleprocedureisnowreservedforsituationsinwhichtheentireduodenumhastoberemoved(e.g.inFAP)orwherethetumourencroachesonthefirstpartoftheduodenumorthedistalstomachandaPPPDwouldnotachieveaclearresectionmargin.
264.Whichofthefollowingcausesofacute pancreatitiscancauserecurrentboutswithoutanyobviouspathology a)SphincterOddidysfunction b)Pancreasdivisum c)Hypertriglyceridemia d)Alltheabove CorrectAnswer-DAnswer-D.AlltheaboveEtiology-Gallstones(mostcommon)Alcoholabuseisthesecondcauseofacutepancreatitis.Occultdiseaseofthebiliarytreeorpancreaticducts,especiallymicrolithiasis,sludge.HypertriglyceridemiaPancreasdivisumPancreaticcancerSphincterofOddidysfunctionCysticfibrosisDrugs-Steroids,Azathioprine,Valproate,Estrogens,L-Asparaginase,6-mercaptopurine,Sulfonamides,Tetracycline,Anti-retroviralagents,ThiazidediureticsFamilialorgeneticHyperparathyroidismHypercalemiaPostERCPMostcommoncausesinchildren:bluntabdominalinjuries,
multisystemdisease(hemolyticuremicsyndromeandinflammatoryboweldisease)biliarystonesormicrolithiasis(sludging),anddrugtoxicity
265.Pseudocystpancreasisdevelopedinhowmuchdurationfollowinganattack ofacutepancreatitis? a)Lessthan1week b)Lessthan2weeks c)3ormoreweeks d)4ormoreweeks CorrectAnswer-DAnswer-D.4ormoreweeksPseudocyststypicallyarisefollowinganattackofmildacutepancreatitis,lieoutsidethepancreas,andrepresentanAPFCthathasnotresolvedandmatured.Formationofapseudocystrequires4weeksormorefromtheonsetofacutepancreatitis.
266.Trueofumbilicalhernia- a)Mostcommoncontentislargeintestine b)Mostoftheumbilicalherniasdisappearspontaneously c)Malesareaffectedmorethanfemales d)Uncomplicatedhereniasarerepairedat1yearofagethrough aninfraumbilicalincision. CorrectAnswer-BAnswer-B.MostoftheumbilicalherniasdisappearspontaneouslyUmbilicalherniadevelopsduetoeitherabsenceofumbilicalfasciaorincompleteclosureofumbilicaldefectininfants.Umbilicalhernia(paraumbilicalhernia)isaprotusionorherniationthroughlineaalbajustaboveorbelowumbilicus.Weakestpartisumbilicalcicatrix.Contentsare-greateromentum,smallintestineandtransversecolon.CLINICALFEATURES-Femalesin5thdecadeSwellinghassmoothsurface,distinctedgesresonantwithdraggingpain.ExpansileimpulseonpatientcoughingSurgicaltreatmentisindicatedifHerniapersistsat2yearsofageorolder.Ifthedefectismorthan2cminsize.Ifitisassociatewithcomplications.Heriorrhaphy(surgeryforrepairofhernia)isdonethroughaninfraumbilicalincision.Defectisclosedwithinterruptedsuturesafterligatingthesac.

267.Treatmentofcongenitalhydrocephalus is- a)Ventriculoperitonealshunt b)Sterestacticradiosurgery c)Diuretics d)Radiotherapy CorrectAnswer-AAnswer-A.VentriculoperitonealshuntCongenitalHydrocephalus-TreatmentOptions 1. Ventriculoperitonealshunt2. Endoscopicthirdventriculostomy3. Lumbarpuncture(temporarymeasuretillashuntisput)
268.Scrotalswellingnonreduciblebut disappearswhenthechildwakesupfromsleepismostlikelytobe- a)Congenitalhydrocele b)Varicocele c)Indirectinguinalhernia d)Noneoftheabove CorrectAnswer-AAnswer-A.CongenitalhydroceleIncongenitalhydrocelethecommunicationwithperitonealcavityisusuallytoosmalltoallowherniationofintra-abdominalcontents.Usuallyhydroceleisanonreducibleswellingbutcongenitalhydrocelecanpassivelygetdrainedthroughintacttunicsvaginalisduetogravitywhilesleeping.Bothvaricoceleandindirectinguinalherniaarereduciblescotalswelling.SothemostprobableanswerisCongenitalhydrocele
269.Howmuchpercentageoftheblunt traumainjuriestospleeninadultsarecurrentlymanagednonoperatively? a)30% b)50% c)80% d)90% CorrectAnswer-CAnswer-C.80%Spleenistheintra-abdominalorganmostcommonlyinjuredinablunttraumatotheabdomen.Upto80%ofbluntsplenicinjuriescanbemanagednon-operatively.Itcanbemanagedin2ways:i)Non-operativelyThisisnowtheorderofthedayandcurrently>70%ofadultswithbluntsplenicinjuriesaremanagednon-operatively.Buttheprimaryrequirementforitis-hemodynamicstability.ii)OperativemanagementPatientswhoarehemodynamicallyunstableorarefailingnon-operativemanagement(eg.requirecontinuingtransfusion)shouldundergooperativetreatment.
270.40yearsmalepresentswithtranslucent scrotalswellinginwhichitispossibletogetabovetheswellingonexamination.Whatisthemostprobablediagnosis? a)Hydrocele b)Indirectinguinalhernia c)Varicocele d)Sebaceouscyst CorrectAnswer-AAnswer-A.HydroceleHydrocelesaretypicallytranslucentanditispossibleto'getabovetheswelling'onexaminationofthescrotum.Ahydroceleisanabnormalcollectionofseriousfluidinapartoftheprocessusvaginalis,usuallythetunica.Encystedhydroceleofthecordisasmoothovalswellingnearthespermaticcord,whichisliabletobemistakenforaninguinalhernia.Theswellingmovesdownwardsandbecomeslessmobileifthetestisispulledgentlydownwards
271.Mostprevalentsymptominpatientsof leiomyomaofesophagusis a)Pain b)Pyrosis c)Dysphagia d)Weightloss CorrectAnswer-CAnswer-C.DysphagiaCLINICALFEATURES-Asymptomatic(<5cm)Dysphagia,painMorecommoninmalesLocation-2/3rdofoesophagus
272.Whichofthefollowingfamous personalitieshadhydroceleassociatedwithhernia,whichprovedtobefatal? a)EdwardGibbon b)ThomasEdison c)MileyWright d)NeilArmstrong CorrectAnswer-AAnswer-A.EdwardGibbonEdwardgibbon(1737-1794),englishhistorianhadlargehydrocele.Thehydrocelewasassociatedwithalargescrotalherniawhichprobablywaspunctured.
273.Followingisnottrueabouttraction diverticulumofesophagus- a)Producedduetotheextraluminalforces b)Itisnotatruediverticulum c)Theoutpouchingisusuallysmallandconical d)Maydeveloptracheoesophagealfistula CorrectAnswer-BAnswer-B.ItisnotatruediverticulumIntractiondiverticulaextraluminalforces(likeinflammed&scarredperibronchial&mediastinallymphnodes)pullthefullthicknessoftheesophagusout,creatingatruediverticula.Tractiondiverticulaaremuchlesscommon.Theyaremostlyaconsequenceofchronicgranulomatousdiseaseaffectingthetracheobronchiallymphnodesduetotuberculosis,atypicalmycobacteriaorhistoplasmosis.Fibrotichealingofthelymphnodesexertstractionontheoesophagealwallandproducesafocaloutpouchingthatisusuallysmallandhasaconicalshape
274.Whichofthefollowingisfalseabout zenkersdiverticulum a)Mostpatientsareabove50yearsofage b)Itisthemostcommonesophagealdiverticulum c)Mucosaloutpouchingthroughthekillianstriangle d)Cervicalwebscanbeassociatedwithzenkersdiverticulumin 80%ofthepatients CorrectAnswer-DAnswer-D.Cervicalwebscanbeassociatedwithzenkersdiverticulumin80%ofthepatientsZenkerdiverticulumoriginatesfromtheposteriorwalloftheesophagusinatriangularareaofweakness,limitedinferiorlybythecricopharyngeusmuscleandsuperiorlybytheinferiorconstrictormuscles(ie,theKilliantriangle).Zenker'sdiverticulum,alsopharyngoesophagealdiverticulum,alsopharyngealpouch,alsohypopharyngealdiverticulum,isadiverticulumofthemucosaofthepharynx,justabovethecricopharyngealmuscle(i.e.abovetheuppersphincteroftheesophagus).Itisapseudodiverticulum(notinvolvingalllayersoftheesophagealwall).Seenin50yearsold.
275.Trueaboutesophagealcarcinomais a)Morelethalthanthecolorectalcancers b)Theseshowanincreasingtrendtowardsthenumberof squamouscellcarcinomas c)Smallersizeesophageallesionshavebettersurvival d)Asymptomaticbenignlesionsshouldbeexcisedimmediately CorrectAnswer-AAnswer-A.MorelethalthanthecolorectalcancersEtiology:alcoholandsmoking:forsquamouscellcarcinomaandadenocarcinomaachalasiaasbestosisBarrettoesophagus:foradenocarcinomacoeliacdiseaseionisingradiationcausticstricture/lyestricturePlummer-VinsonsyndromeEvenwhendetectedasasmalllesion,esophagealcancerhaspoorsurvivalbecauseoftheabundantesophageallymphaticsleadingtoregionallymphnodemetastases.
276.Patientcomplainsofintermittent dysphagiawhichisequalbothforsolidsandliquids,whichofthefollowingisthemostprobablediagnosis? a)Achalasiacardia b)Esophagealstricture c)Carcinomaesophagus d)Diffuseesophagealspasm CorrectAnswer-DAnswer-D.DiffuseesophagealspasmClinicalfeatures-HypertrophyofcircularmusclesDysphagiaChestpain
277.Earlyandlatesuspectedinstrumental perforationofoesophagusshouldfirstbeassessedusing a)Watersolublecontrastswallow b)CTScan c)Dilutebariumswallon d)MRI CorrectAnswer-AAnswer-A.WatersolublecontrastswallowIfthisisnegative,adilutebariumswallowshouldbeconsidered.ACTscancanbeusedtoreplaceacontrastswalloworasanadjuncttoaccuratelydelineatespecificfluidcollections.
278.Downhillesophagusvaricesdevelopas aresultofobstructionofseenin a)Portalvein b)Hepaticvein c)Superiorvenacava d)Inferiorvenacava CorrectAnswer-CAnswer-C.SuperiorvenacavaDownhillvaricesareproducedbytheobstructionofthesuperiorvenacava,whichresultsincollateraldrainagefromthehead,neckandupperextremityvenoussystemsintotheveinssurroundingthemidandupperthoracicesophagusandintotheazygousvein.OnCTvaricesappearasenhancingtubularregionsintheperiesophagealregion.Thisdensityisequaltothevenousbloodpool.
279.Mostcommonimpactedforeignbodyin esophagusinchildrenis a)Coin b)Foodproduct c)Krayon d)Marble CorrectAnswer-AAnswer-A.CoinCoinsarethemostcommonlyimpactedforeignbodiesinchildreninesophagusandoropharynxfollowedbythefoodproducts
280.Pantaloonherniaisalsocalledas- a)Dualhernia b)Saddlebaghernia c)Bochdalekshernia d)Retrosternalhernia CorrectAnswer-BAnswer-B.SaddlebagherniaItisalsocalleddualorsaddlebaghernia.Thistypeofherniaconsistsoftwosacsthatstraddletheinferiorepigastricartery,onesacbeingmedialandtheotherlateraltothisvessel.
281.Rolledupomentumisseenincasesof- a)Peritonealtuberculosis b)Peritonealmetastases c)Perforationperitonitis d)Malrotationofgut CorrectAnswer-AAnswer-A.PeritonealtuberculosisTheclassicappearanceofgreyishwhitemilitarynodulesscatteredovertheperitoneum.Inadditionfibrousbandsandadhesionsarecommon.Theomentummaybecomethickenedpresentingasatransverselyplacedmass(rolledupomentum).Clinicallythemostfrequentpresentationoftheperitonealdiseaseisascites.
282.Stagingofrectalcarcinomaisbestdone by- a)CTScan b)MRI c)TRUS d)Alltheabove CorrectAnswer-BAnswer-B90%ofrectalgrowthscanbefeltbyper-rectalexaminationHigh-resolutionphasedarrayexternalMRIistheinvestigationofchoiceforlocalissuesintheprimarystagingofrectalcancer(bestinvestigationforstaging)aswellasforrestagingafterNACT-RT.ItprovidesthehighestaccuracyforissuesinpretreatmentlocalstagingInvestigationofchoice?rigidsigmoidoscopyandbiopsyToassesslocalspread?TRUS(Endoluminalultrasound)Forlocalstagingandassessmentofproposedcircumferentialresectionmargin?MRI(CTisnotaccurateinlocalstaging)
283.Treatmentofchoiceforlowfistulainano is- a)Fistulotomy b)Intravenousantibiotics c)Stagedsurgicalresection d)Noneoftheabove CorrectAnswer-AAnswer-A.FistulotomyItisdividedinto2types-high&low,accordingtowhethertheirinternalopeningsisbeloworabovetheanorectalring.Theimportanceofdecidingwhetherafistulaisaloworahightypeisthatalowlevelcanbetreatedbyfistulotomy(openingthetract)withoutcausingdamagetothesphincter.
284.ExceptiontoGoodsalsruleisconsidered whentheanteriorexternalopeningismoresituatedmorethancmsfromanalmargin- a)2 b)3 c)4 d)5 CorrectAnswer-BAnswer-B.3Goodsall'sRuleIsusedtodeterminethelocationofinternalopeningAccordingtoit:Fistulaswithexternalopeninganteriortohorizontalimaginarylinedrawnacrossthemidpointofanusconnecttotheinternalopeningbyshortstraighttract.Fistulaswithexternalopeningposteriortothehorizontalline-runacurvilinearcourseandopeninternallyintotheposteriormidline.
285.Whichofthefollowingisnottrueabout medicalmanagementofuncomplicatedGERD? a)PPIsarethemosteffectivedrugtreatmentforGORD b)Householdmeasureoftiltingthebedisefficatious c)LongtermPPItherapyincreasesriskofmalignantchanges d)AdequatedoseofPPIfor8weeksistherecommended treatment CorrectAnswer-CAnswer-C.LongtermPPItherapyincreasesriskofmalignantchangesTreatment-1.Lifestylemodificationwithavoidingorcessationofsmoking,tea/coffee,alcohol.2.Drugs-PPI-antisecretorydrugsAntaacidswithalginateProkinetic-itopride(50mgTID)(Cisaprideandmosapridenotrecommendedasitcausescardiacarrhythmia)LESsphincterisrelaxedbynitrates,atropineandcalciumchannelblocker.3.Mucosaprotectiveagents-sucralfatecolloidalbismuth4.Endotherapy-plexiglassminosphere(PMMA)5.Surgery-antirefluxsurgery
286.Singledrugregimenforcarcinoma esophaguswhichshowssignificantdecreaseintumorsizein15-20%ofpatientsincorporateswhichdrug? a)Cisplatin b)Bleomycin c)Doxorubicin d)Vincristine CorrectAnswer-AAnswer-A.CisplatinSignificantreductionsinthesizeofmeasurabletumormasseshavebeenreportedin15-25%ofpatientsgivensingle-agentcisplatintreatmentandin30-60%ofpatientstreatedwithdrugcombinationsthatincludecisplatin.
287.Criticaldiameterofcaecumwhen perforationisconsideredeminentinpseudo-obstructionis- a)>7cm b)>8cm c)>9cm d)>10cm CorrectAnswer-CAnswer-C.>9cmItisdefinedaslargeboweldistentionresultingfromchronicimpairmentofmotility.Thedegreeofcolonicdistentionmaybesevereenoughtocausecaecalperforation.Imminenetperforationofcaecumisconsideredwhenthecaecaldiameterexceedsthecriticaldiameterof9cm(>9cm).
288.MostcommontypeofHiatalherniais a)Sliding b)Rolling c)Mixed d)Noneoftheabove CorrectAnswer-AAnswer-A.SlidingThereare4typesofhiatalherniaTypeI:Slidinghernia(mostcommontype-70-80%)TypeII:ParaesophagealorrollingherniaTypeIII:MixedTypeIV:Teseherniasaredistinguishedbythepresenceofotherabdominalviscerawihinthedefecti.e.omentum,transversecolon.
289.Incorrosiveinjuryofesophaguscorrect statementisallexcept a)Alkalisareusuallyingestedinlargervolumes b)Alkaliscausemoregasticdamagethanacids c)Alkalisformfibrousscar d)Acidsformeschar CorrectAnswer-BAnswer-B.AlkaliscausemoregasticdamagethanacidsIngeneral,alkalisarerelativelyodourlessandtasteless,makingthemmorelikelytobeingestedinlargevolume.Alkaliscauseliquefaction,saponificationoffats,dehydrationandthrombosisofbloodvesselsthatusuallyleadstofibrousscarring.Acidscausecoagulativenecrosiswithescharformation,andthiscoagulantmaylimitpenetrationtodeeperlayersoftheoesophagealwall.Acidsalsocausemoregastricdamagethanalkalisbecauseoftheinductionofintensepylorospasmwithpoolingintheantrum.
290.Colonoscopyisarenotindicatedin- a)MEN2B b)FAP c)HNPCC d)Cornkitecanadasyndrome CorrectAnswer-AAnswer-A.MEN2BMEN2Bdoesnotpredisposetocoloncancer.FAP,HNPCC,CronkiteCanadaSyndromepredisposetoColonCancerandhencescreeningwithcolonoscopyisneeded
291.Anindividualofage40yrspresentswith dysphagiatobothsolidsandliquidswithsymptomsofregurgitation.Patientalsocomplaintsofchestpainandweightloss.Whichofthefollowingisthepreferredinvestigationfordiagnosisofthispathology? a)BariumswallowX-ray b)BariumswallowX-raywithesophagealmanometry c)BariumswallowX-raywoithesophagealmanometrywith endoscopy d)Noneoftheabove CorrectAnswer-BAnswer-B.BariumswallowX-raywithesophagealmanometryBariumswallow-showscucumberoesophagusorbirdsbeakorpenciltipdeformity.Oesophagealmanometry-CharacteristicsofachalasiacardiaHypertensiveLOSAperistalsisinthebodyofoesophagusBariumswallowshowsadilatedesophaguswithtaperingnarrowingintheterminalendofesophagus,describedas`BIRDBEAK'appearance.
292.Contraindicationforcolostomyplanning areallexcept- a)Age>60yrs b)Stomanearskincreasesandbonyprominences c)Poorlymotivatedpatientforelectivestomy d)Stomasthroughpreviousscars CorrectAnswer-AAnswer-A.Age>60yrsThepatientmustbeexplainedtheprocedureandproperlymotivatedfortakingcareofthestomaThestomashouldpreferablybethroughrectusabdominis(topreventprolapseandparastomalhernias)ItshouldbethroughflatsurfaceofabdomenItshouldavoidskincreasesandbonyprominencesItshouldbeawayfrompreviousscars
293.Whichofthefollowingtumorsmost commonlypresentswithuppergastrointestinalbleeding? a)Primarygastriccancer b)Esophagealcarcinoma c)Metastasestostomach d)Hepaticadenocarcinoma CorrectAnswer-AAnswer-A.PrimarygastriccancerTumorbleedingaccountsforupto5%ofuppergastrointestinalbleeding(UGIB)cases.
294.Trueaboutsmallintestinaltumoris- a)Lymphomasarethemostcommonsmallintestinaltumors b)Carcinoidsaremorecommonintheduodenumandjejunum c)Adenomatouspolypsaremorecommonintheterminalileum d)Riskofdevelopingsmallboweltumorcorrelatespositivelywith colorectalcancer CorrectAnswer-DAnswer-D.RiskofdevelopingsmallboweltumorcorrelatespositivelywithcolorectalcancerThesecondmostcommonsmallboweltumoriscarcinoidaccountingfor35%ofallsmallbowelcarcinomas,90%ofwhicharelocatedintheileum.Lymphomasaccountforthethirdmostcommontumorsofthesmallbowel.Adenomatouspolypstendtooccurintheperiampullaryregionandproximaljejunum,closetotheentranceofbileandpancreaticsecretionsintosmallintestine.
295.Simplestinvestigationtobeperformedin suspectedcasesofgastriccanceris- a)Doublecontrastradiography b)Plainradiography c)CTScan d)Endoscopy CorrectAnswer-CAnswer-CCtscanInvestigationfordiagnosisofgastriccancer:*UltrasoundandCTscan-Toruleoutsecondariesintheliver.-Tolookforenlargedcoeliacnodes.-Candetectascites-guidedfluidtapandcellcytology.-TodetectKrukenbergtumour(pelvicCT).-Usefulindetectingmetastaticdisease.
296.Whichofthefollowingistrueabout gastriculcerbutnottheduodenalulcer? a)Painrarelyoccursatnight b)Melenaismorecommonthanhematemesis c)Usuallyoccursin40-50yrsofage d)Painisrelievedwithingestionoffood CorrectAnswer-AAnswer-A.Painrarelyoccursatnight
297.Nottrueabouthighlyselectivevagotomy - a)Itisalsoknownparietalcellvagotomy b)NervesofLatarjetaresacrifised c)Recurrenceratesarehigherthanvagotomyanddrainageand vagotomyandantrestomy. d)Entiregastricreservoircapacityispreserved CorrectAnswer-BAnswer-B.NervesofLatarjetaresacrifisedInHighlyselectivevagotomy(alsoknownasparietalcellvagotomyorproximalgastricvagotomy)thevagalinnervationtotheantrumandpylorus(nervesofLatarjet)arepreserved,onlythevagalsupplytotheproximaltwo-thirdsofstomach(whereessentiallyalltheparietalcellsarelocated)iscut.Thispreservesgastricmotility.
298.Bowelpuncturedduringlaproscopy a)Trocarkept b)trocarremoval c)trocarrepositioned d)None CorrectAnswer-AAnswer-A.TrocarkeptToassistinidentifyingtheprecisesiteofinjuty.
299.Whatisnotdoneincaseofpuncture woundofleftcolon a)Primarysuture b)Hemicolectomy c)Externalization d)Resectionandanastomosis CorrectAnswer-BAnswer-B.HemicolectomySmallwoundarerepairedprimarilybysimplesuturing.Moreextensivewoundaretreatedbyresectionandanastomosis.Somemayrequirecolostomy(externalization)[RefBaileyerLove25th/ep.1184]
300.Whatisthepercentagebodysurface areainvolvedinhead+faceinburns a)13 b)15 c)17 d)09 CorrectAnswer-DAnswer-D.09Accordingtoruleof9(Wallace'sformula),burnsurfaceareaiscalculatedas: 1. 9%fortheheadandneck.2. 9%foreachupperlimb.3. 9%forthefrontofeachlowerlimb.4. 9%forthebackofeachlowerlimb.5. 9%forthefrontofthechest.6. 9%forthebackofthechest.7. 9%forthefrontoftheabdomen.8. 9%forthebackoftheabdomen.9. 1%forthegenitalia. [RefBailey&Love25thlep.381]
301.PlungingRanulais a)Cysticgrowthofsublingualgland b)Lymphnode c)Atumorinfloorofmouth d)None CorrectAnswer-AAnswer-A.CysticgrowthofthesublingualglandPlungingranulaisarareformofmucousretentioncystthatarisesfromthesublingualsalivaryglands.Mucuscollectsbelowtheglandandperforatesthroughthemylohyoidmusclediaphragmtoentertheneck.
302.Sialolithiasisismostcommonlyseenin whichgland a)Parotid b)Sublingual c)Submandibular d)Minorsalivarygland CorrectAnswer-CAnswer-C.SubmandibularMostcommonsiteforsalivaryglandstone(Sialolithiasis)issubmandibulargland,especiallyductofsubmandibulargland(Wharton'sduct).
303.Whichstructureisnotpreservedin modifiedradicalmastectomy a)Cephalicvein b)Pectoralisminor c)Pectoralismajor d)BranchesoflntercostobrachialN. CorrectAnswer-AAnswer-AInthemodifiedradicalmastectomy,theprocedureinvolvesremovalofthebreastbutpreservationofthepectoralismajormuscle.Theextentofpreservationofthepectoralisminorandaxillarynodesvaries.Preservethoracodorsalnerve/vessels.
304.Duputyrensandpeyoniesarebothtype of a)Fibromatosis b)Fibroblastichyperplesia c)Burncontracture d)Myalgias CorrectAnswer-BAnswer-B.FibroblastichyperplesiaDupuytrencontractureandPeyranie'sdiseasesarefibroblastichyperplasia.Dupuytren'scontractureischaracterizedintheestablishedphasebyflexioncontractureofoneormorefingersfromthickeningandshorteningofpalmaroponeurosis.
305.MidgutVolvulussymptomsappearat a)1"week b)3rdweeks c)2"weeks d)4thweeks CorrectAnswer-AAnswer-A.1"weekMidgutvolvuluscanhappenatanyage,butmostcommonlyoccursduringthefirstfewweeksoflife.Biliousemesisisusuallythefirstsignofvolvulus.
306.Mostcommonnervedamagedduring herniarepair a)ilioinguinalnerve b)Iliohypogastric c)Genitofumoral d)None CorrectAnswer-AAnswer-A.ilioinguinalnerveTherenervesareexposedtoinjuryduringinguinalherniarepair. 1. Ilioinguinalnerve2. Genitofemoralnerve3. Iliohypogastricnerve Themostcommonlyinjurednerveisilioinguinalnerve.
307.Progressivedysphagiaisseeninall except a)CAesophagus b)Diffuseesophagealspasm c)Stricture d)Achalasiacardia CorrectAnswer-BAnswer-B.DiffuseesophagealspasmPropgressivedysphagisisseeninCAesophagusStrictureAchalasiacardiaDysphagiaequalforbothsolidsandliquidsfromonset: 1. Achalasia2. Diffuseesophagealspasm
308.Khersignisseenin a)Splenictrauma b)Hepatictrauma c)Renaltrauma d)Pacreatictrauma CorrectAnswer-AAnswer-A.SplenictraumaInsplenicrupturethepainmaybereferredtothetipoftheleftshoulder.ThisisknownasKehr'ssign.Itoccursduetoirritationoftheundersurfaceofthediaphragmwithbloodandthepainisreferredtotheshoulderthroughtheaffectedfibresofphrenicnerve(C4andC5).
309.ABPIofimminentnecrosis a)<0.3 b)0.3 c)<0.6 d)>0-6 CorrectAnswer-AAnswer-A.<0.3LowerABPIisanindicationofperipheralvasculardiseaseoflowerlimb.Criticalischemia(restpainortissuenecrosis)mostcommonlyisassociatedwithanABI<0.4.
310.Anklebrachialpressureindexis a)1.0 b)10 c)01 d)None CorrectAnswer-AAnswer-A.1.0Ankle-brachialpressureindex(ABPI)istheratioofBPinankletoBPinarm(upperlimb).NormalABPIisaround1(0.9-1.2).
311.InAinhum,constrictiondevelopsusually atthelevelofinterphalangealjointof a)Greattoe b)2ndtoe c)Littletoe d)None CorrectAnswer-CAnswer-C.LittletoeAinhumisbilateralpainfulconstrictionatthebaseof5thtoe(littletoe).Thisleadstobilateralautoamputationoflittletoes.
312.Trueaboutcarcinomaofmalebreastis a)InvasivelobularCaismostcommon b)Estrogenreceptornegative c)Seeninyoungmales d)BRCA2mutationisassociatedwithincreasedrisk CorrectAnswer-DAnswer-D.BRCA2mutationisassociatedwithincreasedriskTheaverageageofmendiagnosedwithbreastcanceris68years(oldage).Breastcancersinmalehaveestrogenreceptorsandhighestorgenlevelisariskfactorforbreastcarcinomainmales.MenwhoinheritabnormalBRCA-1andBRCA-2geneshaveanincreasedriskformalebreastcancer.Mostbreastcancersinmenareductalcarcinomas.
313.Mostcommoncarcinomabreastinmale s a)Lobularcarcinomainsitu b)Ductalcarcinomainsitu c)InfiltratingductalCa d)None CorrectAnswer-CAnswer-C.InfiltratingductalCaMostbreastcancersinmenareductalcarcinomas.
314.Pressureinlaproscopyis a)10-12mmHg b)12-14mmHg c)14-16mmHg d)16-18mmHg CorrectAnswer-BAnswer-B.12-14mmHgInlaproscopywithstandardpressurepneumoperitoneum,thegaspressureis12-14mmHg.
315.Mostcommonsiteofdirecthernia a)Hesselbach'striangle b)Femoralgland c)Nositepredilection d)None CorrectAnswer-AAnswer-A.Hesselbach'striangleItentersthecanalthroughinguinaltriangleofHesselbach.CommoninelderlyAlwaysacquiredHerniationthroughposteriorwalloftheinguinalcanalGlobular/roundinshape;descendsdirectlyforwardbulge.TrusscannotpreventprogressionofSlidingtypeofinguinalhernia
316.Lymphdrainageisincreasedfromlower limbsby a)Massasaging b)Running c)Cycling d)Sleeping CorrectAnswer-AAnswer-A.MassasagingInthehealthylimb,lymphflowislargelyduetointrinsiclymphaticcontractility,althoughthisisaugmentedbyexercise,limbmovementandexternalcompresion(massaging).Lymphaticpump/suctionpump: 1. Skeletalmusclecontraction(skeletalmusclepump);2. Squeezingactionofsmoothmuscleliningthelargerlymphatics;3. Positiveintra-abdominalandnegativeintrathoracicpressure.4. Therefore,compressionoftissuesbyobjectsoutsidethebody(e.g., massageoffoot)increaseslymphflow.
317.Noduleonthyroidwithlymphadenopathy a)Radiation b)Chemotherapy c)Excisionofnodule d)Totalthyroidectomy+MRND CorrectAnswer-DAnswer-D.Totalthyroidectomy+MRNDTotalThyroidectomyisthetreatmentofchoiceforpatientswithMTCbecauseofhighincidenceofmulticentricity.Inpatientswithpalpablecervicalnodesorinvolvedcentralnecknodes,ipsilateralorbilateralmodifiedradicalneckdissectionisrecommended.
318.Treatmentofcontaminatedwoundofleg a)Debridementandantibiotics b)Hyperbaricoxygen c)Amputation d)None CorrectAnswer-AAnswer-A.DebridementandantibioticsAfterdebridement,woundisreassessedandfurthermanagementdependsonthetypeofwound. 1. IfitissmallandcleanPrimaryclosurecanbedone2. IfitislargeandcleanCoverageprocedure(skingraft/musclepedicle graft)shouldbedone. 3. Ifitisstillcontaminated-Dailydressinganddebridementisdonetill thewoundisclean.
319.Mostimportantdiagnosticfeatureof congenitalhypertrophicpyloricstenosis a)Metabolicalkalosis b)Nonbiliousvomiting c)Jaundice d)Fever CorrectAnswer-BAnswer-B.Nonbiliousvomitingmetabolikalkalosisalsooccurs,non-biliousvomitingisthemostimportantfeatureofCHPS.
320.Criteriaforviabilityofmuscleareall except a)Colour b)Intactfascia c)Contractability d)Bleedingwhencut CorrectAnswer-BAnswer-B.IntactfasciaNonviablemusclecanbeidentifiedby4cs-Color?ConsistencyContraction,and?Circulation(bleedingoncut)
321.Gallstoneimpactedcausingintestinal obstruction a)Raynodspentad b)Hepatitis c)Gallstoneileus d)Obstructivejaundice CorrectAnswer-CAnswer-C.GallstoneileusGallstoneileusreferstomechanicalintestinalobstructionresultingfromthepassageofalargegallstoneintothebowellumen.Thestoneenterstheduodenumthroughacholecystoentericfistula.ThesiteofobstructionbytheimpactedGallstonesisusuallytheterminalendoftheileumprovidedthatthemoreproximalsmallbowelisofnormalcaliber.
322.Amiddleagedmalecomplainsofache andmumbnessandsensatedoffatigusoverhiscalfmusclesthatdevelopsonexerciseandisrelievedonrest;thisconditionisnotassociatedwith a)Smoking b)Hypocalcemia c)Peripheralarterialdisease d)Hypertensionanddiabetes CorrectAnswer-BAnswer-B.HypocalcemiaHyperlipidemiaDiabetesHypertensionCigarettesmokingAlcohol,lipoprotein(a)ChlamydiapneumonaePhyscialinactivityHerpesvirusCMVinfection
323.Setonusedinfistulainanois a)Drainingseton b)Cuttingseton c)Dissolvingseton d)None CorrectAnswer-BAnswer-B.Cuttingsetonseton(Tightseton):Madeupofsilk/prolene/monofilamentnylonandusedinfistulainanoItpromotesslowtransectionofexternalsphinctermuscleasaresultofpressurenecrosiswithminimumseparationofds.cutafistula.
324.MultipleairfluidlevelsinX-rayof abdomenisseenin a)Hollowvisceraperforation b)Pyoperitoneum c)Intestinalobstruction d)None CorrectAnswer-CAnswer-C.IntestinalobstructionTheobstructionmaybeduetoadhesion,hernia,neoplasm,foreignbodyetc.Anormalpersonhasonaverage4airfluidlevels.Mostofthemareseeninthecolon.Forthediagnosisofsmallintestinalobstruction,weshouldseemorethan2airfluidlevelsamongthedilatedloopsofsmallbowel.
325.Blunttraumachest,firststepin managementafterinitialresuscitationis a)CTscan b)Angiography c)X-raychest d)USG CorrectAnswer-CAnswer-C.X-raychestFirstinvestigationofchoiceinanytypeofchestinjuryischestX-ray.
326.Hemorrhageafterthyroidectomyisdue to a)Externalcarotidartery b)Internalcarotidartey c)Superiorthyroidartery d)Inferiorthyroidartery CorrectAnswer-CAnswerC.SuperiorthyroidarteryIsusuallyduetoslippingofligatureonthesuperiorthyroidartery.Hematomasmaycauseairwaycompromiseandmustbeevacuatedimmediately.
327.Trueaboutblindloopsyndrome,all except a)Syndromeofbacterialovergrowth b)Steatorrhea,mayeloblasticanemia&deficiencyoffatsoluble vitamins c)Surgeryisnotindicated d)14C-xyloseor14C-cholyglycinebreathtestsareindirecttests forbacterialovergrowth CorrectAnswer-CAnswer-C.SurgeryisnotindicatedFeaturesDiarrheaSteatorrheaMegaloblasticanemia(vit.B12deficiency)WeightlossAbdominalpainFatsolublevitamindeficiencyTreatmentParentralvit.B1,therapyBroadspectrumantibiotics: 1. Tetracyclineorco-amoxyclav2. Cephalexin+metronidazole3. Chloremphenical
328.%ofmalignancyinductectasiais a)Norisk b)1.5:1 c)7:1 d)10:1 CorrectAnswer-AAnswer-A.NoriskDuctectasiaCystsApocrinemetaplasiaApocrinemetaplasiaMildhyperplasiaAdenosisFibroadenoma(withoutatypia)Noincreasedriskforcancer
329.Recurrentanalfistula,mostappropriate investigationis a)EndorectalUS b)Colonoscopy c)MRI d)Proctoscopy CorrectAnswer-CAnswer-C.MRIMRIismostaccurateinvestigationfordeterminingpresenceandcourseofrecurrentanalfistulae.
330.Analfissurediagnosedby a)TRUS b)Colonoscopy c)Clinicalexamination d)Baenema CorrectAnswer-CAnswer-C.ColonoscopyAnalfissureisalinearulcerofthelowerhalfoftheanalcanal,thuscanbediagnosedbyvisuallyinspectingtheanalvergewithgentleseparationoftheglutealcleft.Thehistoryistypicalofpainandbleedingwithdefecation
331.70yroldfemalewithbleedingfrom proximalcolon a)Colitis b)Polyp c)Diverticulitis d)Cacolon CorrectAnswer-DAnswer-D.CacolonBleedingperrectuminoldagesuggeststhediagnosisofcolorectalcarcinoma.Symptomsofcolorectalcarcinomaarenon-specificandgenerallydevelopwhenthecancerislocallyadvanced.Symptomsvarywiththeanatomiclocationofthetumor.Abd.painAnemia(microcytichypochronicanemiaindicativeofirondef.)Fatigue,palpitationandevenanginapectorisMassinrightiliacfossaAcecalcarcinomacanactasleadpointinintussusception
332.A27yearoldpatientpresentedwithleft sidedabdominalpain6hoursafterRTA.HewashemodynamicallystableandFASTpositive.CTscanshowedgradeIIIsplenicinjury.Whatwillbeappropriatetreatment a)Splenectomy b)Splenorrhaphy c)Splenicarteryembolization d)Conservativemanagement CorrectAnswer-DAnswer-D.Conservativemanagement"Inearlyreports,mostinvestigatorsexpressedextremecautionregardingnonoperativemanagementofgradesIIIandIV,evenwithhemodynamicstability.Asexperiencehasaccumulated,mostfeelcomfortablewithobservingstablegradeIIIinjuries,andmanyhavebegunobservinggradeIVandVinjuries".--Sabiston
333.Retractionballseenin a)Burns b)Acutepancreatitis c)Diffuseaxonalinjury d)Tracheobronchialinjury CorrectAnswer-CAnswer-C.DiffuseaxonalinjuryAtthedistaltipoftheamputatedaxonthereisoftenandenlargedballshapedcollectionofcytoplasmtermeda'retractionball'.Axonalretractionballs-TheHallmarkofDiffuseAxonalInjury.
334.Prophylacticthyroidectomyisindicated in a)Hashimotothyroiditis b)MENtype2 c)Riedelthyroiditis d)De-Quervain'sthyroiditis CorrectAnswer-BAnswer-B.MENtype2MENtype2syndromeconsistsofmedullarycarcinomathyroid,forpatientsinlowtohigh-riskgroups,prophylacticthyroidectomyisrecommendedbyage5.ProphylacticthyroidectomyisindicatedinMEN2Bsyndrome.Medullarythyroidcarcinoma(MTC)canbeinheritedasfamilialMTC,MEN2AorMEN2Bsyndromes.TheseconditionsareautosomaldominantandoccurduetoRETproto-oncogenemutation.IndividualswithRETmutationareverylikelytodevelopMTCatayoungerage.Oncethemutationisconfirmed,itisadvisedtoundergoprophylacticthyroidectomy.
335.ProphylacticThyroidectomyforMEN2is recommendedatageof a)5years b)Before1year c)Whendetected d)Anytime CorrectAnswer-AAnswer-A.5yearsMENtype2syndromeconsistsofmedullarycarcinomathyroid,forpatientsinlowtohighriskgroups,prophylacticthyroidectomyisrecommendedbyage5.
336.Cleftpalateisideallyrepairedat a)5monthofage b)1yearofage c)Beforegoingtoschool d)6-8yearsofage CorrectAnswer-BAnswer-B.1yearofageTimingofRepairofCleftPalateAccordingtoSabiston-before12monthsAccordingtoSchwartz-at9to12monthsofageAccordingtoBailey&Love-between6and18months
337.Radiofrequencyablationis a)DerivedfromACcurrent b)Usedtoseparatefascialplanesduringsurgery c)Usesmicrowave d)Usedforhemostasis CorrectAnswer-AAnswer-A.DerivedfromACcurrentRadiofrequencyablationisamedicalprocedureinwhichpartoftheelectricalconductionsystemoftheheart,tumororotherdysfunctionaltissueisablatedusingtheheatgenerationfromhighfrequencyalternatingcurrent(intherangeof350-500KHz).
338.Malignantmelanomafalseis a)Radiosensitive b)Surgeryisthetreatmentofchoice c)Acrallentiginoushasworstprognosis d)Treatmentismidelocalexcision CorrectAnswer-AAnswer-A.RadiosensitiveMelanomaareamongthemostradioresistanttumors.Thereare4commontypeofmelanoma(theseareinorderofdecreasingfrequency) 1. Superficialspreadingtype(mostcommon)2. Nodular3. Lentigomaligna4. Acrallentiginous(leastcommon) Widelocalexcisionoftheprimarytumoristhemanagementofchoice.[RefSabistonIV/Yep.742;Schwartz10th/ep.488,490;Harrison17thlep.541;ChandrasomaTaylor3rd/ep.895]
339.DVTnotcommoncauses a)Prolongedimmobilizationa b)Extensivepelvicsxof>30minutes c)Obesity d)Agelessthan40 CorrectAnswer-DAnswer-D.Agelessthan40A)VenousThrombosis 1. Inherited:FactorVLeiden(Leidenfactor),antithrombinIII deficiency,ProteinSdeficiency,ProteinCdeficiency. 2. Acquired:Oldage,immobilization,prolongedbedrest,major surgery(e.gorthopaedicshipsurgery),majortrauma,pregnancyandpureperium,obesity,infection.B)Botharterialandvenousthrombosis 1. Inherited:Homocystinuria/homomtinemia,dysfibrinogenemia.2. Acquired:Malignancy,antiphospholipidsyndrome(lupus anticoagulant),hormonaltherapy(estrogencomponentofDCPs),polycythemia,PNH,DIG.
340.Fogarty'scatheterisusedfor a)Embolization b)Embolectomy c)Radiofrequencyablation d)Angiography CorrectAnswer-BAnswer-B.EmbolectomyFogarty'scatheterisanembolectomycatheterindicatedfortheremovaloffresh,softemboliandthrombifromvesselsinthearterialsystem.TheFogartyembolectomycatheterhasbeenfoundusefulinminimizingbloodlossinlargesurgicalproceduresaboutthehipandpelvis.Itsutilityliesintheabilitytoachievetemporaryintraluminalocclusionofthecommoniliacarterywhiletheproposedsurgicalprocedureisbeingcarriedout.
341.Choledochalcystisdilatationof a)Gallbladder b)CBD c)Hepaticduct d)Bileduct CorrectAnswer-DAnswer-D.BileductAcholedochalcystisanisolatedorcombinedcongenitaldilatationoftheextrahepaticorintrahepaticbiliarytree.
342.Falseaboutretroperitonealfibrosisis a)Ureterismostcommonlyinvolved b)Morecommoninfemales c)Primaryidiopathicformiscalledormond'sdisease d)Corticosteroidsaremainstayoftreatment CorrectAnswer-BAnswer-B.MorecommoninfemalesRETROPERITONEALFIBROSIS(ORMOND'SDISEASE)RPFisanuncommoninflammatoryconditioncharacteristedbyproliferationoffibroustissueintheretroperitoneum.Themajorstructureinvolvedare- 1. Ureter-Mostcommonlyinvolved2. Aorta3. Inferiorvenacava Corticosteroids,withorwithoutsurgery,arethemainstayofmedicaltherapy.
343.Mostcommonlyaffectedinormond's disease a)Aorta b)IVC c)Ureter d)Gonadalvessels CorrectAnswer-CAnswer-C.UreterThemajorstructureinvolvedare- 1. Ureter-Mostcommonlyinvolved2. orta3. Inferiorvenacava
344.Mucoceleofgallbladder,falsestatement is a)Complicationofgallstones b)Treatmentisearlycholecystectomy c)Obstructionatneckofgallbladder d)Gallbladderisneverpalpable CorrectAnswer-DAnswer-D.GallbladderisneverpalpableItisoneofthecomplicationsofGallstones.Causedduetoobstructionofthestoneattheneckofthebladder.Thet/tisearlycholycystectomy.
345.Investigationofchoicefor74yroldmale patientscantybleedingperrectumirregularbowelhabits a)Sigmoidoscopy b)Bariumenema c)Colonoscopy d)Bariummealfollowthrough CorrectAnswer-BAnswer-B.BariumenemaTheclinicalfeaturessuggestdiagnosisofdiverticulosis.Theyaremainlyfoundinthecolon(mainlytheleftsideofcolon)withsigmoidcolonbeingthemostcommonsite.ColonicdiverticulosisisbestdiagnosedbyBariumenema.
346.Obstructionanddilatationoflarge intestineinabsenceofanymechanicalobstruction a)Ogilvesyndrome b)Hirschsprungdisease c)Chagasdisease d)None CorrectAnswer-AAnswer-A.OgilvesyndromeOgilviesyndrome,oracutecolonicpseudo-obstruction(ACPO),isaclinicaldisorderwiththesigns,symptoms,andradiographicappearanceofanacutelargebowelobstructionwithnoevidenceofactualphysicalcauseoftheobstruction.
347.Thyroidnoduleincreasedradioisotope uptakeIOCis a)Biopsy b)Thyroidscan c)FNAC d)None CorrectAnswer-CAnswer-C.FNACisotopescanningathyroidnodulecanbe'hot,'warm'or'cold'.FNACistheinvestigationofchoiceforsolitarythyroidnodule.
348.Whatisintussuscepiens a)Theentirecomplexofintussusception b)Theenteringlayer c)Theouterlayer d)Theprocessofreducingtheintussusception CorrectAnswer-CAnswer-C.TheouterlayerAnintussusceptioniscomposedofthreeparts:Theenteringorinnertube-intussusceptumThereturningormiddletubeThesheathoroutertube-intussuscipiens
349.Gumtumorwith5cmindimensionand contralaterallymphnodeenlargementof2cm.Thereisnodistantmetasis.Thestageoftumor: a)T3N2M0 b)T2N2M0 c)T1N2Mo d)T3N3M0 CorrectAnswer-AAnswer-A.T3N2M0TNMSTAGINGOFORALCARCINOMAPrimarytumor,asfollows:T0-NoprimarytumorTis-CarcinomainsituT1-Tumor2cmorsmallerT2-Tumor4cmorsmallerT3-Tumorlargerthan4cmT4-Tumorlargerthan4cmanddeepinvasiontomuscle,bone,ordeepstructures(eg,antrum)Lymphaticnodeinvolvement,asfollows:N0:NoregionallymphnodemetastasisN1:MetastasisinasingleipsilaterallymphnodeN2a:Metastasisinasingleipsilaterallymphnode>3cmbutnot>6cmN2b:Metastasisinmultipleipsilaterallymphnodes,none>6cmingreatestdimensionN2c:Metastasisinbilateralorcontralaterallymphnodes,none>6
cmingreatestdimensionN3:Metastasisinanylymphnode>6cmTumormetastasis(M),asfollows:M0-NometastasisM1-MetastasisnotedStagingStageI:T1,N0,M0.StageII:T2,N0,M0.StageIII:T3,N0,M0T1,T2,T3,N1,M0StageIV:T4,N0,M0AnyT,N2orN3,M0AnyT,anyN,anyM
350.Mostcommoncauseofduodenal obstructioninadults a)Lymphoma b)Capancreas c)Caliver d)Cagallbladder CorrectAnswer-BAnswer-B.CapancreasMostcommoncauseofduodenalobstruction(gastricoutletobstruction)inadultsPepticulcerdisease.MostcommoncauseofgastricoutletobstructionPyloricstenosisMostcommoncancercausingduodenalobstructionPancreaticcancer.
351.Caseofdiagnosedcholecystitis presentationacutepainsharpgoingtothebackdiagnosis a)Acutepancreatitis b)Cholecystitis c)Appendicitis d)Aorticaneurysm CorrectAnswer-AAnswer-A.AcutepancreatitisClinicalfeatures-Acutesevere,refractory,upperabdominalpainradiatingtobackSomepatientsmaygainreliefbysittingorleaningforwardsIcteruscanbecausedbybiliaryobstructioningallstonepancreatitisGreyturner'ssign?bluishdiscolourationoftheflanksCullen'ssign?bluishdiscolorationaroundumbilicusFoxsign?discolorationbelowinguinalligamentShock,acuterenalfailure,ARDS,MODSLeftsidedpleuraleffusion
352.Manpresentedwithacuteabdomen, whenmanwasputinkneechestpositionhelpedtorelievethepainwhatmightbethecause. a)Acutepancreatitis b)Cholecystitis c)Superiormesentericarteryischemia d)Renal CorrectAnswer-AAnswer-A.AcutepancreatitisAcutesevere,refractory,upperabdominalpainradiatingtobackSomepatientsmaygainreliefbysittingorleaningforwardsIcteruscanbecausedbybiliaryobstructioningallstonepancreatitisGreyturner'ssign?bluishdiscolourationoftheflanksCullen'ssign?bluishdiscolorationaroundumbilicusFoxsign?discolorationbelowinguinalligamentShock,acuterenalfailure,ARDS,MODSLeftsidedpleuraleffusion
353.Mangunshotwoundinthoraxchesttube 1900mlblood,200mlofbloodlostperhr.nextstep a)Bloodtransfusion b)Thoracotomy c)PPV d)FFP CorrectAnswer-BAnswer-B.ThoracotomyInitialdrainageofmorethan1500mlbloodorongoinghemorrhageofmorethan200ml/hrover3-4hoursisgenerallycon?sideredanindicationforthoracotomy.
354.Mostcommoncauseofacuteabdomen inyounggirl a)Acuteappendicitis b)ovariantorsion c)Mitzschmerz d)Renalcolic CorrectAnswer-AAnswer-A.AcuteappendicitisMostcommoncauseofacuteabdomeninyoundwomenisacuteappendicitis.Othercausesareintestinalobstruction,diverticulitis,adnexaltorsion,ovariancystrupture/hemorrhage,PID,endometriosisanddysmenorrhea.
355.Decubitusulceris a)Venousulcer b)Wetgangrene c)Trophiculcer d)Postthromboticulcer CorrectAnswer-CAswer-C.TrophiculcerTrophiculcersareneurogeniculcerswhicharecausedbyvariousfactorssuchasimpairmentofnutritionofthetissues,inadequatebloodsupplyandneurologicaldeficit.
356.Lateralborderoftonguecarcinomaafter resection a)Chemotherapy b)Radiotherapy c)Observation d)Neckdissection CorrectAnswer-DAnswer-D.NeckdissectionIfregionallymphnodesareinvolved-Modifiedradicalneckdissectionorselectiveneckdissectionisdone.Indicationsforpostoperativeradiationtherapyincludeevidenceofperineuralorangiolymphaticspreadorpositivenodaldisease.
357.Maximumweightreductionisbywhich surgery a)BPD b)RouxenYgastricbypass c)Sleevegastrectomy d)Gastricbanding CorrectAnswer-AAnswer-A.BPDMixedprocedures 1. Gastricbypass(RouxenYgastricbypass)2. Sleevegastrectrneywithduodenalsnitch3. Implantablegastricstimulation Ingeneralmalabsorptiveproceduresleadtomoreweightlossthanrestrictiveprocedureshowevermorbidityrisksaregreater.StudieshaveshownthatitismaximumwithBiliopancreaticdiversion(BPD).
358.Mostcommonorganismassociatedwith breastabscess a)Streptococcus b)Staphylococcusaureus c)Klebsiella d)None CorrectAnswer-BAnswer-B.StaphylococcusaureusStaphylococcusaureusisthemostcommoncauseofbreastabscess.MostarecausedbyS.aureusand,ifhospital-acquired,arelikelytobepenicillin-resistant.Staphylococcusaureuscausestheclottingofmilkintheblockedductandmultiply.Ductinitiallygetsblockedbyepithelialdebrisorbytheretractednipple.
359.Allistrueaboutskullfractureexcept a)Puppesrulegivesthesequenceoffracture b)Pondfractureisamilddepressedfracture c)Fissuredfractureismostcommon d)Skullfracturesareduetotraction CorrectAnswer-BAnswer-B.PondfractureisamilddepressedfractureTypesofSkullfracture- 1. Linearorfissuredfracture:arethemostcommonskullfractures.2. DepressedFracture3. Comminutedfracture4. Pondorindentedfracture-Thisisasimpledentoftheskull, occurringonlyinskullofinfants,foreg.inobliquebulletwounds. 5. Gutterfractures6. Ringorforamenfractures Puppe'sruleIthelptoassessthechronologicalorderinwhichfracturewereformed,sincelaterfractureswilltypicallystopatpreviouslyformedones.
360.Cecumformstheposteriorwallofwhich hernia a)Slidinghernia b)Rollinghernia c)Incisionalhernia d)Hiatushernia CorrectAnswer-AAnswer-A.SlidingherniaSlidingherniaisdefinedasanyherniainwhichpartofthesac(usuallytheposterior)isformedbythewallofaviscus.Cecumisinvolvedontherightsideandsigmoidcolonisinvolvedonleftside.Itshouldbeclearlyunderstoodthatthecaecum,appendixorpartofcolonwhollywithinahernialsacdoesnotconstituteaslidinghernia(Thevisceramustformawallofthesactobetermedasslidinghernia).[Ref:Bailey&Love26th/ep.956]
361.Inlastdecade,duodenalulcerandits morbidityisreduceddueto a)Lifestylemodification b)EradicationofHpylori c)Protonpumpinhibitors d)None CorrectAnswer-CAnswer-C.ProtonpumpinhibitorsInlastdecades,withtheintroductionofprotonpumpinhibitorsandincreasedknowledgeofperforatedpecticulcer(PPU)etiologytheincidenceofPPUhasreportedlydecreasedinwesterncountries.
362.Posteriorduodenalulcerisrelatedto a)Gartoduodenalartery b)Spleenicartery c)LeftgastricArtery d)Supmesentricartery CorrectAnswer-AAnswer-A.GartoduodenalarteryGastroduodenalarteryisthemostcommonarteryinvolvedinduodenalulcerhaemorrhage.AlsorememberPepticulceristhemostcommoncauseofmassiveuppergastrointestinalbleed(Duodenalulcers>Gastriculcer)
363.Postioninsurgeryforpilonidalsinus a)Sim's b)Tredelenberg c)Lithotomy d)Jackknife CorrectAnswer-DAnswer-D.JackknifeFormostprocedures,patientisplacedinpronejackknifepositionwithslighttrendelenburg.JackknifepositionAnanatomicalpositioninwhichthepatientisplacedonthestomachwiththehipsflexedandthekneesbentata90?angleandthearmoutstretachedinfrontofthepatient.
364.Lengthofflexiblesigmoidoscope a)30cm b)40cm c)60cm d)70cm CorrectAnswer-CAnswer-C.60cmThelengthofrigidsigmoidoscopeis25cm,whereasflexiblesigmoidoscopeare60cmlong.[RefBailey&Love25th/ep.1221;www.medicinenet.com]
365.Herniacommoninchildren a)Umbilical b)Bockdelac c)Morgagni d)Inguinal CorrectAnswer-AAnswer-A.Umbilical"Congenitalumbilicalherniasrepresentthemostcommonabdominalwalldefectininfantandchildren".Theincidenceofumblicalherniasis5-10%inwhitechildrenandmaybeashighas25-50%inblackchildren.
366.Gangrenenotcausedby a)Frostbite b)Burger'sdisease c)Varicoseveins d)Atherosclerosis CorrectAnswer-CAnswer-C.VaricoseveinsImportantcausesofgangreneDiabetesPVD(Buerger'sdisease)TraumaObesityAtherosclerosisRaynaud'sdiseaseFrostibite
367.Bilateralbreastcarcinoma a)Invasivelobular b)Infiltractiveductal c)Ductalcainsitu d)None CorrectAnswer-AAnswer-A.InvasivelobularLobularcarcinoma(invasive)isfrequentlybilateral.Histologichallmark:patternofsingleinfiltratingtumorcellsoftenonlyonecellinwidthorinlooseclustersorsheets.Signetringcellscommon.Lobularcarcinomahaveadifferentpatternofmetastasiscomparedtootherbreastcancers.
368.Drainageofcervicalabscessisan exampleof- a)Cleancontaminatedwound b)Cleanuncontaminatedwound c)Uncleanuncontaminatedwound d)Dirtyinfectedwound CorrectAnswer-DAnswer-D.DirtyinfectedwoundClassIV:Thisclassofwoundisconsidereddirty-contaminated.Theseincludewoundsthathavebeenexposedtofecalmaterial.CervicaldrainageabscessisanexampleofClassIVwound.
369.Mostcommonlocationofectopic salivaryglandis- a)Cervicallymphnodes b)Anteriormediastinum c)Posteriortriangle d)Parathyroidgland CorrectAnswer-AAnswer-A.CervicallymphnodesSalivaryglandheterotopiaiswheresalivaryglandacinicellsarepresentinanabnormallocationwithoutanyductsystem.Themostcommonlocationisthecervicallymphnodes.Otherreportedsitesofheterotopicsalivaryglandtissuearethemiddleear,parathyroidglands,thyroidgland,pituitarygland,cerebellarpontineangle,softtissuemedialtosternocleidomastoid,stomach,rectumandvulva
370.Allofthefollowingareparaneoplastic syndromesforrenalcellcarcinomaexcept- a)Fever b)Anaemia c)Amyloidosis d)AcanthosisNigricans CorrectAnswer-DAnswer-D.AcanthosisNigricansHypertensionAbnormalliverfunction(Staufferssyndromeienonmetastatichepaticdysfunction)HypercalcemiaNeuromyopathyAmyloidosisIncreasedESR(MCparaneoplasticsyndrome)DysfibrogenemiaGalactorrhoeaFeminizationandmasculanization
371.ESWLiscontraindicatedinwhichofthe followingstones- a)Cysteinestones b)OxalateStones c)Uratestones d)Phosphatestones CorrectAnswer-AAnswer-A.CysteinestonesContraindicationsforESWL- 1. UncorrectedBleedingdisorder2. Cysteinestones3. Weightgreaterthan300pounds4. Pregnancy5. CardiacPacemaker6. SevereOrthpaedicdeformity7. UTI8. SevererenalFailure
372.
Traumaticopticneuropathyduetoclosedheadtraumacommonlyaffectswhichpartofopticnerve- a)Opticcanal b)Intraocularpart c)Intracranialpart d)Optictract CorrectAnswer-AAnswer-A.OpticcanalIndirectopticnerveinjuryStudieshaveshownthatforcesappliedtothefrontalboneandmalareminencesaretransferredandconcentratedintheareaneartheopticcanal.Thetightadherenceoftheopticnerve'sduralsheathtotheperiosteumwithintheopticcanalisalsothoughttocontributetothissegmentofthenervebeingextremelysusceptibletothedeformativestressesoftheskullbones.Suchinjuryleadstoischemicinjurytotheretinalganglioncellswithintheopticcanal.
373.Allofthefollowingaremedicalusesof erythropoietinexcept- a)Treatmentofanaemiaassociatedwithrenaldisease b)Chemotherapyinducedanemia c)AnaemiaAssociatedwithCrohn'sDisease d)MegaloblasticAnaemis CorrectAnswer-DAnswer-D.MegaloblasticAnaemisUsesoferythropoetinAnaemiaofrenalfailure.Anaemiaassociatedwithchemotherapyfordiseaseslikemylodysplasia.Anaemiaassociatedwithinflammatoryboweldisease.
374.
Whichofthefollowingisthemostconservativeneckdissection- a)Supraomohyoidneckdissection b)Radicalneckdissection c)Modifiedradicalneckdissection d)Allareconservative CorrectAnswer-AAnswer-A.SupraomohyoidneckdissectionSelectiveneckdissection-PreservationofanyofthelevelsIthroughVduringneckdissectionisknownasSelectiveneckdissection-Theprinciplebehindpreservationofcertainnodalgroupsisthatspecificprimarysitespreferentiallydraintheirlymphaticsinapredictablepattern.TypesofSNDincludethesupraomohyoidneckdissection,thelateralneckdissection,andtheposterolateralneckdissection.
375.Whichofthefollowingismostmalignant tumor? a)GlioblastomaMultiforme b)Meningioma c)Osteochondroma d)Giantcelltumor CorrectAnswer-AAnswer-A.GlioblastomaMultiformePrimaryBrainTumours1.Gliomas(43%).a.Astrocytomasarethecommonesttype.Theyareusuallymalignant.GradeI?CysticGradeII?DiffuseGradeIII?AnaplasticGradeIV?Glioblastomamultiforme(MC)2.Meningiomas(18%)3.Schwannoma(8%)4.Pituitarytumors(12%)5.Craniopharyngiomas(5%)6.Bloodvesseltumors(2%)OthertumorsTheyarepinealregiontumors,pituitaryadenomas,craniopharyngiomas,choroidplexustumors,etc.
376.Mosttissuereactionisseenwith a)PlainCatgut b)Polydiaxonone c)Silk d)Chromiccatgut CorrectAnswer-CAnswer-C.SilkReactiontocatgutdependsonthestageofabsorptionandismainlyhistocyticintype.Nonabsorbablesuturesareencapsulatedbyarimofconnectivetissue,whilenearthesuturehistocytes,giantcellsandlymphocytesarefound.Thisismostmarkedwithsilkandcotton,lesssowithDacron,andleastwithnylonandwire.
377.
Supraomohyoiddissectionisatypeof- a)Selectiveneckdissection b)Modifiedradicalneckdissection c)Radicalneckdissection d)Posterolateraldissection CorrectAnswer-AAnswer-A.SelectiveneckdissectionSelectiveneckdissection-PreservationofanyofthelevelsIthroughVduringneckdissectionisknownasSelectiveneckdissection-Theprinciplebehindpreservationofcertainnodalgroupsisthatspecificprimarysitespreferentiallydraintheirlymphaticsinapredictablepattern.TypesofSNDincludethesupraomohyoidneckdissection,thelateralneckdissection,andtheposterolateralneckdissection.
378.Polydiaxononesutureisnormally absorbedin a)2weeks b)4weeks c)6weeks d)6months CorrectAnswer-DAnswer-D.6monthsRetains70%ofitsoriginaltensilestrengthat2weeks,50%at4weeks,and25%at6weeks.Prolongeddermalsupportforatleast6monthshasbeenassociatedwithdecreasedscarspreading
379.Alagillesyndrome-allofthefollowing aretrueexcept- a)MutationinJAG1AndNotch2geneareseen b)CanCauseAutoimmunehepatitis c)AutosomalRecessiveDisease d)Valvularanomalisofheartseen CorrectAnswer-CAnswerC.AutosomalRecessiveDiseaseAutosomaldominantdiseaseJAGIandNOTCH2mutationAffectstheliverandHeartHeartPulmonarystenosisTetralogyofFallotVSDOverridingaorta
380.Whichofthefollowingisanabsorbable suture a)Polyglactin b)Silk c)Polyester d)Ethilon CorrectAnswer-AAnswer-A.PolyglactinAbsorbablesutures- 1. Catgut2. Polyglactin3. Polyglyconate4. Polyglycolicacid5. Polydioxanone6. Poliglecaprone
381.Whichistheinvestigationofchoicefor stagingofalowerlimbsarcoma? a)MRI b)CTScan c)PETScan d)PETCT CorrectAnswer-AAnswer-A.MRIMRI:Investigationofchoiceforsofttissuesarcomasinextremities.CTSCAN:Investigationofchoiceforretroperitonealsarcomas.
382.Allofthefollowingaretrueaboutthe bareareaoftheliverexcept a)Infectioncanspreadfromtheabdominaltothoraciccavityat thisarea b)Itisnotasiteofportocavalanastomosis c)Formedbythereflectionsofcoronaryligaments d)Itistriangularinshape CorrectAnswer-BAnswer-B.ItisnotasiteofportocavalanastomosisThecoronaryligamentsrepresentreflectionsofthevisceralperitoneumcoveringtheliverontothediaphragm.Assuch,betweenthetwolayersofthecoronaryligamentliesthebareareaoftheliver,andisattachedtothediaphragmbyareolartissue.ThebareareaoftheliverisstillcoveredbyGlisson'scapsule,thefibrouscapsulethatsheathestheentireliver.Bareareaisasiteofportocavalanastomoses
383.Allofthefollowingarepremalignant conditionsexcept- a)Bowen'sDisease b)SenileKeratosis c)XerodermaPigmentosum d)PyodermaGangrenosum CorrectAnswer-DAnswer-D.PyodermaGangrenosum Bowen'sDisease LupusvulgarisLongstandingchroniculcer(e.g.marjolin's ActinicKeratosis ulcers) Xeroderma Followingburn,venousulcer,oldscaretc. pigmentosumSenileKeratosis
384.Earlystageofnonsmallcelllungcancer canbetreatedby a)Surgicalresection b)Surgicalresectionwithadjuvantchemotherapy c)Radiotherapy d)Immunotherapy CorrectAnswer-CAnswer-C.RadiotherapyCystoscopyisendoscopyoftheurinarybladderviatheurethra.Itiscarriedoutwithacystoscope.
385.Chimericchemotherapyisbeing investigatedforthetreatmentofwhichmalignancy? a)Leukemia b)RenalCellCarcinoma c)CAPancreas d)GlioblastomaMultiforme CorrectAnswer-AAnswer-A.LeukemiaArtificialTcellreceptors(alsoknownaschimericTcellreceptors,chimericimmunoreceptors,chimericantigenreceptors(CARs))areengineeredreceptors,whichgraftanarbitraryspecificityontoanimmuneeffectorcell.Typically,thesereceptorsareusedtograftthespecificityofamonoclonalantibodyontoaTcell.TwoCAR-TtherapieshavebeengrantedthreetotalFDAapprovalforthetreatmentofpatientswithhematologicmalignancies.ThemostadvanceddatatodateconcernCARTtherapiesforthetreatmentofpatientswithmultiplemyelomaandchroniclymphocyticleukemia.Amongthefirststudiesinvestigatinganti-CD19CARTtherapeuticefficacywasacasereporttreatingapatientwithCLL.
386.AnteriorMediastinalnodesareincluded inwhichleveloflymphnodes- a)I b)III c)V d)VII CorrectAnswer-DAnswer-D.VIILevelsofnecklymphnodesLevelI,submentalandsubmandibulargroupLevelII,upperjugulargroupLevelIII,middlejugulargroupLevelIV,lowerjugulargroupLevelV,posteriortrian-glegroupLevelVI,anteriorcompartment.LevelVII,anteriorandsuperiormediastinal
387.InCeliacarterycompressionsyndrome whichstructureisthemaincauseofcompression- a)MedianArcuateLigament b)RectusSheath c)DeepInferiorEpigastricartery d)LacunarLigament CorrectAnswer-AAnswer-A.MedianArcuateLigamentMedianarcuateligamentsyndrome(MALS,alsoknownasceliacarterycompressionsyndrome,celiacaxissyndrome,celiactrunkcompressionsyndromeorDunbarsyndrome)isaconditioncharacterizedbyabdominalpainattributedtocompressionoftheceliacarteryandpossiblytheceliacgangliabythemedianarcuateligament.Theabdominalpainmayberelatedtomeals,maybeaccompaniedbyweightloss,andmaybeassociatedwithanabdominalbruitheardbyaclinician.Itisalsocalledceliacarterycompressionsyndrome.
388.TheBipolarcauteryispreferredover monopolarcauteryinthefollowingsurgeriesexcept a)HandSurgery b)SurgeryaroundPenis c)SurgeryoftheHip d)Surgeryaroundtheface CorrectAnswer-CAnswer-C.SurgeryoftheHipBipolarcauterypreferredin:HandsurgerySurgeryaroundappendageslikepenisSurgeryaroundtheface
389.AllofthefollowingaretrueaboutNissen Fundoplicationexcept a)ItisdoneforGERD b)Reinforcmentisdoneonlyintheanteriorhalf c)Upperpartofstomachisplicatedaroundtheloweresophagus d)Itisdoneforparaesophagalhiatushernia CorrectAnswer-BAnswer-B.ReinforcmentisdoneonlyintheanteriorhalfInafundoplication,thegastricfundus(upperpart)ofthestomachiswrapped,orplicated,aroundthelowerendoftheesophagusandstitchedinplace,reinforcingtheclosingfunctionoftheloweresophagealsphincter.Theesophagealhiatusisalsonarroweddownbysuturestopreventortreatconcurrenthiatalhernia,inwhichthefundusslidesupthroughtheenlargedesophagealhiatusofthediaphragm.InaNissenfundoplication,alsocalledacompletefundoplication,thefundusiswrappedalltheway360degreesaroundtheesophagus.
390.A65yearoldmalepresentingwithacure pancreatitisisnowhavingrefractoryhypoxia.TheXRAYofchestwouldshow a)Bilateralinfiltrates b)Pneumatocoeles c)Groundglassappearances d)Hilarlymphadenopathy CorrectAnswer-AAnswer-A.BilateralinfiltratesSystemiccomplicationsofpancreatitisHypovolemicshockDICARDSDiabetes
391.WhichIsFalseaboutstressurinary incontinence- a)Morecommoninmen b)Itoccursduringincreasedabdominalpressure c)Itisduetoweakeningofpelvicfloormuscles d)Prostatesurgerymaybeacause CorrectAnswer-AAnswer-A.MorecommoninmenLeakingurineuponsuddenincreaseinabdominalpressure.Itismostcommontypeinwomen.CausesareWeakeningofpelvicfloormuscles/urethra/sphinctermuscles.BPH,CAprostateorprostatesurgery.
392.Whichofthefollowingtypesofshock wiilusuallyhavewarmperipheralextremities? a)HypovolemicShock b)NeurogenicShock c)CardiogenicShock d)AnaphylacticShock CorrectAnswer-BAnswer-B.NeurogenicShockBradycardiaHypothermiaLossofsympathetictone-->arterious&venousvasodilationWarmanddryskin(theheartrateslowsdown,aswellaswarm,dry,andflushedskinduetovenouspoolingsecondarytolossofvasculartone)Othersallshockhavecoldextremities(anaphylatic,cardiogenicandhypovolemic)
393.Antibioticprophylaxisisbestgiven- a)1daybeforesurgery b)2hoursbeforesurgery c)Beforethetimeofincision d)Onlypostoperatively CorrectAnswer-CAnswer-C.BeforethetimeofincisionAboutprophylacticantibioticspointstoknoware?Bydefinitionprophylaxisislimitedtothetimepriortoandduringtheoperativeprocedure.bemosteffectivetheprophylacticantibioticagentshouldbeadministeredintravenouslybeforetheincisionismadesothatthetissuelevelsarepresentatthetimethewoundiscreatedandexposedtothebacterialcontamination.
394.Whichofthefollowingisafeatureof crushsyndrome- a)Hypokalemia b)Hypercalcemia c)Myoglobinuria d)Hypophosphatemia CorrectAnswer-CAnswer-C.MyoglobinuriaFeaturesofcrushsyndrome:HyperkalemiaHypocalcemiaHyperphosphatemiaHyperuricemiaLacticacidosisMyoglobinuriaIncreasedBUN&creatinine.
395.Ayoungmalepresentswithatesticular massontherightside.TheAFPiselevatedwhiletheHCGisnormal.Themostappropriatenextstepis a)Biopsy b)USG c)Orchidectomy d)WaitandWatch CorrectAnswer-BAnswer-BApainlesstesticularmassispathognomicforatesticularmalignancy.USGofthetestisisindicatedwheneveratesticularmalignancyisconsideredandforpersistentorpainfultesticularswelling.
396.CrushSyndromeisassociatedwithallof thefollowingfeaturesexcept- a)Hypercalcemia b)Hyperkalemia c)Increasedserumcreatinine d)Myoglobinuria CorrectAnswer-AAnswer-A.HypercalcemiaFeaturesofcrushsyndrome:HyperkalemiaHypocalcemiaHyperphosphatemiaHyperuricemiaLacticacidosisMyoglobinuriaIncreasedBUN&creatinine.
397.A45yearoldmalepresentingwithpenile cancerextendinguptotheglanspenisistreatedwith a)PartialPenectomywith2cmmargin b)Circumcision c)PartialpenectomywithInguinalnodesexploration d)Partialpenectomywith4cmmargin CorrectAnswer-AAnswer-A.PartialPenectomywith2cmmarginThegoaloft/tininvasivepenilecarcinomasiscompleteexcisionwithadequatemargins:a)ForlesioninvolvingtheprepuceSimplecircumcisioniseffective.b)ForlesionofglansordistalshaftPartialpenectomywitha2cmmargin(lessaggressivesurgicalresectionssuchasMohsmicrographicsurgeryandlocalexcisionsdirectedatpenilepreservationcanbedone).c)ForlesioninvolvingtheproximalshaftorwhenpartialpenectomyresultsinapenilestumpofinsufficientlengthforsexualfunctionordirectingtheurinarystreamTotalpenectomywithperinealurethrostomy
398.Whichofthefollowingstonesare commonwithinfection? a)Struvite b)XanthineStones c)Cysteinestones d)CalciumOxalatestones CorrectAnswer-AAnswer-A.StruviteCalciumphosphatecanbecombinedwithammoniumandmagnesiumtoformthetriplephosphatecalculusieCalciumamnomiummagnesiumphosphatealsok/astruvitestones.Smoothanddirtywhiteandsoltitary.Seeninalkalineurineespwithproteusinfectionwhichsplitureatoammonia.
399.A45yearoldmaleisdiagnosedwith carcinomapenis.Thesurgeonmustlookoutforwhichlymphnodes a)Paraaortic b)Externaliliac c)InternalIliac d)Inguinal CorrectAnswer-DAnswer-D.InguinalMorethan50%ofpatientspresentwithenlargedinguinallymphnodes(buthalfofthesearereactiveenlargementd/tsepsis).ThepresenceandtheextentofmetastasistotheinguinalregionisthemostimportantprognosticfactorforsurvivalinpatientswithCaPenis.
400.HighFlyingprostateisasignof- a)ExtraperitonealBladderrupture b)IntraperitonealBladderRupture c)MembranousUrethralInjury d)BulbarUrethralInjury CorrectAnswer-CAnswer-C.MembranousUrethralInjuryTheprostateisattachedtopubisbypuboprostaticligamentanddisruptionofthepuboprostaticligamentwithacompleteruptureoftheurethracanleadtoafloatingprostate--Vermooten'ssign(highflyprostate).Basedonascendingurethrogram,posteriorurethralinjuryisclassifiedas(McCallum-Colapintoclassification). 1. TypeI:Elongationofposteriorurethra,butintact2. TypeII:Prostate"pluckedoff''membranousurethra withextravasationofurineabovesphincteronly--Floatingprostate--Vermooten'ssign 3. TypeIII:Totaldisruptionoftheurethrawithextravasationof urinebothaboveandbelowthesphincter.
401.Whichofthefollowingisfalseabout undescendedtestis? a)Morecommonontherightside b)Hormonaltherapyiseffective c)Increasedriskofmalignancy d)Secondarysexualcharacteristicsarenormal CorrectAnswer-BAnswer-B.HormonaltherapyiseffectiveInundescendedtestis-thetestesisarrestedinsomepartofitspathtothescrotum.InEctopictestis-thetestisisabnormallyplacedoutsideitspath.Retractiletestis-ininfancy80%ofinapparenttestisareretractiletestisandrequirenot/t.Approx70-77%ofcryptorchidtesteswillspontaneouslydescend,usuallyby3monthsofage.Morecommoninpreterm,smallforgestationalage,LBW&twinneonates.MorecommononRt.SideSecondarysexualcharactersticsarenormal
402.RPLNDandChemotherapymaybeused inmanagementof a)NonseminomatousGermcelltumoursoftestis b)Nongermcelltumours c)SeminomatousGermcelltumours d)Lymphomatestis CorrectAnswer-AAnswer-A.NonseminomatousGermcelltumoursoftestisRPLNDretroperitoneallymphnodedissectionExtragonadalGermcelltumorsInfrequentlyGCTsarisefromanextragonadalsite.Theyhavepoorprognosis.Theyaretreatedbychemotherapy.
403.BPHinvolves- a)Centralzone b)Peripheralzone c)Transitionalzone d)Prostatecapsule CorrectAnswer-CAnswer-C.TransitionalzoneCarcinomaofprostatearisesmostcommonlyintheperipheralzoneBenignprostatichyperplasiaoriginatesinthetransitionzoneBPHtypicallyaffectsthesubmucousgroupofglandsinthetransitionalzone,forminganodularenlargement.Eventually,thisovergrowthcompressesthePZglandsintoafalsecapsuleandcausestheappearanceofthetypical'lateral'lobes.Ref-BaileyandLoveVolume-1/Edition27th
404.Whichofthefollowingisatumour markerforbladdercancer? a)AFP b)CEA c)Bladdersurfaceprotein d)NuclearMatrixprotein22 CorrectAnswer-DAnswer-D.NuclearMatrixprotein22TumourmarkersinCABladderNuclearmatrixprotein22BTATRAK
405.A45yearoldlawyerpresentswithpain intheabdomenmoresointheepigastricregionthatworsenswitheatingspicyfoodandisrelievedbybendingforward.Complicationsoftheabovementionedconditioncouldbeallexcept a)Perforation b)Bleeding c)GastricOutletObstruction d)SplenicVeinThrombosis CorrectAnswer-DAns.-D.SplenicVeinThrombosisBleeding,perforationandgastricoutletobstructionarethecomplicationsofpepticulcer.Perforation:Thisallowsstomachcontentstoescapeintotheperitoneum,causingperitonitis.Itismorecommoninduodenalthaningastriculcers.Gastricoutletobstruction:Themostcommoncauseisanulcernearthepylorus,butoccasionalcasesareduetoantralcanceroradulthypertrophicpyloricstenosis.Bleeding
406.Prostatecancerthatislimitedtothe capsuleandnottheurethrawouldbesatgedas- a)Tx b)T1 c)T2 d)T3 CorrectAnswer-CAnswer-C.T2
407.A65yearoldmalepresentswithCA prostate.ThetumourislimitedtothecapsuleanditispalpableonPRexamination.ThepatientisdiagnosedasstageT1b.Thebesttreatmentwouldbe- a)Radicalprostatectomy b)Chemotherapy c)Palliativeradiotherapy d)Orchidectomy CorrectAnswer-AAnswer-A.RadicalprostatectomyT1bT1c,T2RadicalprostactectomyorradiotherapyExternalbeamradiotherapy-T1orlowT2diseaseBranchytherapy-lowT1disease
408.A55yearsoldmalewithaknoenhistory ofgallstonespresentswithchiefcomplintsofsevereabdominalpainandelevatedlevelsofserumlipasewithperiumbilicalecchymosis.Allofthefollowingareprognosticcriteriatopredictseverityoftheconditionexcept a)Age b)SerumLDH c)Basedeficit d)SerumGGT CorrectAnswer-DAnswer-D.SerumGGT
409.If90%ofjejunumandiliumisremoved thenallofthefollowingfeatureswillbeseenexcePt- a)Steatorrhoea b)Diarrhoea c)Weightgain d)Megaloblasticanemia CorrectAnswer-CAnswer-C.WeightgainChangesseeninTerminalilealresectionResectionofterminalileumresultsinmalabsorptionofbilesaltsandVit.B12whicharenormallyabsorbedinthisregionVitBIZMalabsorption:MegaloblasticanemiaBilesaltsmalabsorption:-Unabsorbedbilesaltsescapeintocolonwheretheystimulatefluidsecretionfromthecolon,resultinginwaterydiarrhoea.Reductioninbilesaltpoolleadtosteatorrheaandmalabsorptionoffatsolublevitamins(duetomalabsorptionoffat).Unabsorbedfattyacidsbindwithcalcium,andthusleadtoincreasedconcentrationoffreeoxalates(oxalatesbindwithcalciumnormallyandthereforeescapewithoutintestinalabsorption)Freeoxalatesareabsorbed.Increasedconc.ofbodyoxalatesleadtooxalatekidneystones.
410.Mostcommonlocationofbreastcancer is a)Lowerinnerquadrant b)Nipple c)Upperinnerquadrant d)Upperouterquadrant CorrectAnswer-DAnswer-D.UpperouterquadrantBreastcancerisfoundmostfrequentlyintheupperouterquadrant.Leastfrequentlyinlowerinnerquadrant.
411.A22yearoldwomancomeswithanon progressivemassinaleftbreastsince6months.Therearenoassociatedsymptoms.Examinationshowsamobilemassnotattachedtotheoverlyingskinorunderlyingtissue.Thepossiblediagnosisis a)Fibroadenoma b)CystasarcomaPhylloides c)ScirrhousCarcinoma d)Fibroadenosis CorrectAnswer-AAnswer-A.FibroadenomaClinically,theypresentasfirmmassesthatareeasilymovable.(MOUSEINTHEBREAST)Theyslideeasilyundertheexaminingfingersandmaybelobulatedorsmooth.Mammographyisoflittlehelpindiscriminatingbetweencystsandfibroadenomas;however,ultrasonographycanreadilydistinguishbetweenthembecauseeachhasspecificcharacteristics.FNAbiopsycanalsobeusedtoconfirmtheimagingfindings
412.PerihepaticfibrosisoccurringinFitz HughCurtisSyndromeisdueto- a)PelvicInflammatoryDisease b)BileDuctInjry c)ChronicAlcoholism d)ViralHepatitis CorrectAnswer-AAnswer-A.PelvicInflammatoryDiseaseItisthoughttoresultfromdirectintraperitonealspreadofinfectiontowardstheperihepaticregionfrominitialpelvicinflammation/infectionTrichomonasvaginalis,UreaplasmaurealyticumandMycoplasmahominismayalsocauseFHCS.
413.45yearoldfemaleunderwentmodified radicalmastectomywithaxillaryclearanceforCAbreast.Aftersurgeryshecouldnotliftherarmabovehead.Whichnerveislikelytobeinjured? a)Intercostobrachialnerve b)LongthorasicnerveofBell c)NervetolatissimusDorsi d)LateralPectoralnerve CorrectAnswer-BAnswer-B.LongthorasicnerveofBellOverheadabductioniscausedby:- 1. Serratusanterior:suppliedbylongthoracicnerve.2. Trapezius:suppliedbyspinalaccessorynerve.
414.Whichisthemostcommontypeofmale breastcancer? a)InfiltratingDuctalCarcinoma b)LobularCarcinoma c)MucinousCarcinoma d)ColloidCarcinoma CorrectAnswer-AAnswer-A.InfiltratingDuctalCarcinomaIttendstopresentasalumpandismostcommonlyaninfiltratingductalcarcinoma.Morethan90%ofcasesareinfiltratingductcarcinoma,about10%areductalcarcinomainsitu(DCIS).Theknownpredisposingcausesincludegynecomastiaandexcessendogenousorexogenousestrogen.
415.A25yearoldmaleisreceiving conservativemanagementforanappendicularmasssince3daysnowpresentswitharisingpulserate,tachycardiaandfever.Themodeofmanagementmustbe- a)Ochsnersherrenregimen b)Continueconservativemanagement c)Proceedtolaparotomyandappendicectomy d)Intravenousantibiotics CorrectAnswer-CAnswer-C.ProceedtolaparotomyandappendicectomyIfanappendixmassispresentandtheconditionofthepatientissatisfactory,thestandardtreatmentistheconservativeOchsner?Sherrenregimen.10-20%needemergencyoperationduetospreadinginfection.
416.CABreastmaylocallyspreadtoallofthe followingmusclesexcept a)PectoralisMajor b)PectoralisMinor c)LatissimusDorsi d)SerratusAnterior CorrectAnswer-CAnswer-C.LatissimusDorsilatissimusDorsiisusedforbreastreconstructionandmayrarelybeinvolvedinCABreastlocalSpread'Musclesinvolvedinbreastcancer 1. Pectoralmuscles:Pectoralismajor,Pectoralisminor.2. Musclesofchestwall:Intercostalmuscles,serratusanterior
417.Underwhatguidelinesistreatment startedforapatientpresentingwithappendicularmassonCTScan? a)OchsnerSherrenRegimen b)Conservativemanagementanddischarge c)ImmediateLaprotomy d)Kocher'sRegimen CorrectAnswer-AAnswer-A.OchsnerSherrenRegimenOchsnerSherrenregimenistheexpectantmanagementgivingtoapatientwithanappendixmass.Itisexpectantbecauseitisexpectedthatthesymptomsandsignsthepatientpresentedwithwillimproveduringthecourseofthemanagementandthepatientmaylaterbescheduledforelective/intervalappendicectomy.
418.Whichofthefollowingistrueabout menetrier'sdisease a)Itispremalignantcondition b)Thereisincreasedgastricacidsecretion c)Atrophiedmucosalfoldsareseen d)Affectsthestomachandsmallintestines CorrectAnswer-AAnswer-A.ItispremalignantconditionM?n?trierdiseaseisarare,acquired,premalignantdiseaseofthestomachcharacterizedbymassivegastricfolds,excessivemucousproductionwithresultantproteinloss,andlittleornoacidproduction.Thedisorderisassociatedwithexcessivesecretionoftransforminggrowthfactoralpha(TGF-).
419.Apatientpresentswithdifficultywith swallowingliquidsbutnotsolids.Thebestinvestigationtomakeadiagnosisis a)Endoscopy b)Endoscopicultrasound c)Manometry d)PETCT CorrectAnswer-CAnswer-C.ManometryH/Oofdysphagiawithmoretoliquidsthansolidssuggestsachalasiacardia.Inallotherobstructivecausesofesophagus,dysphagiaforsolidsisseenfirstManometryisusedfordiagnosis.
420.Spontaneousesophagealruptureismost commonin a)Belowthediagphragmaticaperture b)Pharyngoesophagaljunction c)Abovethediagphragmaticaperture d)Atthecrossingofthearchofaorta CorrectAnswer-CAnswerC.AbovethediagphragmaticapertureInstrumentalperforationiscommoninthepharynxordistalesophagus.Spontaneousrupturemayoccurjustabovethediaphragmintheposterolateralwalloftheesophagus.
421.Aneurysmaldilationofthesmallbowelis seenin a)SmallbowelLymphoma b)GallStoneIleus c)DuodenalAtresia d)SjogrensSyndrome CorrectAnswer-AAnswer-A.SmallbowelLymphomaAneurysmaldilatation:30%,itoccursduetoreplacementofmuscularisbytumourorinfiltrationofmyentericnerveplexusDespitetheextensiveinvolvement,smallbowelobstructionisuncommonbecauseoflackofdesmoplasticreaction,andperforationisrare
422.Whichisthemaincontraindicationfora liverbiopsy? a)Thrombocytopenia b)Hemangioma c)Ascites d)All CorrectAnswer-DAnswer-DContraindicationsforliverbiopsyincludethefollowing:Increasedprothrombintime(PT),internationalnormalizedratio(INR)greaterthan1.6Thrombocytopenia,plateletcountlowerthan60,000/LAscites(transjugularroutepreferred)[10]Difficultbodyhabitus(transjugularroutepreferred)SuspectedhemangiomaSuspectedechinococcalinfectionUncooperativepatient
423.Treatmentofchoiceofmucinous carcinomaofGallBladderconfinedtothelaminapropriais- a)Simplecholecystectomy b)Extendedcholecystectomy c)Cholecystectomywithwedgeresectionofliver d)Chemotherapyonly CorrectAnswer-AAnswer-A.SimplecholecystectomyTreatment-StageIandII-simplecholecystectomyStageIII?cholecystectomy+adjacenthepaticresection(atleast2cmdepth)+regionallymphadenectomyPoorprognosis
424.Allofthefollowingareprinciplesof negativepressurewoundtherapyexcept a)Stabilizationofwoundenvironment b)Clearanceofinfection c)Macrodeformationofthewound d)Decreasededema CorrectAnswer-BAnswer-B.ClearanceofinfectionFourprimaryeffectsofNPWTonwoundhealing:Macro-deformation-drawingthewoundedgestogetherleadingtocontraction.Stabilisationofthewoundenvironment-ensuringitisprotectedfromoutsidemicroorganismsinawarmandmoistenvironment.Reducedoedema-withremovalofsofttissueexudates.Micro-deformation-leadingtocellularproliferationonthewoundsurface.
425.MostcommonprecipitantofRaynaud's phenomenonis a)Exposuretocold b)Exposuretoheat c)Psychosocialtriggers d)Exertion CorrectAnswer-AAnswer-A.ExposuretocoldIsdsofyoungwomen(F:Mratiois5:1)Commonlytheupperlimbsareaffectedspeciallythefingers(thethumbisgenerallyescaped)Exposuretocoldismaintrigger
426.NigroRegimenisusedfor- a)AnalCarcinoma b)RectalCarcinoma c)SigmoidColonCarcinoma d)DuodenalCarcinoma CorrectAnswer-AAnswer-A.AnalCarcinomaNigroregimenreferstocombinedchemotherapyandradiotherapyforprimarytreatmentofmalignanttumorsoftheanalcanal.
427.Theinvestigationofchoicefor dysphagiais a)Endoscopy b)ManometricStudy c)CtScan d)BariumSwallow CorrectAnswer-AAnswer-A.EndoscopyTheinvestigationofchoiceinisendoscopy.Bariumswallowisthefirstinvestigation.
428.A54yearoldwomanisdiagnosedas havingcarcinomaoftherenalpelvisofsizelessthan4cmwithoutanymetastasis.Thebesttreatmentoptionis a)Partialnephrectomy b)RadicalNephrectomy c)Chemotherapyandimmunaotherapy d)PalliativeRadiotherapy CorrectAnswer-AAnswer-A.PartialnephrectomyPartialnephrectomyisnowbeingusedasprimarysurgicaltherapyforpatientswithtumourlessthan4cminsize,earlierRadicalnephrectomywasthet/tofchoicefortumorofanysize.
429.A35yearoldmalecamewithjaundice, palpablemassintherighthypochondriumnotassociatedwithpain.Theprobablediagnosisis- a)CarcinomaAmpullaofVater b)Acutecholecystitis c)ChronicCholecystitis d)ChronicPancreatitis CorrectAnswer-AAnswer-A.CarcinomaAmpullaofVaterCourvoisier'slawstatesthatinthepresenceofanenlargedgallbladderwhichisnontenderandaccompaniedwithjaundicethecauseisunlikelytobegallstones.Thisisbecausegallstonesareformedoveralongerperiodoftime,andthisresultinashrunken,fibroticgallbladderwhichdoesnotdistendeasily.
430.A60yearmalepresentedwithjaundice, palestools,darkurineandmassintheepigastricregion.Whichofthefollowingdiagnosisinunlikely? a)Pancreaticcancer b)BiliaryCancer c)PeriampullaryCancer d)ChronicCholecystitis CorrectAnswer-DAnswer-D.ChronicCholecystitisChroniccholecystitisischaracterizedbyrepeatedattacksofpain(biliarycolic)thatoccurwhengallstonesperiodicallyblockthecysticduct.
431.Allofthefollowingmayleadtoagall bladdercarcinomaexcept- a)GallBladderPolyps b)Typhoidcarriers c)Exposuretocarcinogenslikenitrosamine d)EchinococcusGranulosusInfection CorrectAnswer-DAnswer-D.EchinococcusGranulosusInfectionRiskfactorsforgallbladder 1. Gallstones2. Adenomatousgallbladderpolyps(particularlypolypslargerthan10 mm) 3. Calcified(porcelain)gallbladder4. Choledochalcyst5. Estrogens6. Anomalouspancreaticobiliaryductjunction7. Exposuretocarcinogens(azotoulene,nitrosamine)8. Typhoidcarriers9. Sclerosingcholangitis 10. Cholecystoentericfistula
432.Allofthefollowingmayleadto pneumatoceleformationexcept a)Staphylococcalpneumonia b)Positivepressureventilation c)Hydrocarboninhalation d)ARDS CorrectAnswer-DAnswer-DPneumatoceleformationoccursasasequelatoacutepneumonia,commonlycausedbyStaphylococcusaureus.However,pneumatoceleformationalsooccurswithotheragents,includingStreptococcuspneumoniae,Haemophilusinfluenzae,Escherichiacoli,groupAstreptococci,Serratiamarcescens,Klebsiellapneumonia,adenovirus,andtuberculosis.Noninfectiousetiologiesincludehydrocarboningestion,trauma,andpositivepressureventilation.Inprematureinfantswithrespiratorydistresssyndrome,pneumatocelesresultmostlyfromventilator-inducedlunginjury.
433.Whichofthefollowingstomaisformed inHartman'sprocedure? a)EndColostomy b)EndIliostomy c)LoopIliostomy d)Caecostomy CorrectAnswer-AAnswer-A.EndColostomyHartman'sprocedureThisisacompromisetypeofresectioninwhichtherectumand/orsigmoidcolonareresectedandanendcolostomydone.Anastomosisandperinealdissectionofthedistalrectumandperineumarenotdone.
434.Mostcommonsiteofcolorectal carcinomais- a)Rectum b)SigmoidColon c)AscendingColon d)DescendingColon CorrectAnswer-AAnswer-A.RectumRectosigmoidregionisthemostcommonsiteforcolorectalcarcinoma.Rectumisinvolvedin38%ofcasesandsigmoidcolonisinvolvedin21%cases.
435.Whichofthefollowingstagesoflip carcinomadonothavenodalinvolvement? a)T2N1 b)T3NO c)T1N1 d)T2N2 CorrectAnswer-BAnswer-B.T3NO
436.Whichofthefollowingistrueabout MalloryWeisstear- a)Itisamucosaltearnotextendingthroughthemusclelayer b)Itismorecommoninwomenthanmen c)Itiscommoninyoungindividuals d)Itisassociatedwithachalasiacardia CorrectAnswer-AAnswer-A.ItisamucosaltearnotextendingthroughthemusclelayerAMallory-Weisstear(MWT)isforcefulorretchingvomitingmayproducealongitudinalmucosaltearimmediatelybelowsquamocolumnarjunctionatthecardiaorgastroesophagealjunctionmainlyandalsoinoesophagus.Seeninmales,alcholicsandpregnantfemales.Themeanageismorethan60and80%aremen.Hyperemesisgravidarum,whichisseveremorningsicknessassociatedwithvomitingandretchinginpregnancy,isalsoaknowncauseofMallory-Weisstear.
437.A40yearoldmalepresentedwitha penetratingtraumatochest.Heisdyspnoeicwithdistendedneckveinswithhypotensionandmediastinumisshiftedtooppositeside.Thereisasuckingwoundoverthechest.Themostappropriatemanagementwouldbe a)Insertionofalargeboreneedleinthe2"ICSinthemid clavicularline b)FluidResuscitation c)StartingInotropicsupport d)EndotrachealIntubation CorrectAnswer-AAnswer-A.Insertionofalargeboreneedleinthe2"ICSinthemidclavicularlineThisisacaseoftensionpneumothorax.Treatmentconsistsofimmediatedecompression,initiallybyrapidinsertionofalarge-boreneedleintothesecondintercostalspaceinthemidcalvicularlineoftheaffectedhemithorax,andthenfollowedbyinsertionofachesttubethroughthefifthintercostalspaceintheanterioraxillaryline.
438.A40yearoldmalewithchesttrauma presentswithbreathlessness,decreasedrespiratorysoundsontherightside,hyperresonanceonpercussionanddistendedneckveins.Thepossiblediagnosisis a)TensionPneumothorax b)CardiacTamonade c)FlailChest d)MyocardialInfarction CorrectAnswer-AAnswer-A.TensionPneumothoraxTensionpneumothoraxoccurswhenairbecomestrappedinthepleuralspaceunderpressure.Itdevelopswhena'one-wayvalve'airleakoccurseitherfromthelungorthroughthechestwall.causesarePenetratingchesttrauma,Bluntchesttrauma,Iatrogeniclungpunctures(e.g.duetosubclaviancentralvenepuncture)andMechanicalpositive-pressureventilationClinicalClinicalsignsandsymptomsincludedyspnea,tachypnea,hypotension,diaphoresis,anddistendedneckveins.
439.Whichofthefollowingisanindication forthoracotomyincaseofhemothorax? a)Persistantdrainageof250ml/hr b)Totaloutputof1000m1ofblood c)Fallingbloodpressure d)Shiftofmediastimumtotheoppositeside CorrectAnswer-AAnswer-A.Persistantdrainageof250ml/hrItisclassifiedaccordingtotheamountofblood.Minimalhemothorax350mlModeratehemothorax350-1500mlMassivehemothorax1500mlormoreTotalhemorrhagicoutputexceeds1500mlofblood
440.A55yearoldwomanpresentedwith historyofrecurrentepisodesofrightupperabdominalpainforthelastoneyear.Shepresentedtoemergencywithhistoryofjaundiceandfeverfor2days.Onexamination,thepatientappearedtoxicandhadabloodpressureof100/60mmHg.Shewasstartedonintravenousantibiotics.Ultrasoundoftheabdomenshowedpresenceofstonesinthecommonbileduct.Whatwouldbethebesttreatmentoptionforher- a)ERCPandbileductstoneextraction b)Laparoscopiccholecystectomy c)Opensurgeryandbileductstoneextraction d)Lithotripsy CorrectAnswer-AAnswer-A.ERCPandbileductstoneextractionMethodsofbiliarydecompressiona.ERCPwithsphincterotomyandstoneextractionItistheprocedureofchoice,Earlyendoscopyallowsnotonlydiagnosisbycholangiographyanddirectvisualizationoftheampullabutalsopermitsbiliarydecompressionbysphincterotomyandstoneextraction.
Ifthestonescannotberemoved,anasobiliarycatheterorstentisinsertedtodecompressthebiliarytract.b.Percutaneoustranshepaticroute(PTC)PTCisperformedifTheendoscopicprocedurehasfailedornotavailable.Iftheobstructionismoreproximalorperihilar.Ifthereisastrictureinabiliaryentericanastomosis.c.Surgicalbiliarydecompression
441.Rigler'ssignissuggestiveof- a)Pneumothorax b)Pneumoperitoneum c)Peritonitis d)Hemothorax CorrectAnswer-BAnswer-B.PneumoperitoneumBothsidesofbowelbecomevisiblebecauseoffreegasonanabdominalx-ray.ThisisknownasRigler'sSign.
442.A70yearpldmalecomplainingofper rectalbleedingwasdiagnosedofhavingrectal/anorectalcancer.Thedistalmarginofthetumorwas5cmfromtheanalvergethetreatmentofchoicewouldbe- a)PalliativeRadiotherapy b)Abdominoperinealrecection c)Lowanteriorresection d)LocalExcision CorrectAnswer-CAnswer-C.LowanteriorresectionSurgeriesPre-operativeneo-adjuvantradiotherapyinresectablerectalcancerreducestheincidenceoflocalrecurrenceAdjuvantchemotherapyimprovessurvivalinnode-positivecasesAnterior(low)resection:sphinctersavingprocedure,midrectumProximal2/3rdofrectum(lesions6cmsabovethedentateline/2ormorecmsaboveanalcanal)WelldifferentiatedtumoursizetumourT1/T2,NOtumoursRectosigmoidtumorsandupperthirdrectaltumors:Highanteriorresection(rectumandmesorectumaretakentoamargin5cmdistaltothetumourandcolorectalanastomosisisperformed)Tumoursinthemiddleandlowerthirdsofrectum:completeremovalofrectumandmesorectum(TME-totalmesorectalexcision)
Abdominoperinealresection(Mike'sprocedure)-LowerrectumHartmann'sprocedure?forelderlyandunstablepatientswhocannotwithstandlongprocedureofAPRColonoscopyisalwaysperformedeitherbefore(forsynchronoustumours)orwithinafewmonths(formetachronoustumours)ofsurgicalresectionfortumourdetection
443.Allofthefollowingaretrueabout Hirschsprungdiseaseexcept- a)AbsenceofGanglioncellsintheinvolvedsegment b)Swenson,DuhamelandSoavearesurgicalproceduresforthis condition c)Mainlypresentsininfancy d)Thenonperistalticaffectedsegmentisdialted CorrectAnswer-DAnswer-D.ThenonperistalticaffectedsegmentisdialtedInHirchprung'sdiseasetheabsenceofganglioncellsgivesrisetoacontractednonperistalticsegmentwithadilatedhypertrophiedsegmentofnormalcolonaboveit.Almostallcasespresentininfancyandchildhood.Approximatelyhalfarediagnosedintheneonatalperiodandmostoftheremainderarediagnosedinearlychildhood.Inthenewbornperiod,mostcommonsymptomsareabdominaldistentionandfailuretopassmeconium.Olderpatientsmayexperiencechronicorintermittentconstipation.Occasionally,infantsmaypresentwithadramaticcomplicationofHirschprung'sdiseasecalledenterocolitis.Surgeryisalwaysrequired.Thebasicprincipleisremovalofmostoralloftheaganglionicsegmentandanastomosingtheganglionicboweltotherectum
444.MostcommoncauseofgoiterinIndiais a)DiffuseEndemicGoitre b)PapillaryCarcinoma c)ToxicMultinodularGoitre d)Hashimoto'sThyroiditis CorrectAnswer-AAnswer-A.DiffuseEndemicGoitreThelackoftheiodineleadstodecreasedsynthesisofthyroidhormonesandacompensatoryincreaseinTSH,whichinturnleadstofollicularcellhypertrophyandhyperplasiaandgoitrousenlargement-Diffusehyperplasticgoiter.Mostlypatientsareeuthyroid.
445.Allofthefollowingareclinicalfeatures suggestiveoftrachea-esophagalfistulaexcept- a)ChokingandCoughing b)Regurgitation c)Cyanosis d)Fever CorrectAnswer-DAnswer-D.FeverRepeatedepisodesofcoughing,chokingandcyanosisoccuronfeedingwithTOF.
446.InSubtotalThyroidectomy,Whatistrue a)Removalofonelobeandisthmus b)Removalofbothlobesleavingbehind6-8gramsoftissue c)Removalofentirethyroidwithcervicallymphnodes d)Removalof1lobewithisthmusandthesecondlobepartially CorrectAnswer-BAnswer-B.Removalofbothlobesleavingbehind6-8gramsoftissueSubtotalthyroidectomy--Removalofmajorityofbothlobesleavingbehind4-5grams(equivalenttothesizeofanormalthyroidgland)ofthyroidtissueononeorbothsides--thisusedtobethemostcommonoperationformultinodulargoitre.
447.Mostimportantpresentingfeatureof periampullarycarcinomais- a)Jaundice b)Pain c)WeightLoss d)PalpableMass CorrectAnswer-AAnswer-AHallmarkofpresentationofperiampullarycarcinomaisobstructiveJaundice.Jaundicesecondarytoobstructionofthedistalbileductisthemostcommonsymptomthatdrawsattentiontoampullaryandpancreaticheadtumors.Itischaracteristicallypainlessjaundicebutmaybeassociatedwithnauseaandepigastricdiscomfort.
448.Allofthefollowingaresurgicaloptions inmanagementofesophagealcarcinomaexcept- a)IvorLewisApproach b)Mckeown'sApproach c)Transhiatalremoval d)Sistrunkoperation CorrectAnswer-DAnswer-D.SistrunkoperationIvorlewisoperationissubtotalesophagectomydoneforCaesophagusinlowerthirdofesophagusIntranshiatalesophagectomy(THE)Esophagusisremovedthroughthediaphragmatichiatusbymakingincisionsintheabdomenandtheneck.McKeownsSurgery-3IncisionApproach
449.A65yearoldfemalepresentswitha swellingintheneckdiagnosedasasolitarythyroidnodule.Thepatientisinvestigatedandascanshowsincreaseduptakeofiodine.SerumT3andT4areelevated.Mostprobabediagnosisis a)BenignColloidNodule b)ToxicAdenoma c)Follicularadenoma d)ToxicMultinodulargoitre CorrectAnswer-BAnswer-B.ToxicAdenomaElavatedthyroidhormonelevelswithahyperfunctioningnoduleissuggestiveofatoxicadenoma.Acoldnoduleismorelikelytobemalignantthenahotnodule.
450.Chronichemolyticanaemiaisassociated withwhichofthefollowing- a)BrownPigmentstoneofthegallbladder b)BlackPigmentstoneofthegallbladder c)UricacidRenalCalculus d)IntestinalObstruction CorrectAnswer-BAnswer-B.BlackPigmentstoneofthegallbladderHemolysiscausesblackpigmentstoneofgallbladder.Chronichemolyticstates(eg.hereditaryspherocytosis,sicklecelldisease).
451.Allofthefollowingistrueaboutheart transplantationexcept- a)Immmunosupressionisstartedpreoperatively b)Itisonlyorthotopicandnotheterotopic c)ABeatingheartcadaver/donorisneeded d)HighPulmonaryarterialresistanceinacontraindication CorrectAnswer-BAnswer-B.ItisonlyorthotopicandnotheterotopicTypesoftransplant:HeterotopicOrthotopicAtypicalhearttransplantationbeginswhenasuitabledonorheartisidentified.Theheartcomesfromarecentlydeceasedorbraindeaddonor,alsocalledabeatingheartcadaver.
452.Thyroidnoduleina65yearoldmalewho isclinicallyeuthyroidismostlikelytobe a)Follicularadenoma b)FollicularCarcinoma c)ThyroidCyst d)Multinodulargoiter CorrectAnswer-AAnswer-A.FollicularadenomaMostcommonsolitarythyroidnoduleisbenigncolloidnodule.2"dmostcommoncauseofsolitarythyroidnoduleisfollicularadenoma.
453.Incaseofpolytraumawithmultiple injuriestothechest,neckandabdomen,highestpriorityisgivento a)Stabilizationofcervicalspine b)Staringofkoids c)Vasopressors d)Assesingdisability CorrectAnswer-AAnswer-A.Stabilizationofcervicalspine1.Primarysurvey-itshouldbedonefirstwhenanypolytraumapatientpresentsinthefollowingorder:1.1Airwaymaintenancewithcervicalspineprotection1.2Breathingandventilation1.3Circulationwithhemorrhagecontrol1.4Disability/Neurologicassessment1.5Exposureandenvironmentalcontrol2.Secondarysurvey-completehistoryandexamination.
454.Achildpresentedwithbluntabdominal trauma,thefirstinvestigationtobedoneis- a)USG b)CTScan c)CompleteHemogram d)AbdominalXray CorrectAnswer-AAnswer-A.USGU/Sistheinvestigationofchoiceinbothstableaswellasunstablepatients.
455.Whichofthefollowingprecancerous conditionsiftreatedwouldnotleadtocancer a)CervicalintraepithelialNeoplasia b)Ductalcarcinomainsituofbreast c)LobularCarcinomainsituofbreast d)VaginalintraepithelialNeoplasia CorrectAnswer-AAnswer-A.CervicalintraepithelialNeoplasiaThecancers,whichhavewellknownprecancerouscondition,canbediagnosedandtreatedatpre-cancerousstagetopreventthedevelopmentofcancer:- 1. CervixGIN2. ColonPolyp
456.Vacuumassistedclosureis contraindicatedinwhichofthefollowingconditions- a)Chronicosteomyelitis b)Largeamountofnecrotictissuewitheschar c)Abdominalwound d)Surgicalwounddehiscence CorrectAnswer-BAnswer-B.LargeamountofnecrotictissuewithescharNegative-pressurewoundtherapy(NPWT),alsoknownasavacuumdressingorV.A.C.dressing("vacuumassistedclosure"),isatherapeutictechniqueusingasuctiondressingtoremoveexcessexudationandpromotehealinginacuteorchronicwoundsandsecond-andthird-degreeburns.ContraindicationsforNPWTuse 1. Malignancyinthewound2. UntreatedOsteomyelitis3. Nonentericandunexploredfistulas4. Necrotictissuewithescharpresent5. Exposedbloodvessels,anastomoticsites,organsandnervesinthe periwoundarea(mustavoiddirectfoamcontactwiththesestructures)
457.Allofthefollowingarecausesof UrothelialCarcinomasExcept- a)Smoking b)Industrialsolvents c)Exposuretothorotrast d)Alcoholconsumption CorrectAnswer-DAnswer-D.AlcoholconsumptionIndustrialdyesorsolventsExcessiveanalgesicintake,Balkannephropathy,andThoseexposedtoThorotrast,(acontrastagentpreviouslyusedforretrogradepyelography
458.MostcommonsiteforCysticHygromais - a)Lowerthirdofneck b)Overlyingtheparotidgland c)AlongtheZygomaticProminence d)Postauricular CorrectAnswer-AAnswer-A.LowerthirdoftheneckMostcystichygromasinvolvethelymphaticjugularsacsandpresentintheposteriorneckregion.Theothercommonsitesaretheaxillary,mediastinum,inguinal,andretroperitonealregions,andapproximately50%ofthempresentatbirth.Cystichygromasusuallypresentassoftcysticmassesthatdistortthesurroundinganatomy,includingtheairway,whichcanresultinacuteairwayobstruction.
459.Thesizebeyondwhichtheriskof ruptureofanabdominalaneurysmsignificantlyincreasesisgreaterthan- a)5.5cm b)6cm c)6.5cm d)7cm CorrectAnswer-AAnswer-A.5.5cmHighriskAAAEndovascularaneurysmalrepair(EVAR)AAArepair-5.5cm/>5cmfemalesSymptomaticaneurysm
460.TrueaboutMALTomais a)Theyaresecondarygastriclymphomas b)H.Pyloriinfectionisariskfactor c)Commonlyseeningastriccardia d)TheyareatypeofTcelllymphoma CorrectAnswer-BAnswer-B.H.PyloriinfectionisariskfactorStomachisthemostcommonextranodalsiteoflymphoma.Itisoftwotypes-Primary&SecondaryMostoftheprimarylymphomas(about60%)ariseinMALT(mucosaassociatedlymphoidtissue).MALTisusuallyassociatedwithchronicH.pyloriinfection.Solow-gradeMALTlymphomasarethoughttoarisebecauseofchronicH.pyloriinfection.Theselowgradelymphomasmaylaterondegeneratetohighgradelymphomas.mucosaassociatedlymphoidtissue.ItisusuallyprimaryGIlymphoma(4%ofgastriclymphoma);ofnon-Hodgkin'sBcelltype.
461.Acysticmassatthebaseofumbilical cordinaneonatecouldbe a)AllantoicCyst b)Meckel'sDiverticulum c)Ventralmesogastrium d)CysticHygroma CorrectAnswer-AAnswer-A.AllantoicCystAllantoiccystsareatypeoftruecystoftheumbilicalcord.Theallantoisformsfromthepartofthefetalyolksacthateventuallybecomestheprimitivehindgut(thecloaca).Thecloacadividesintothehindgutposteriorlyandtheurogenitalsinusanteriorly.
462.Chvosteksigncouldbeseenafter- a)Gastrojejunostomy b)Totalthyroidectomy c)SubtotalThyroidectomy d)HellersCardiomyotomy CorrectAnswer-BAnswer-B.TotalthyroidectomyChvostek'ssignreferstothecontractionoftheipsilateralfacialmusclesonpercussionofthefacialnervebelowthezygoma.Itisduetohypocalcemiaandonecommoncauseofhypocalcemiaisacomplicationduringthyroidectomies.Theparathyroidglandproducesaparathyroidhormone(PTH),andresectioncanleadtoadecreaseinPTHlevels.PTHregulatescalciumlevelsinthebody;hence,areductioninitsserumlevelwillleadtohypocalcemia.So,amongthemostcommonpostoperativecomplicationsfollowingthyroidsurgeryishypocalcemia.
463.AnIncisionalwoundhealsby a)PrimaryHealing b)SecondaryHealing c)DelayedprimaryHealing d)Reepithelization CorrectAnswer-AAnswer-A.PrimaryHealingPrimaryHealingPrimarywoundhealingorhealingbyfirstintentionoccurswithinhoursofrepairingafull-thicknesssurgicalincision.Thissurgicalinsultresultsinthemortalityofaminimalnumberofcellularconstituents.
464.Afullthicknesswoundthatisnot suturedhealsby a)PrimaryHealing b)SecondaryHealing c)DelayedprimaryHealing d)Reepithelization CorrectAnswer-BAnswer-B.SecondaryHealingSecondaryHealingAthirdtypeofhealingisknownassecondaryhealingorhealingbysecondaryintention.Inthistypeofhealing,afull-thicknesswoundisallowedtocloseandheal.Secondaryhealingresultsinaninflammatoryresponsethatismoreintensethanwithprimarywoundhealing.Inaddition,alargerquantityofgranulomatoustissueisfabricatedbecauseoftheneedforwoundclosure.Secondaryhealingresultsinpronouncedcontractionofwounds.
465.Foodcancommonlygetobstructedin theesophagusatallofthefollowinglocationsexcept a)Crossingofleftbronchus b)Crossingofarchofaorta c)Diagphragmaticaperture d)Crossingofthehemiazygousvein CorrectAnswer-DAnswer-D.CrossingofthehemiazygousveinPharyngoesophagaljunction-6inches9
466.ReactionaryHemorrhageoccursdueto- a)Dislodgementofclot b)Infection c)Damagetoabloodvessel d)Pressurenecrosis CorrectAnswer-AAnswer-A.DislodgementofclotDislodgementofclotLigatureslipNoramlisationofBPandvasodilation
467.Allofthefollowingistrueabout congenitalhypertrophicpyloricstenosisexcept a)RamStedtPyloromyotomyisthetreatmentofchoice b)NonBiliousvomitingisseen c)Metabolicacidosisoccurs d)Morecommoninmales CorrectAnswer-CAnswer-C.MetabolicacidosisoccursCharactersticallythefirstbornmalechildisaffected.Theconditonismostcommonlyseenat4weeksafterbirthrangingfromthe3rdweektoonrareoccasionNon-biliousvomiting,becomingincreasinglyprojectile,occursoverseveraldaystoweeksInfantsdevelopametabolicalkalosiswithseveredepletionofpotassiumandchlorideions.Treatment-Surgery:Fredet-Ramstedtpyloromyotomy.(Initthepyloricmassissplitwithoutcuttingthemucosa)
468.Whichofthefollowingiscorrect managementofabdominalcompartmentsyndrome a)Antihypertensives b)UrgentOpeningofthesurgicalwoundandapplicationofthe Bogotabag c)UrgentFasciotomy d)Waitandmonitorfor24hours CorrectAnswer-BAnswer-B.UrgentOpeningofthesurgicalwoundandapplicationoftheBogotabagAbdominalcompartmentsyndromeisasurgicalemergencyandtreatmentincludesrapiddecompressionoftheelevatedintraabdominalpressurebyopeningtheabdominalwoundandperformingatemporaryclosureoftheabdominalwallwithmeshoraplasticbag.Permanentclosureisdone5to7dayslaterwhentheconditionresolves.
469.Transplantedkidneyisrelocatedto whichregionintherecepient'sbody? a)Retroperitonealrgion b)Lumbarregion c)Epigastrium d)BesidethedysfunctionalKidney CorrectAnswer-AAnswer-A.RetroperitonealrgionThemostcommonlocationforplacingakidneytransplantisinretroperitonealiliacfossai.InmostcasesKidneyisplacedretroperitoneallyandtheiliacarteriesandveinsareusedforperfusionandureteristransplanteddirectlyintobladder.
470.Claudicationduetopopliteofemoral incompetenceisprimaryseenin a)Thigh b)Calf c)Buttocks d)Feet CorrectAnswer-BAnswer-B.CalfAortaandCommonIliac-ButtocksFemoralArtery-ThighSuperficialfemoralartery-CalfandpoplitealarteryPosteriortibialArtery-Feet
471.Balthazarscoringsystemisusedfor? a)AcutePancreatitis b)AcuteAppendicitis c)Acutecholecystitis d)Cholangitis CorrectAnswer-AAnswer-A.AcutePancreatitisBalthazarscoreisusedintheCTseverityindex(CTSI)forgradingofacutepancreatitiswhichhastwocomponents.Usingimagingcharacteristics,BalthazarandassociateshaveestablishedtheCTseverityindex.ThisindexcorrelatesCTfindingswiththepatient'soutcome1)GradingofpancreatitisA:Normalpancreas0B:Enlargementofpancreas1C:Inflammatorychanges2D:Ill-definedsinglefluidcollection-43E:Twoormoreill-definedfluidcollections42)PancreaticnecrosisNone030%2>30-50%4>50%6
472.Triagesystemusedfor a)Burn b)Earthquack c)Polytrauma d)Floods CorrectAnswer-CAnswer-C.PolytraumaTheusualprincipleoffirstcome,firsttreated",isnotfollowedinmassemergencies.Triageconsistsofrapidlyclassifyingtheinjuredandthelikelyhoodoftheirsurvivalwithpromptmedicalintervention.Higherpriorityisgrantedtovictimswhoseimmediateorlong-termprognosiscanbedramaticallyaffectedbysimpleintensivecare.
473.Ilealobstructionduetoroundworm obstructiontreatmentis a)Resectionwithendtoendanastomosis b)Resectionwithsidetosideanastomosis c)Enterotomy,removalofwormsandprimaryclosure d)Diversion CorrectAnswer-CAnswer-C.Enterotomy,removalofwormsandprimaryclosureDiversionisthefirststepincaseofcolonicobstruction,followedbyresectionandanastomosisofaffectedsegmentandthenclosureofdiversioncolostomyatalaterdate.IntestinalluminalobstructionsuchasduetoBezoarsorfecolithsofwormintestationsaredealtwithbyenterotomyandremovalfollowedbyprimaryclosure.
474.Surgeryforperforationduetoround wormis a)Resectionwithendtoendanastomosis b)Resectionwithsidetosideanastomosis c)Primaryclosure d)Diversion CorrectAnswer-AAnswer-A.Resectionwithendtoendanastomosis
475.Whichisacleansurgery a)Herniasurgery b)Gastricsurgeru c)Cholecystectomy d)Rectalsurgery CorrectAnswer-AAnswer-A.HerniasurgeryCleanWound(ClassI)Includethoseinwhichnoinfectionispresent;onlyskinmicroflorapotentiallycontaminatethewound.Nohollowviscusisentered.NoinflammationExamples:Herniarepair,breastbiopsy
476.Steroidisinjurioustowoundwhen given a)On1stday b)<2weeks c)2-4weeks d)>4weeks CorrectAnswer-BAnswer-B.<2weeksSteroidsusedafterfirst3to4dayspostinjurydonotaffectwoundhealingasseverlyaswhenusedinimmediatepostoperativeperiod.Delayinuseofsuchdrugsforabout2weekspostinjuryappearstolesserthewoundhealingimpairment.
477.Insurgicalpatientmalnutritionisbest assessedby a)Serumalbumin b)Hblevel c)Midarmcircumference d)Tricpesskinfoldthickness CorrectAnswer-AAnswer-A.SerumalbuminSerumalbuminisclassicmethodtopredictpostoperativecomplications,hospitallengthofstay,morbidityandmortalityandmighttobeassociatetonurtitionalstatusanddiseaseseverity.
478.Abbesflapisusedfor a)Eyelid b)Tongue c)Lip d)Ear CorrectAnswer-CAnswer-C.LipAbbeflap,alsocalledlipswitchflap,isusedforlipreconstruction.
479.InLAHSHALterminologyforcleftlip& cleftpalate,LAHSHALdenotes a)Bilateralcleftpalateonly b)Bilateralcleftliponly c)Bilateralcleftlip&palate d)Nocleft CorrectAnswer-CAnswer-C.Bilateralcleftlip&palateLAHSHALclassificationofcleftlipandpalatewasproposedbyKreins0.Itisadiagrammaticclassificationofcleftcip&palate.Accordingtothisclassification,mouthisdividedintosixparts.LAHSALcodeindicatescompletecleftwithandincompetecleftwithsmalllatter.
480.Postoperativeabscesstreatmentof choice a)Hydration b)IVantibiotics c)Imageguidedaspiration d)Reexploration CorrectAnswer-CAnswer-C.ImageguidedaspirationThediagnosisandtreatmentofintraabdominalabscesseshaveimprovedwiththeadventofimagingtechniquessuchasultrasonographyacdCT.Theseadvanceshavemademinimallyinvassivedrainagetechniquesavailable;suchtechniquescomplementtraditionalsurgicaldrainageforpatientswithabdominalabscesscavities.
481.Maythurnerorcockettsyndrome involves a)Commoniliacarteryobstruction b)Internaliliacarteryobstruction c)Internaliliacveinobstruction d)Leftiliacveincompression CorrectAnswer-DAnswer-D.LeftiliacveincompressionMay-Thurnersyndrome/cockettsyndrome/iliocaval/iliacveincompressionsyndrome.Occursduetocompressionofleftiliacveinbyoverridingrightiliacartery.Itresultsinleftiliofemoraldeepveinthrombosis.
482.Methodofreductionofinguinalhernia a)Kugelmaneuvve b)Taxis c)Macvayproceedure d)Stopa'stechnique CorrectAnswer-BAnswer-B.TaxisTaxis(herniareduction)wasthetreatmentofchoiceforincarciratedhernia.Manualreductionofherniaisknownastaxis.Taxisisanarchaictermusedtodescribeanattemptatreductionofaherniathatisacutelyirreducible(incarcirated).
483.Subclavianstealsyndromeis a)Reversalofbloodflowintheipsilateralvertebralartery b)Reversalofbloodflowinthecontralateralcarotidartery c)Reversalofbloodflowinthecontralateralvertebralartery d)B/Lreversalofbloodflowinvertebralarteries CorrectAnswer-AAnswer-A.ReversalofbloodflowintheipsilateralvertebralarterySubclavianstealsyndromemayoccurifthefirstpartofthesubclavianarteryisoccluded.Armexercisecausessyncopebecauseofreversedflow,inthevertebralarteryleadingtocerebralischemia.Itcanbetreatedbyangioplastyorsurgeryandisrare.Thatistheresultofanipsilateralhemodynamicallysignificantlesionoftheproximalsubclavianartery.
484.MostcommoncauseofacquiredAV fistulais a)Bacterialinfection b)Fungalinfection c)Blunttrauma d)Penetratingtrauma CorrectAnswer-DAnswer-D.Penetratingtrauma"Penetratinginjuriesarethemostcommoncause,butfistulasaresometimesseenafterblunttrauma"?CSDT
485.Bestapproachinthoracictraumais a)Midlinesternotomy b)Parasternalthoracotomy c)Anterolateralthoracotomy d)Posterolateralthoracotomy CorrectAnswer-CAnswer-C.Anterolateralthoracotomyleft'anterolateralthoracotomyisthebestinitialoperativeapproachforunstablepatientsrequiringresuscitationorwhenthelocationoftheintrathoracicinjuryisunclear.
486.Venousairembolismismostcommonin whichpositioninsurgery a)Sitting b)Prone c)Lateral d)Lithotomy CorrectAnswer-AAnswer-A.SittingVenousairembolismisapotentialhazardwhenevertheoperativesiteisabovethelevelofpatientsheart.The'sitting'positionanditsmodification"beachchair"positionsareassociatedwithagreaterincidenceofvenousairembolism
487.Fatalexsanguinationsoccursmostlyin a)Closedfractureoffemurshaft b)Openfractureoffemur&tibia c)Partialtransactionofartery d)Completeresectiontransactionofartery CorrectAnswer-CAnswer-C.Partialtransactionofartery"Bleedingismoreoftenexsanguinatingaftersharpinjuryandpartialvesseltransection".
488.Premalignantlesionforcarcinoma rectumis a)Familialpolyposis b)FAP c)Juvenilepolyp d)Adenomatouspolyp CorrectAnswer-AAnswer-A.FamilialpolyposisPrecancerouslesionsforCArectum 1. Villouspapilloma2. Adenomas3. Familialpolyposis
489.Mostcommonsiteforanalfissureis a)3O'clock b)6O'clock c)2O'clock d)10O'clock CorrectAnswer-BAnswer-B.6O'clockThevestmajorityofanalfissuresoccurinposteriormidline.FissureinAno(orAnalfissure)Mostcommonsiteismid-lineposteriolyMCsymptomis-4painassociatedwithdefecationFissurestartsproximallyatthedentateline
490.Alvaradoscoreconsistof a)Leucopenia b)Anorexia c)Diarrhea d)Periumbilicalpain CorrectAnswer-BAnswer-B.AnorexiaThemostwidelyusedistheAlvaradoscore.Ascoreof7ormoreisstronglypredictiveofacuteappendicitis. AlvaradoscoreSymptoms Score Migrationofpain 1 Anorexia 1 Nausea&vomiting 1 Signs Tendernessintherightlowerquadrant 2 Reboundtenderness 1 Elevatedtemperature 1 Laboratory Leucocytosis 2 Ashiftofwhitebloodcellcounttotheleft 1 10 Total
491.Mostcommonageforintussusception is a)0-6Months b)6Months-3yrs c)3-5Yrs d)>5yrs CorrectAnswer-BAnswer-B.6Months-3yrsIntussusceptionisthetelescopingofoneportionoftheintestineintotheother.Itisthemostcommoncauseofintestinalobstructioninearlychildhood(3monthsto6years)
492.Retrocardiaclucencywithairfluidlevel isseenin a)Hiatushernia b)Distalendesophagealobstruction c)Eventrationofdiaphragm d)None CorrectAnswer-AAnswer-A.HiatusherniaHiatusherniashowsretrocardiclucencywithairoranair-fluidlevelabovethediaphragm.
493.Mostcommonsiteforcarcinoidtumor is a)Esophagus b)Lung c)Appendix d)Ileum CorrectAnswer-DAnswer-DHistorically,themostcommonsiteofgastrointestinal(GI)carcinoidtumorswastheappendix.Currently,however,themostcommonsiteofcarcinoidsintheGItractisthesmallintestine(30%),followedbytherectum(19.6%).Inmoststudies,theappendixisonlythethirdmostcommonsiteofGIcarcinoids,andinsomestudies,itisthefourthmostcommon.
494.Mostcommondifferentialdiagnosisfor appendicitisinchildrenis a)Gastroenteritis b)Mesentriclymphadenopathy c)Intussusception d)Meckel'sdiverticulitis CorrectAnswer-BAnswer-B.MesentriclymphadenopathyDifferentialdiagnosisofappendicitisinchildren- 1. Acutegastroenteritis2. Intussusception3. Meckel'sdiverticulitis4. Mesentriclymphadenitis(MC)5. Inflammatoryboweldisease6. Constipation7. Functionalpain
495.AbdominalsurgeryunderLA,patient suddenlyfeltpaindueto a)Liver b)Gut c)Parietalperitoneum d)Visceralperitoneum CorrectAnswer-CAnswer-C.ParietalperitoneumEmbryologicallyparietalperitoneumisderivedfromthesomatopleurallayerofthelateralplatemesoderm.Itsbloodsupplyandnervesupplyarethesameasthoseoftheoverlyingbodywall.Becauseofthesomaticinnervation,itispainsensitive."--BDCAnatomy
496.A55yearmalehashistoryofdysphagia withvomitingofundigestedfoodthroughouttheday,weightloss,emaciated&dehydrated.Nomasspalpableperabdomen.Themodalityoftreatmentis a)IVtotalparenteralnutrition b)Endoscopicdilation c)IVnormalsaline d)pHmonitoring CorrectAnswer-BAnswer-B.EndoscopicdilationThesymptom&signcomplexindicatesdiagnosisofAchalasia.Oneofthetreatmentmodalitiesforachalasiacardiaisendoscopicdilation.
497.Hosepipeappearanceofintestineisa featureof a)Crohnsdisease b)Malabsorptionsyndrome c)Ulcerativecolitis d)Hirsprungdisease CorrectAnswer-AAnswer-A.CrohnsdiseaseCrohn'sdiseasehasahosepipeappearance.
498.Inoldageforrectalprolapsepalliative surgeryinapatientunfitforsurgeryis a)Delorme'sprocedure b)Well'sprocedure c)Thiersch'soperation d)Lowanteriorresection CorrectAnswer-CAnswer-C.Thiersch'soperationTighteningtheanuswithavarietyofprostheticmaterials(analencirclement)-Analencirclementproceduresgenerallyhavebeenabandoned.Analencirlementhaslimitedapplicationandisreservedbymostsurgeonsforpatientsofthehighestsurgicalriskorlimitedlifeexpectancybecauseitcanbedoneunderlocalanesthesia.TheoriginalThierschprocedureinvolvedplacingasilverwirearoundtheexternalsphincterwithintheishiorectalfat.Nowsyntheticmeshorsiliconetubesareusedinsteadofwire.Thesafetyofcurrentanesthetictechniquesandthelowmorbidityandrelativefunctionalsuccessofperinealproctectomyhavemadeanalencirclement,forthemostpart,aprocedureofthepast.
499.40yearoldmalecomplaintsofGERD,on endoscopyshowsdysplasia.Treatment a)Fundoplication b)Esophagealresection c)PPI d)Dietmodification CorrectAnswer-BAnswer-B.EsophagealresectionIfseveredysplasiaorintranuralcarcinomaisfoundonmucosalbiopsy,anesophagealresectionshouldbedone.
500.Renalstoneswhicharelaminatedand irregularinoutlineare a)Uricacid b)Calciumoxalate c)Struvite d)Cystine CorrectAnswer-BAnswer-B.CalciumoxalateCalciumoxalatestones-Usuallysingle,hard(akaMulberrystone)-Darkcoloredd/tstainingwithalteredblood.Spiky.OnsectionWavyconcentriclaminae.Theremaybesecondaryphosphatedepositonsurface.Highcalciumcontent.
501.Inacaseofperforationperitonitis, emergencylapratomy,2ndpostoperativedaydevelopsoliguria.Diagnosis a)SevereUTI b)Fluidretention c)Dehydration d)Catheterobstruction CorrectAnswer-CAnswer-C.DehydrationOliguriamayreflectinadequaterenalarteryperfusionduetohypotension,hypovolemiaorlowQT.Itcanalsobeasignofintrinsicrenaldysfunction.
502.Etiologyofbloodydischargefrom nipple a)Ductpapiloma b)Breastabscess c)Fibroadenoma d)Cyst CorrectAnswer-AAnswer-A.Ductpapiloma Table1:CausesofnippledischargePhysiologicDuctabnormalitiesIntraductalpapillomaDuctectasiaPeriductalmastitisCarcinomaGalactorrheaHyperprolactinemiaHypothyroidismMedications:oralcontraceptives,cimetidine,verapamil,phenothiazine,metoclopramide,alpha-methyldopaConditionsthatmaymimcnippledischargeEczemawithdrainagePaget'sdiseaseofthebreastNippleadenoma
503.Carcinomabreastisleastseenin a)Superiorouterquadrant b)Inferiorouterquadrant c)Subareolar d)Lowerinnerquadrant CorrectAnswer-DAnswer-D.LowerinnerquadrantUpperinner-+12-15%Upperouter=50%Lowerinner3-5%Lowerouter6-10%Central/areolar20%[RefLove&Bailey25th/ep.840,S.Das7thiep.607]
504.Intraoperativesentinellymphnode detectioninaxillaisdonebyusing a)Mammography b)Isosulfanbluedye c)MRI d)CT CorrectAnswer-BAnswer-B.IsosulfanbluedyeLymphaticmappingisperformedbyusingisosulfanbluedye,technetium-labelledsulfurcolloidalbuminoracombinationofboth.
505.Sentinellymphnodebiopsyin carcinomabreastisdoneif- a)LNpalpable b)Breastmassbutnolymphnodepalpable c)Breastlumpwithpalpableaxillarynode d)MetastaticCAbreast CorrectAnswer-BAnswer-B.BreastmassbutnolymphnodepalpableAxillarynodesifclinicallypalpableareremovedbysurgicaldissection.
506.Indicationforsentinelnodebiopsyis a)Nonpalpableaxillarylymphnode b)Palpableaxillarylymphnode c)Mass>5cm d)Metastasis CorrectAnswer-AAnswer-A.NonpalpableaxillarylymphnodeThiswillbeofgreatsignificanceinearlybreastcarcinomaswhereinlymphnodesarenotclinicallypalpablenordetectedbyinvestigationssuchasultrasound/CTscanoftheaxilla..Indication:Earlybreastcancerwith(T1orT2No)clinicallynodenegativeaxilla.
507.Breastconservationsurgeryis contraindicatedinallexcept- a)Tumor>4cm b)Multicentrictumor c)AxillaryLNinvolvement d)Diffusemicrocalcifications CorrectAnswer-DAnswer-D Breastconservativesurgery IndicationsLump<4cm Contraindications Clinicallynegativeaxillary Tumour>4cm nodes Plositiveaxillarynodes>N1 Mammographicallydetected Tumourmarginisnotfreeoftumourlesion afterbreastconservativesurgery Well-differentiatedtumour needsMRM withlowSphase Poorlydifferentiatedtumour Adequatesizedbreastto Multicentrictumour allowproperRTtobreast Earlierbreastirradiation Breastofadequatesizeand Tumour/breastsizeratioismorevolume (centeraltumour) Feasibilityofaxillary Tumourbeneaththenipple dissectionandradiotherapy Extensiveintraductalcarcinoma tointactbreast
508.Thefollowingaresuitableforsimple mastectomyexcept- a)Pagetsdisease b)Fibroadenoma c)Cystosarcomaphyllodes d)None CorrectAnswer-BAnswer-B.FibroadenomaIndicationsforsimplemastectomyA)Withoutanaxillaryprocedure 1. Risk-reducingmastectomy2. Localrecurrenceinapreviouslytreatedbreastcanceriii)Malignant phyllodestumor(cystosarcomaphyllodes)b)Withconcomitantaxillaryprocedure 3. Locallyadvancedbreastcancer(includingpagetdisease)4. Multifocalbreastcancer5. Extensiveductalcarcinomainsitu(DCIC)6. Patientisunsuitableforbreast-conservativeapproach
509.Whichofthefollowingisusedinthe treatmentofwelldifferentiatedthyroidcarcinoma a)I131 b)99mTc c)32p d)MIBG CorrectAnswer-AAnswer-A.I131I131istheradioisotopeofchoiceforradiotherapyofthyroidcarcinomaandhyperparathyroidism.
510.Treatmentformalignantmelanomais a)Wideexcision b)Radiotherapy c)Excision d)Chemotherapy CorrectAnswer-AAnswer-A.WideexcisionManagementofMalignantMelanomaWidelocalexcisionoftheprimarytumoristhemanagementofchoice.Therecommendedmarginofresectiondependsonthethicknessofthetumor.
511.Mouressignisseenin a)Carcinoma b)Appendicitis c)Varicosevein d)Pancreatitis CorrectAnswer-AAnswer-A.Carcinoma"Innormalpersons,aclickisfeltwhenlarynxismovedfromsidetosideoververtebralcolumn,thisiscalledlaryngealclick(postcricoidcrepitus)Itisabsentinpostcricoidcarcinoma".--Moure'ssign
512.Acuteorchitisallareseenexcept a)Increasedlocaltemprature b)Decreasedbloodflow c)Etythematousscrotum d)RaisedTLC CorrectAnswer-BAnswer-B.DecreasedbloodflowUSGshowsincreasedbloodflowinacutestage.Ischemicorchitismaysetin,inlatestagesresultinginreducedbloodflowonUSG.
513.Allaretrueaboutcarcinomapenis except a)Mostcommontypeisverrocous b)Spreadsbybloodbornemetastasis c)Leadstoerosionofartery d)Slowlyprogressive CorrectAnswer-AAnswer-A.MostcommontypeisverrocousMCtype?SCCETIOLOGY-Premalignantlesions-Genitalwarts-Bushke-Lowensteintumourisagiantpenilecondyloma(verrucouscarcinomaofpenis)ErythroplasiaofQueyratorPaget'sdiseaseofpenis-precancerouslesionSPREAD-BloodspreadisrareDeathmayoccurduetoerosionsoffemoralvesselsbyiguinalLN.Slowlyprogressive
514.Intesticulartorsion,surgerywithinhow muchtimecansaveviabilityoftestis a)6hr b)12hr c)24hr d)1week CorrectAnswer-AAnswer-A.6hr"Morethan80%testescanbesalvagedifsurgeryisperformedwithin6hours".[RefSchwartz9th/ep.1469]
515.Investigationusingdyetofindoutstone insalivarygland a)Sialography b)Mammography c)MRangiography d)USG CorrectAnswer-AAnswer-A.SialographySalivaryductstones&stricturesChronicsialadenitisTumorsofsalivaryglands
516.ERCPisindicatedforthefollowing except a)DistalCBDtumor b)Hepaticportatumor c)Proximalcholangiocarcinoma d)Gallstonepancreatitis CorrectAnswer-CAnswer-C.ProximalcholangiocarcinomaERCPisnottechnicallypossibleinproximalbiliaryobstructions.
517.Alagillesyndromeis a)Bileductpaucity b)IHBRdilation c)PBC d)PSC CorrectAnswer-AAnswer-A.Bileductpaucityyndromicpaucityofinterlobularbileducts(Alagillesyndrome)isthemostcommonformoffamilialintrahepaticcholestasis.Chroniccholestasisaffects95%ofpatients.Peripheralpulmonicstenosisisobservedinapproximately90%.Vertebralarchdefectsareseen(e.g.,butterflyvertebrae,hemivertebrae,andadecreaseintheinterpediculardistance).Ophthalmologicexaminationmayrevealposteriorembryotoxon,retinalpigmentation,andirisstrands.
518.Hemorrhagicpancreatitis,bluish discolorationofflank a)Greyturnersign b)Cullensign c)Trosseuesign d)None CorrectAnswer-AAnswer-A.GreyturnersignInacutepancreatitis: 1. Cullen'ssign:Ecchymosis(bluish-purplecolor)aroundumbilicus (periumbilicalarea) 2. Greyturner'ssign:Ecchymosis(bluish-purplecolor)inflank
519.Complicationofchronicpancreatitis includeallexcept- a)Renalarterythrombosis b)Pseudocyst c)Splenicveinthrombosis d)Fistulae CorrectAnswer-AAnswer-A.RenalarterythrombosisComplications-ObstructivejaundiceCarcinomaofpancreasPseudocystsPancreaticductleakwithascitesorfistulaThrombosisofsplenicveinAbscessPerforation
520.Duringfunctionalendoscopicsinus surgerythepositionofpatientis a)Trendelenberg b)Lateral c)Reversetrendelenberg d)Lithotomy CorrectAnswer-CAnswer-C.ReversetrendelenbergUsingthereversetrendelenburgpositionduringfunctionalendoscopicsinussurgery(FESS)issafe,simple,andcost-freemethodthathasbeenfoundtoreduceintraoperativebloodloss.
521.Head&faceburnininfantis a)15% b)18% c)12% d)32% CorrectAnswer-BAnswer-B.18%"Infantshave21%oftheTBSAintheheadandneck"--SabistonChildrenhavearelativelylargerportionofthebodysurfaceareaintheheadandneckwhichiscompensatedforbyarelativelysmallersurfaceareainthelowerextremities.Infantshave21%ofTBSAintheheadandneckand13%ineachleg.

522.Plasmaexpandersareusedin a)Endotoxicshock b)Neurogenicshock c)Vasovagalshock d)Anaphylacticshock CorrectAnswer-AAnswer-A.EndotoxicshockUsesofplasmaexpanders4tocorrecthypovolemia,e.g.inburns,hypovolemicandendotoxicshock,severetrauma.Contraindications-Severeanaemia,cardiacfailure,pulmonaryedema,renalinsufficiency.
523.Whiplashinjuryistearofwhich ligament a)Ligamentaflava b)Ant.longitudinalligament c)Post.longitudinalligament d)Supraspinalligament CorrectAnswer-BAnswer-B.Ant.longitudinalligamentHyperextensiontheorydescribeswhiplashinjury.Hyperextensionmostcommonlyresultsinanteriorcervicalcolumninjuriesintheformofanteriorlongitudinalligamentandintervertebraldiscruptures.
524.Neurosurgeryisindicatedforallexcept a)SDH b)EDH c)Intracerebralbleed d)Diffuseaxonalinjury CorrectAnswer-DAnswer-D.DiffuseaxonalinjuryDiffuseaxonalinjurydoesnothaveanyspecifictreatmentSymptomatictreatmentandstabilizationofpatientisrequired.Subduralhematoma,(SDH),epiduralhematoma(EDH)andintracerebralhematoma(parenchymalhemorrhage)mayrequiresurgery
525.Youngmalewithhistoryoftrauma havingleftsidedtestisswollen&erythematous.Othersidenormaldiagnosis a)Torsion b)Carcinoma c)Hematoma d)Hernia CorrectAnswer-CAnswer-C.HematomaBlunttraumatotestiscancausehematoma.Thereisassociatedtenderness,swellingandecchymosisofthehemiscrotum.
526.Centralstellatescarisseenin a)Focalnodularhyperplasia b)Chronicregenerativehyperplasia c)Hepatoblastoma d)None CorrectAnswer-AAnswer-A.FocalnodularhyperplasiaFocalnodularhyperplasiaisararetumor-likeconditionpredominantlyfoundinwomenduringtothirdtothefifthdecadeoflife.Mostcommonlyitisincidentallydiscoveredasasymptomaticmass.Thepathognomonicmicroscopicfeature.Acentralstellatescarseeninliverimagingstudiesisahighlycharacteristicfeatureoffocalnodularhyperplasia.ItisalsoseeninRenalcellcarcinoma(RCC).
527..Zplastyidealangle a)90? b)45? c)60? d)75? CorrectAnswer-CAnswer-C.60?The60degreeZ-plasty(ie,classicZ-plasty)ismostcommonlyusedbecauseitprovidestheoptimalbalancebetweenlengtheningandeaseofclosure.
528.Lymphedemaprecoxallaretrueexcept a)U/L b)Morecommoninmen c)Affectsthelegs d)2-35yrsofage CorrectAnswer-BAnswer-B.MorecommoninmenPrimarylymphedemawithageofonsetb/wages1yearand35years.MCformsofprimarylymphedema.Female:Male-10:1Swellinginvolvesfootandcalf.Usuallyunilateral
529.RTAwithmultiplefracturesinitial treatmentwouldbe- a)Managementofshock b)Splintingoflimbs c)Airwaymanagement d)Cervicalspineprotection CorrectAnswer-CAnswer-C.AirwaymanagementManagementoftraumabeginswithprimarysurvey.TheATLS(AdvancedTraumaLifeSupport)definesprimarysurveyasassessmentofthe`A,B,C'i.e.,Airwaywithcervicalspineprotection,Breathingandcirculation.
530.Withblunttraumaalloverbodythe amountofN2&nitrogenendproductslost/day a)35gm b)45gm c)55gm d)65gm CorrectAnswer-AAnswer-A.35gmAfterinjury,theinitialsystemicproteolysis,mediatedprimarilybyglucocorticoidesincreasesurinarynitrogenexcretiontolevelsinexcessof30gm/day.
531.InvertedChampagnebottleappearance isseenin a)Varicoseveins b)DVT c)Lipodermatosclerosis d)Venousulceration CorrectAnswer-CAnswer-C.LipodermatosclerosisThemostcommonlyrecognizedformoflipodermatosclerosis(LDS),chronicLDSpresencewithindurationandhyperpigmentationoftheskininvolvingtheoneorbothofthelowerlegsinacharacteristic"invertedchampagnebottle"appearance.Associatedwithvenousinsufficiency,LDSismostcommoninmiddle-agedwomen.
532.10cmtumoronanteriorsurfaceofthigh, whatisdonetoknowtodiagnosis a)Incisionbiopsy b)Excisionbiopsy c)FNAC d)USG CorrectAnswer-AAnswer-A.IncisionbiopsyIncisionalbiopsyisindicatedwhentissuesamplesarenotobtainedbyFNACorcoreneedlebiopsyasindeeptumorsandforsuperficialsofttissuetumors>3cm.Excisionalbiopsyisindicatedforeasilyaccesibleextremityortruncallesions<3cm.
533.Flashburn,tender,red&painful,which ofthefollowingtypeistheburn- a)Scaldedburn b)Firstdegreeburn c)Seconddegreeburn d)Fourthdegreeburn CorrectAnswer-BAnswer-B.FirstdegreeburnFirstdegreeburn(alsok/aSuperficialorEpidermalburn)TheseburnsinvolveonlytheepidermisTheydonotblister,AreerythematousbecauseofdermalvasodilationBlanchtotouch(i.e.showcapillaryrefilling)ArequitepainfulHealwithoutscarringin5to10days.Theyresultinpainandreddeningoftheepidermis(outerlayeroftheskin).Theclinicalfearuresareblisteringand/orlossoftheepidermis.Theunderlyingdermisispinkandmoist.Thecapillaryreturniscleariyvisiblewhenblanched
534.Inafemaleabdominalintestinal perforationoperatedhasserousdischargeon5thdaywithwoundgap.Whatisyourdiagnosis a)Wounddehiscence b)Enterocutaneousfistula c)Scroma d)Peritonitis CorrectAnswer-AAnswer-A.WounddehiscenceDehisencemostoftendevelops7to10dayspostoperativelybutmayoccuranytimeaftersurgery,from1tomorethan20days."-Sabiston
535.Peritonitis a)Neutrophils>250mm3 b)WBC<100/m1 c)Ascitislactatelevel<25mg/dl d)AscitisfluidpH>735 CorrectAnswer-AAnswer-A.Neutrophils>250mm3Morethan250neutrophills/cummofascitisfluidsuggestanacuteinflammatoryprocess,themostcommonofwhichisspontaneousbacterialperitonitis.Anasciteslactatelevelofmorethan25mg/diwasfoundtobe100%sensitiveandspecificinpredictingactivespontaneousbacterialperitonitis.
536.Treatmentofchoiceinanorectal carcinoma a)Chemoradiation b)APRCombinedsurgeryandradiotherapy c)Chemotherapyalone d)All CorrectAnswer-BAnswer-BAbdominal-perinealresectionwithcolostomywasthepreferredsurgicalprocedureformostmajorcancersoftheanalcanal,
537.Rectalprolapsesurgeryis a)Rectalmucosalstapling b)Mucosalresection c)Placation/wiring d)Rectopexy CorrectAnswer-DAnswer-D.RectopexyAbdominalapproach 1. Reductionofperinealherniaandclosureofced-de-sac (Moschowitz'soperation) 2. Fixationofrectumeitherwithaprostheticsling(Ripsteinandwells rectopexy)orbySutureRectopexy. 3. Resectionofredundantsigmoidcolon?rectalfixation(Resection rectopexy).
538.Crohn'sdisease a)Continousinvolvement b)Sinus&fistula c)Mesentericlymphadenitis d)Studulcer CorrectAnswer-BAnswer-B.Sinus&fistulaClinicalfeatures-Intermittentmilddiarrhea,fever,abdominalpain(MC)Rightlowerquadrantmass,weightloss,anemiaSometimesmimicsappendicitisorbowelperforationAnalcomplaints(fissure,fistula,abscess)?frequentFat/vitaminmalabsorptionpresent
539.Stepladderpatternofgasshadowis seenin a)Duodenalobstruction b)Intestinalobstruction c)Gastricoutletobstruction d)Sigmoidvolvulus CorrectAnswer-BAnswer-B.IntestinalobstructionStepladdersignrepresentstheappearanceofgas-fluiddistendedsmallbowelloopsthatappeartobestackedontopofeachother,typicallyobservedonerectabdominalradiographsinthesettingofsmallbowelobstruction.
540.Whatisnotseeninshortbowel syndrome a)Hypergastrinemia&highgastricsecretionisseen b)Diarrhea,dehydrationandmalnutrition c)Hirsutism d)ChronicTPNdependence CorrectAnswer-CAnswer-C.HirsutismResectionofjejunumisbettertoleratedthanresectionofileum,asthecapacityforbilesaltandvitaminB12absorptionisspecifictotheileumMalabsorptionaftermassivesmallbowelresectionisexacerbatedbyacharacteristichypergastrinemiaassociatedgastricacidhypersecretionthatpersistsfor1to2yearspostoperativelyShort-bowelsyndromeisadisorderclinicallydefinedbymalabsorption,diarrhea,steatorrhea,fluidandelectrolytedisturbances,andmalnutrition.
541.Ogilvie'ssyndromemostcommonly involves a)Stomach b)Colon c)Gallbladder d)Smallintestine CorrectAnswer-BAnswer-B.ColonOgilviesyndrome,oracutecolonicpseudo-obstruction(ACPO),isaclinicaldisorderwiththesigns,symptoms,andradiographicappearanceofacutelargebowelobstructionwithnoevidenceoftheactualphysicalcauseoftheobstruction.Thecolonmaybecomemassivelydilated;ifnotdecompressed,thepatientrisksperforation,peritonitis,anddeath.Pseudo-obstructionmostcommonlyoccursinhospitalizedpatientsandisassociatedwiththeuseofnarcotics,bedrest,andco-morbiddisease.Thisconditiondescribesanobstruction,usuallyofthecolon,thatoccursintheabsenceofamechanicalcauseoracuteintra-abdominaldisease.Abdominalradiographsshowevidenceofcolonicobstruction,withmarkedcecaldistensionbeingacommonfeature.
542.Spigelianherniais a)Throughlineaalba b)Throughlateralborderofrectusabdominis c)Throughmedialwallofiinguinalcanal d)Throughlateralwallofinguinalcanal CorrectAnswer-BAnswer-B.ThroughlateralborderofrectusabdominisSpigelianherniascanoccuranywherealongthelengthofthespigelianlineorzone--anaponeuroticbandofvariablewidthatthelateralborderoftherectusabdominis.Themostfrequentlocationoftheserarehemiasisatorslightlyabovethelevelofthearcuateline.Spigelianherniaoccursthroughthelinearsemilunariswhichcorrespondstothelateralmarginoftherectusabdominis.Aspigelianherniaoccursthroughthespigelianfascia,whichiscomposedoftheaponeuroticlayerbetweentherectusmusclemediallyandthesemilunarlinelaterally.
543.24dayneonatewithprojectilevomiting& failuretogainweight.whatisthediagnosis a)CHPS b)NEC c)Duodenalatresia d)Hirschsprung'sdisease CorrectAnswer-AAnswer-A.CHPSProjectilevomitingin4thweekisquitesuggestiveofCHPS.Induodenalatresiathevomiting(usuallybilious)isrightfromthe1stdayoflife.NECandHirschsprung'sdiseasehavedifferentclinicalpresentation.
544.NottrueaboutBarrett'sesophagus a)Metaplasiaofcells b)PredisposestoSCC c)Precancerouscondition d)Intestinaltypeisthemostcommontype CorrectAnswer-BAnswer-B.PredisposestoSCCBarrett'sesophagusispremalignantconditionforadenocarcinmaesophagusandnotSCC.
545.Gastrotomyis a)Openthestomachclosedaftertubeinsertion b)Openingthestomach c)Resectingtheterminalpartofstomach d)Resectingtheproximalpartofstomach CorrectAnswer-BAnswer-B.OpeningthestomachGastrostomyreferstoasurgicalopeningintothestomach.;creationofanartificialexternalopeningintothestomachfornutritionalsupportorgastrointestinalcompression.Typicallythiswouldincludeanincisioninthepatient'sepigastriumaspartofaformaloperation.
546.Parrotbeakappearanceisseenin a)Volvulus b)Intussuption c)Rectalatresia d)CAcolon CorrectAnswer-AAnswer-A.VolvulusBarium/gastrograffinenema-demonstratesthepointofobstruction,pathognomonic"Birdbeckdeformity"orparrotbeakdeformity.Contrastenemaiscontraindicatedifgangreneissuspected."Parrotbeaked"clawingofneilsisalsoseeninchroniccocaineabuse.
547.Raspberrytumorisseenin a)Umbilicalfistula b)Meckel'sdiverticulum c)Umbilicaladenoma d)Umbilicalgranuloma CorrectAnswer-CAnswer-C.UmbilicaladenomaAnumbilicaladenomaisalsoknownas"Raspberrytumor".Itisduetoapartiallyunobliteratedvitellointestinalduct.Itisseenininfants.Theprolapsingmucosagivestheraspberryappearanceandbleedsontouch.Treatment:Aligatureistiedarounditsbase,anditfallsoffafterafewdays.Recurrenceistreatedwithsurgery.
548.Aftermastectomy,breastreconstruction isdoneby a)Deltopectoral b)latissimusdorsi c)Serratusanterior d)Trapezius CorrectAnswer-BAnswer-B.latissimusdorsiIftheskinatthemastectomysiteispoor(e.g.followingradiotherapy)orifalargervolumeoftissueisrequired,amusculocutaneousflapcanbeconstructedeitherfromthelatissimusdorsimuscle(anLDflap)orusingthetransversusabdominismuscle(aTRAMflapas).Thelattergivesanexcellentcosmeticresultinexperiencedhandsbutisalengthyprocedureandrequirescarefulpatientselection.
549.PeaudeorangeappearanceofCabreast, whatisthestage a)T4b b)T4a c)T3a d)T3b CorrectAnswer-AAnswer-A.T4bStageI:T1N0M0StageIla:T0N1M0;T1N1M0;T2N0M0.StageIlb:T2N1M0;T3N0M0StageIlla:T0N2M0;T1N2M0;T2N2M0;T3N1M0;T3N2M0StageIllb:T4N0M0;T4N1M0;T4N2M0StageIllc:AnyTN3M0StageIV:AnyT,anyN,MEarlybreastcancer___StageIand11J1N1,1-2N1;13NOLocallyadvancedbreastcancer(LABC)--Stage111AIMetastaticbreastcancer--StageIVPrimarytumor(T)TX-Primarytumorcannotbeassessed T0-Noevidenceofprimarytumor Tis-Carcinomainsitu Tis(DCIS)-Ductalcarcinomainsitu
Tis(Paget)-PagetdiseaseofthenippleNOTassociatedwithinvasivecarcinomaand/orcarcinomainsitu(DCIS)intheunderlyingbreastparenchyma.CarcinomasinthebreastparenchymaassociatedwithPagetdiseasearecategorizedonthebasisofthesizeandcharacteristicsoftheparenchymaldisease,althoughthepresenceofPagetdiseaseshouldstillbenoted T1-Tumor20mmingreatestdimension T1mi-Tumor1mmingreatestdimension T1a-Tumor>1mmbut5mmingreatestdimension(roundanymeasurement>1.0-1.9mmto2mm) T1b-Tumor>5mmbut10mmingreatestdimension T1c-Tumor>10mmbut20mmingreatestdimension T2-Tumor>20mmbut50mmingreatestdimension T3-Tumor>50mmingreatestdimension T4-Tumorofanysizewithdirectextensiontothechestwalland/ortotheskin(ulcerationorskinnodules),notincludinginvasionofdermisalone T4a-Extensiontochestwall,notincludingonlypectoralismuscleadherence/invasion T4b-Ulcerationand/oripsilateralsatellitenodulesand/oredema(includingpeaud'orange)oftheskin,whichdonotmeetthecriteriaforinflammatorycarcinoma T4c-BothT4aandT4b T4d-Inflammatorycarcinoma

550.Notacomponentoftripletestin detectionofCabreast- a)Breastselfexamination b)USG/mammography c)FNAC/trucutbiopsy d)Clinicalexamination CorrectAnswer-AAnswer-A.BreastselfexaminationTripleassessmentincludesexaminationbyaclinician.Selfexaminationisnotapartoftripleassessment.BaileyandLovewrites--"Inanypatientwhopresentswithabreastlumporothersymptomssuspiciousofcarcinoma,thediagnosisshouldbemadebyacombinationofclinicalassessment,radiologicalimagingandatissuesampletakenforeithercytologicalorhistologicalanalysis,thesocalledtripleassessment.Thepositivepredictivevalue(PPV)ofthiscombinationshouldexceed99.9%."
551.Treatmentforhydroureter a)Antibioticprophylaxisalone b)Immediateureterolithotomy c)Endoscopicureteralstenting d)Urinaryalkalization CorrectAnswer-CAnswer-C.EndoscopicureteralstentingHydroureterismainlycausedbyintrinsicandextrinsicobstructionofureter.Causesare-Calculi(ureteric/VVJ)-intrinsicstrictureandRetroperitonealfibrosis-extrinsic.
552.Diversionofurineisbestdoneat a)Ileum b)Jejunum c)Caecum d)Colon CorrectAnswer-AAnswer-A.IleumIlealsegmentisthebestforurinarydiversionaftercystectomy.Stillbettermethodis"Continentcutaneousdiversion"method.Butthebestmethodis"Orthotopicneobladders".
553.Ureterosigmoistomy a)Hyperchloremicwithhypokalemicacidosis b)Hyperkalemia c)Metabolicalkalosis d)Hyponatremia CorrectAnswer-AAnswer-A.HyperchloremicwithhypokalemicacidosisThereishyperchloremicmetabolicacidosiswithhypokalemia.
554.Injurytopeniswhichofthefollowing preventsextravasationofblood? a)Bucksfascia b)Fasciaofcamper c)Fasciatransversalis d)None CorrectAnswer-AAnswer-A.BucksfasciaSuperficialtotunicaalbugineathereisBuck'sfacia(deeplayerofsuperficialfasciaofpenis),aprolongationofcolle'sfascia(membranouslayerofsuperficialfasciaofperineum).IfBuck'sfaciaremainsintact,hematomaisrestrictedtopenileshaftonlycausingegg-plantdeformity.IfBuck'sfasciaisdisrupted,hematomacanextendtoscrotum,perineumandsuprapubicregions.
555.Phimosisisassociatedwith a)Paraphimosis b)Meatalstenosis c)Balanoposthitis d)Hypospadias CorrectAnswer-CAnswer-C.BalanoposthitisPhismosiscausedduetochronicinfectionCongenitalAcquired-trauma,PenisCaandBalanitisPathologicalphimosis(asopposedtothenaturalnon-retractabilityoftheforeskininchildhood)israreandthecausesarevaried.Somecasesmayarisefrombalanitis(inflammationoftheglanspenis).Phimosismayoccurafterothertypesofchronicinflammation(suchasbalanoposthitis),repeatedcatheterization,orforcibleforeskinretraction.Phimosismayalsoariseinuntreateddiabetics
556.Circumcisioniscontraindicatedin a)Balanitis b)Hypospadias c)Paraphimosis d)Exostrophyofbladder CorrectAnswer-BAnswer-B.HypospadiasCircumcisionisnotdoneinpatientswithhypospadiasastheprepucecanlaterbeusedinsurgicalrepair.Circucisionismostlydoneforculturalreasons.Themedicalindicationsforcircumcisionare:Phimosis&ParaphimosisRecurrentbalanoposthitis(i.e.inflammationoftheforeskin)Recurrenturinarytractinfection
557.Whichisnottrueaboutcancerof tongue a)Adenocarcinomamostcommon b)Lateralsurfaceinvolved c)Deepcervicallymphnodesnotinvolved d)Tobaccoisthecause CorrectAnswer-AAnswer-A.AdenocarcinomamostcommonSCCisthemostcommontypeofmalignancy,butleiomyosarcomasandrhabdomyosarcomasarealsoencountered(rarely).Tumorsonthetonguemayoccuronanysurfacebutaremostcommonlyseenonthelateralandventralsurfaces.Theregionallymphaticsoftheoralcavityaretothesubmandibularspaceandtheuppercervicallymphnodes.Riskfactors-tobaccoandalcohol.
558.Kasaioperation a)Biliaryatresia b)Choledochalcyst c)Hepatocellularcarcinoma d)Primarybiliarycirrhosis CorrectAnswer-AAnswer-A.BiliaryatresiaKasaioperationisalsoknownashepatoportoenterostomy.BiliaryatresiaiscurrentlyMCindicationforpediatriclivertransplantation.
559.Falseabouthepaticadenoma a)Benignlesion b)OCPuse c)Olderfemales d)Coldonisotopescan CorrectAnswer-CAnswer-C.OlderfemalesHepaticadenomasarebenignsolidneoplasmsofliver.MCseeninyoungerfemales(20-40yearsofage)UsuallysolitaryRiskfactor-Prior/currentuseofestrogens(OCP)
560.Bismuthclassificationinwhichclass, hepaticductconfluenceisinvolved a)TypeI b)TypeII c)TypeIII d)TypeIV CorrectAnswer-BAnswer-B.TypeIIBismuth-CorletteclassificationisusedtoclassifycholangiocarcinomaTypeI-Commonhepaticductinvolvement.TypeII-CHD+bifurcation/confluenceofhepaticducts.TypeIII-a)Extensiontorightsecondaryintrahepaticduct.b)Extensiontoleftsecondaryintrahepaticduct.TypeIV-Involvingbothrightandleftsecondaryintrahepaticducts.
561.Parathyroidglandsareremovedby surgery,forrecurrenceinvestigationofchoice a)Technetiumscan b)SPECT c)MRI d)Neckultrasound CorrectAnswer-AAnswer-A.TechnetiumscanPreoperativelocalisationtestsforparathyroidglandsare 1. Tc99mlabeledsestamibiscan(>80%sensitive)2. Singlepositronemissioncomputedtomography(SPECT).3. CTandMRIscan.4. Neckultrasound.
562.Whichisnotacontraindicationfor pancreaticoduo-denectomy a)Metastasis b)Portalveininvolvement c)StageIIICAheadofpancrease d)Invasionofsuperiormesentricvein CorrectAnswer-BAnswer-B.PortalveininvolvementTumor(T)TX-Primarycan'tbeasscessed.To-Noevidenceofprimarytumor.T1-Cainsitu.T1-Limitedtopancreasand2cm.T2-Limitedtopancreasand>2cm.T3-Extendsbeyondpancreasbutnoinvolvementofceliacaxisorsuperiormesentricartery.T4-InvolvesceliacaxisorSup-mesentericartery(unresectableprimary).Regionallymphnodes(N)NX-RegionalLI\l,canotbeassessed.N1-NoregionalLNmetastasis.N2-RegionalLNmetastasis.Regionallymphnodes(N)Mx-Distantmetastasiscannotbeassessed.Mo-Nodistantmetastasis.M1-Distantmetastasis.
563.Mostcommoncauseofchronic pancreatitis a)Chronicalcoholism b)Tropicalpancreatitis c)Pancreasdivisium d)Gallstonedisease CorrectAnswer-AAnswer-A.Chronicalcoholism"Worldwide,alcoholconsumptionandabuseisassociatedwithchronicpancreatitisinupto70%ofcases"
564.Mostcommoncauseofacuteparotitis- a)StaphylococcusAureus b)StreptococcusPneumonia c)Klebsiella d)StreptococcusViridans CorrectAnswer-AAnswer-A.StaphylococcusAureusMCorganismisStaphylococcusAureus>>streptococcusviridans>>pneumococcus.Acutebacterialparotitis:ismostoftencausedbyabacterialinfectionofStaphylococcusaureusbutmaybecausedbyanycommensalbacteria.UsuallyascendinginfectionStaphylococcusaureusRef-ManipalManualofSurgery5thedition
565.Pre-operativeprophylaxisfor pheochromocytoma a)Alphablockadeafterbetablocker b)Betablockadeafteralphablocker c)Alphablocker d)Betablocker CorrectAnswer-BAnswer-B.BetablockadeafteralphablockerBetablockerssuchaspropanalolatdosesof10to40mgevery6to8hoursareoftenneededinpatientswhohavepersistanttachycardiaandarrhythmias.n-blockersshouldonlybeaddedafteradequatea-blockadeandrehydration.a-adrenergicagonistsmaybeneededinimmediatepostoperativeperiodtopreventpostoperativehypotensionandcardiovascularcollapse.
566.Childwithpolytraumacametocasualty, doseofpackedcell a)10ml/kg b)20ml/kg c)30ml/kg d)40ml/kg CorrectAnswer-AAnswer-AUsualdosageofpackedredcellsinpediatricpopulationsis10-15ml/kg,butcanbeincreasedupto20ml/kgincaseswherehigherincreaseinHCtisneeded,asintrauma.Colloid10ml/kgCrystalloid20ml/kgRequiring>20ml/KgofPRBCsinthe1sthourofresuscitationPediatricMTGPack(50Kg)?4UnitsofPRBCs?2Unitsofthawedplasma?1Unitofapheresisplatelets
567.PolytraumapatientcametoEMS,doseof crystalloidgivenshouldbe a)500mlRLbolusthenregulatedbyindicators b)2000mlbolus c)1000mlbolusthenregulatedbyclinicalindicators d)250mlbolus CorrectAnswer-CAnswer-C.1000mlbolusthenregulatedbyclinicalindicatorsFluidresuscitationbeginswith1000mlbolusofRLforadultand20ml/kgforachild.Responsetotherapyismonitoredbyclinicalindicatorsasbloodpressure,skinperfusion,urinaryoutputandmentalstatus.
568.ApolytraumaCTscan,CTbrainshowsa lesionwithconcavemargin a)EDH b)SDH c)Contusion d)Diffuseaxonalinjury CorrectAnswer-BAnswer-B.SDHSDH-OnheadCT,theclotisbrightormixeddensity,cresentshaped(Lunate),mayhavealessdistinctborderanddoesnotcrossmidline.Contusion-ThecontusedareaappearsbrightonCTScan.
569.Apersoninhaledpeanuttwodaysback& nowunabletocoughitoutwhereisthepossiblelocation a)Rightsuperiorlobe b)Rtmiddlelobe c)Rtlowerlobe d)Ltlowerlobe CorrectAnswer-CAnswer-C.RtlowerlobeThemostcommonanatomiclocationforaforeignbodyistherightmainstembronchusortherightlowerlobe.
570.Herniamostcommonlystrangulatesin a)Indirect b)Direct c)Spigelian d)Incisional CorrectAnswer-AAnswer-A.Indirect"Moststrangulatedherniasareindirectinguinalhernias,however,femoralherniashavethehighestrateofstrangulation(15-20%)ofallhernias".
571.VanNuysprognosticindexisnotbased on: a)Age b)Microcalcification c)Size d)ERstatus CorrectAnswer-DAnswer:D.ERstatusDCISmaybeclassifiedusingtheVanNuyssystem,whichcombinesthe 1. Patient'sage,2. TypeofDCIS3. Presenceofmicrocalcification,4. Theextentofresectionmargin5. Thesizeofthedisease. VanNuysPrognosticIndex Parameter Score1 Score2 Score3 VanNuys Group1 Group2 Group3 Classification Non-high Non-High Highnuclear nucleargrade nucleargrade gradewithor withoutnecrosis withnecrosis withoutnecrosis Margins 10mm 1?9mm Size 16?40mm >40mm Age >60 40?60 <40

572.Commoncauseofchronicpancreatitis a)Chronicalcohol b)Chronicpancreaticcalculi c)pancreasdivisum d)Gallbladderstones CorrectAnswer-AAnswer:A.Chronicalcohol"Worldwide,alcoholconsumptionandabuseisassociatedwithchronicpancreatitisinupto70%ofcases"
573.ThefollowingstatementaboutKeloidis true a)Itcontaingrowthfactor b)Extendedexcisionisthetreatmentofchoice c)Itdonotextendbeyondthewound d)Noneoftheabove CorrectAnswer-AAnswer:A.ItcontaingrowthfactorVaccinations,injections,insectbites,earpiercing,ormayarisespontaneously.Keloidstendtooccur3monthstoyearsaftertheinitialinsult,andevenminorinjuriescanresultinlargelesions.Theyvaryinsizefromafewmillimeterstolarge,pedunculatedlesionswithasofttorubberyorhardconsistency.Althoughtheyprojectabovesurroundingskin,theyrarelyextendintounderlyingsubcutaneoustissues.Certainbodysiteshaveahigherincidenceofkeloidformation,includingtheskinoftheearlobeaswellasthedeltoid,presternal,andupperbackregions.Theyrarelyoccuroneyelids,genitalia,palms,soles,oracrossjoints.Keloidsrarelyinvolutespontaneously,whereassurgicalinterventioncanleadtorecurrence,oftenwithaworseresult.
574.Whichofthefollowinglayersarecut duringfasciotomy? a)Skin b)Skin+subcutaneousfascia c)Skin+subcutaneoustissue+Superficialfascia d)Skin+subcutaneoustissue+Superficialfascia+deepfascia CorrectAnswer-DAnswer:D.Skin+subcutaneoustissue+Superficialfascia+deepfasciaFasciotomyorfasciectomyisasurgicalprocedurewherecompleteopeningofallfascialenvelopeisdonetorelievetensionorpressurecommonlytotreattheresultinglossofcirculationtoanareaoftissueormuscle.Fasciotomyisalimb-savingprocedurewhenusedtotreatacutecompartmentsyndrome.
575.Whichstatementisnottrueregarding crohn'sdisease: a)Rectumisnotinvolved b)Continuouslesionvisualizedinendoscopy c)Noncaseatinggranulomas d)Cobblestoneappearance CorrectAnswer-BAnswer:B.ContinuouslesionvisualizedinendoscopyCrohn'sdiseaseisfrequentlyassociatedwith"skiplesions,"discontinuousareasofactivediseaseinthecolonandsmallintestinewithinterveningsegmentsthatappearnormal.
576.Whichisthebestinvestigationfor carcinomaheadofpancreas: a)Guidedbiopsy b)ERCP c)Transduodenal/transperitonealsampling d)EUS CorrectAnswer-DAnswer:DMedicalimagingtechniques,suchascomputedtomography(CTscan)andendoscopicultrasound(EUS)areusedbothtoconfirmthediagnosisandtohelpdecidewhetherthetumorcanbesurgicallyremoved
577.Anabdominalmassisbest demonstratedincongenitalhypertrophicpyloricstenosisby: a)Inpalpationovertheepigastrium b)Inlefthypochondriac c)Rightiliacfossa d)Duringfeeding CorrectAnswer-DAnswer:D.DuringfeedingCongenitalhypertrophicpyloricstenosisPresentationVomitingisthepresentingsymptom(childvomitsmilkandnobileispresent)Immediatelyaftervomitingthechildishungryi.e.lossofappetitedoesnotoccur.Weightlossisstrikingandrapidlytheinfantbecomesemaciatedanddehydrated.However,greaterawarenessofpyloricstenosishasledtoearlieridentificationofpatientsandhencewithfewerincidencesofchronicmalnutritionandseveredehydration-Nelson10th/1130Thediagnosisisusuallymadewithatestfeed:Inthis,thebabyisfedwiththebottlebyanurseormotherandsurgeon:IHPS/CHPScanbediagnosedclinically.Duringatestfeed,thereisvisiblegastricperistalsispassingfromlefttorightacrosstheupperabdomenandinarelaxedbaby,thepyloric'tumor'ispalpableasan'olive'intherightupperquadrant.Thediagnosiscanbeconfirmedbyanultrasound,whichshows
thethickenedpyloricmuscle.
578.CalculateGCSof25yearoldheadinjury patientwithfollowingparametersconfused,openingeyesinresponsetopain,localizingpainwillbe- a)6 b)11 c)12 d)7 CorrectAnswer-BAnswer:B.11Glasgowcomascale:
579.RETproto-oncogeneisassociatedwith thedevelopmentof a)Medullarycarcinomathyroid b)Astrocytoma c)Paraganglioma d)Hurthlecelltumorthyroid CorrectAnswer-AAnswer:A.MedullarycarcinomathyroidRETproto-oncogeneisagrowthfactorreceptor(receptortyrosinekinase)TheRETproteinisareceptorfortheglialcellline-derivedneurotrophicfactorandstructurallyrelatedproteinsthatpromotecellsurvivalduringneuraldevelopment.RETisnormallyexpressedinthefollowingcellsParafollicularCcellsofthethyroidAdrenalmedullaMedullarycarcinomamayoccurincombinationwithadrenalphaeochromocytomaandhyperparathyroidism(HPT)(usuallyduetohyperplasia)inthesyndromeknownasmultipleendocrineneoplasiatype2A(MEN-2A).Thesearetumorsoftheparafollicular(Ccells)derivedfromtheneuralcrestratherthanthecellsofthethyroidfollicleasareotherprimarythyroidcarcinomasWhenthefamilialformisassociatedwithprominentmucosalneuromasinvolvingthelips,tongueandinneraspectoftheeyelids,withaMarfanoidhabitus,thesyndromeisreferredtoasMENtype2B.RETassociations:MEN2a,MEN2b,Familialmedullarycarcinoma
thyroid.
580.Cutoffforsurgeryinanabdominalaortic aneurysminasymptomaticpatients- a)5.5cm b)6.5cm c)7.5cm d)8.5cm CorrectAnswer-AAnswer:A.5.5cmOperativerepairoftheaneurysmwiththeinsertionofaprostheticgraftorendovascularplacementofaorticstentgraftisindicatedfor:Abdominalaorticaneurysmsofanysizethatareexpandingrapidlyorareassociatedwithsymptoms.Forasymptomaticaneurysms,abdominalaorticaneurysmrepairisindicatedifthediameteris>5.5cm.
581.InACLSwhichdrugcanbegiven followingventricularfibrillationaftercardiacarrestotherthanepinephrine? a)Amiodarone b)Dopamine c)Adenosine d)Atropine CorrectAnswer-AAns.A.AmiodaroneV-FiborVFisthemostcommonrhythmthatoccursimmediatelyaftercardiacarrest.Inthisrhythm,theheartbeatswithrapid,erraticelectricalimpulses.Treatment:Shock/Defibrillation:every2minutesinasingleoneshock,successive,shockableincrements200joules-FollowedbyimmediateCPRfor2minutes/giveandcirculateadrug(s)300joules-FollowedbyimmediateCPRfor2minutes/giveandcirculateadrug(s)360joules-FollowedbyimmediateCPRfor2minutes/giveandcirculateadrug(s)Drugs:GiveEpinephrine1mgofa1:10,000solu,onevery3to5minutes[NoLimit]Giveeither:Amiodarone[ifnotcontraindicated,canbegiven2x]:300mgfirstdose/150mgseconddoseat3to5minutesincrements.
Lidocaine:Firstdose:1mg/kgor1.5mg/kg.Canrepeatitathalftheoriginaldoseuptoatotalof3mg/kg[Secondandremainingdosesaregivenateither0.5mg/kgor0.75mg/kgdependingonyourstar,ngdosage.]
582.A6-year-oldboyexperiencedlife threateningshock,hisCTscanshowedlargeamountofascites,bowelwallthickeningandpoororabsentenhancementofthestrangulatedbowelsegment,showinggangrenousbowelonsurgicalexploration.Trueaboutanastomosisis- a)Shouldbedonebycontinuouslayersasittakeslesstime b)ShouldbeDonewithcatgut c)ShouldbeDonebysinglelayerseromuscularlembertsutures d)ShouldbeDonebySinglelayertakingsubmucosa CorrectAnswer-CAnsC.ShouldbeDonebysinglelayerseromuscularlembertsuturesDiagnosisisofcongenitalIHwithstrangulatedsmallbowelwithgangrenoussmallbowelTransmesentericherniawasthemostcommontypeinolderchildrenaswellasinneonates.IHresultsfromincompleteclosureofsurgicallycreatedmesentericdefects,andusuallyacquiredresultingfrompreviousabdominalsurgeryespeciallyRoux-en-YanastomosisAnastomosisshouldbedonebysinglelayerseromuscularlembertsuturesTheLambertsuturegenerallyisusedinabdominalsurgery.Itisaninvertingsuture,thatcanbeeithercontinuousorinterrupted,usedto
invertingsuture,thatcanbeeithercontinuousorinterrupted,usedtojointwosegmentsofanintestinewithoutenteringthelumen(theinnerchannelthroughwhichstomachcontentsflow).
583.WhichofthefollowingisthebestStent forFemoropoplitealBypass? a)Dacron b)Reversedsaphenous c)PTFE d)None CorrectAnswer-CAnswer:C-PTFE-coveredself-expandingnitinolstentsPTFE-coveredstentsareengineeredwitha30?100-micronporesizetoallowfortheendothelialliningofthestent-graftandvesselhealing.Thetwomaingrafttypesusedforlowerextremitybypassesarethegreatsaphenousveinsandpolytetrafluoroethylene(PTFE)grafts.Oneofthemostwidelyusedstent-graftsinthetreatmentofchroniclowerextremityischemiaistheViabahnendoprosthesis(GoreMedical,Flagstaff,Ariz).Itisconstructedwithexpandedpolytetrafluoroethylene(ePTFE)linerattachedtoanexternalnitinolstent.Theinnersurfaceisbondedwithheparin.
584.Maastrichtclassificationofdonation aftercardiacdeath.Whatcategoryisstage3? a)Awaitingcardiacarrest b)Broughtindead c)Unsuccessfulresuscitation d)Cardiacarrestafterbrain-stemdeath CorrectAnswer-AAnswer:A>AwaitingcardiacarrestThefirstlevelofdefinitionissimpleandbasedonwhetherthesituationisuncontrolled(categoriesIandII)orcontrolled(categoriesIII,IV,andV).CategoryI,thepatientisdeclared"deadonarrival"and,CategoryII,thereisan"unsuccessfulresuscitation"whetheritoccurredoutorinthehospitalforbothsituations.CategoryIIIisthemostusualsituationinwhichthetreatingphysicianandfamilyare"awaitingcardiacarrest"todeclarethedeathofthepatient.CategoryIVisalwayscharacterizedby"cardiacarrestduringbraindeath."ThespecialsituationoftheBelgianlawallowingtheeuthanasiaiselaboratedCategoryV"euthanasia,"andincludespatientswhograntaccesstomedicallyassistedcirculatorydeath.Organdonationaftereuthanasiaisallowedunderthescopeofdonationaftercirculatory
585.ApatientarrivedinERfollowinganRTA withhypotension,respiratorydistressandsubcutaneousemphysemawithnoentryofairononeside.Whatwillbethebestmanagement? a)Needledecompressionin5thintercostalspaceinthe midaxillaryline b)ContinuePPV c)ShifttoICUandincubate d)SecureIVlineandstartfluidresuscitationafterinsertionofthe wide-boreIVline CorrectAnswer-AAns-A.Needledecompressionin5thintercostalspaceinthemidaxillarylineAtensionpneumothoraxdevelopswhena'one-wayvalve'airleakoccurseitherfromthelungorthroughthechestwall.Airissuckedintothethoraciccavitywithoutanymeansofescape,completelycollapsingthencompressingtheaffectedlung.Etiology-Themostcommoncausesarepenetratingchesttrauma,bluntchesttraumawithparenchymallunginjuryandairleakthatdidnotspontaneouslyclose,iatrogeniclunginjury(e.g.duetocentralvenepuncture)andmechanicalpositivepressureventilation.C/F-Thepatientisincreasinglyrestlesswithtachypnoea,dyspnoeaanddistendedneckveins
Treatment-Treatmentconsistsofimmediatedecompression,initiallybyrapidinsertionofalarge-borecannulaintothesecondintercostalspaceinthemidclavicularlineoftheaffectedside,thenfollowedbyinsertionofachesttubethroughthefifthintercostalspaceintheanterioraxillaryline.Ref-BaileyandLove,Shortpracticeofsurgery,27theditionpublishedin2018Pg367,920
586.TreatmentofRenalcellcarcinomaof lessthan4cmwillbe- a)Partialnephrectomy b)Radicalnephrectomy c)Radicalnephrectomy+postoperativeradiotherapy d)Radicalnephrectomy+chemotherapy CorrectAnswer-AAns:A.PartialnephrectomyPartialnephrectomyisnowbeingusedasprimarysurgicaltherapyforpatientswithatumorlessthan4cminsize.EarlierRadicalnephrectomywasthetreatmentofchoicefortumorsofanysize.Classicradicalnephrectomyconsistsofremovalofthekidney,perirenalfat,adrenalgland,andregionallymphnodes.MoststageIandstageIItumors.Partialnephrectomy-isindicatedforpatientswithaT1tumor(accordingtotheUICCTNMstagingsystem)andanormalcontralateralkidney.Interventionshouldbeconsideredforgrowthto>3-4cmorby>O.5cmperyear.
StageT1aN0M0Partialnephrectomyrecommended.Thiscanbedoneviaopen/laparoscopic/roboticproceduresStageT1bN0M0PN(open/laparoscopic/robotic)incaseswheretechnicallyfeasibleLaparoscopicRNshouldbeofferedifaPNisnotfeasibleOpenRNiflaparoscopicsurgeryisnotpossible.StageT2N0M0RN?open/laparoscopic/roboticPN?open/laparoscopic/roboticStageT3RN?open,laparoscopicorrobotic-assistedResectionofvascularthrombosiswhenapplicable(usuallyopen)ResectionofallgrossdiseaseincludinghilarorretroperitonealextensionPNmaybeattemptedinhighlyselectedcasesbyexperiencedsurgeonsRef-BaileyandLove,Shortpracticeofsurgery,27theditionpublishedin2018Pg1420https://www.researchgate.net/publication/263933944_Surgical_management_of_renal_cell_carcinoma_Canadian_Kidney_Cancer_Forum_Consensus
587.WhichisnotseeninAsepsisscore- a)Erythema b)Induration c)Serousdischarge d)Purulentexudate CorrectAnswer-BAnswer-B.Induration Criterion Points Additionaltreatment 0 Antibioticsforwoundinfection 10 Drainageofpusunderlocalanaesthesia 5 Debridementofwoundundergeneral 10 anaesthesiaSerousdischarge Daily0-5 Erythema Daily0-5 Purulentexudate Daily0-10 Separationofdeeptissues Daily0-10 Isolationofbacteriafromwound 10 Stayasinpatientprolongedover14days 5 asresultofoundinfection Ref-BaileyandLove,Shortpracticeofsurgery,27theditionpublishedin2018Pg48
588.Esophagealmanometrywasperformed- itrevealedpanesophagealpressurizationwithdistalcontractileintegrityas>450mmHgpressureinthebody.Whatwillbethediagnosis? a)Type1achalasia b)Type2achalasia c)Type3achalasia d)Jackhammeresophagus CorrectAnswer-CAns:C.Type3achalasiaTypeIachalasia(classicachalasia)-ElevatedmedianIRP(>15mmHg),100%failedperistalsis(DCl<100mmHg.s.cm)PrematurecontractionswithDCIvalues<450mmHg.s.cmsatisfycriteriaforfailedperistalsisTypeIIachalasia(withesophagealcompression)-ElevatedmedianIRP(>15mmHg),100%failedperistalsis,panesophagealpressurisationwith>20%ofswallows.ContractionsmaybemaskedbyoesophagealpressurizationandDCIshouldnotbecalculated.Typelllachalasia(spasticachalasia)ElevatedmedianIRP(>15mmHg),nonormalperistalsis,premature(spastic)contractionswithDCI>450mmHg.s.cmwith>20%ofswallows
Maybemixedwithpanesophagealpressurization'Diffuseesophagealspasm'andjackhammer(nutcracker)esophagus-Spasticpressuresonmanometryof400-500mmHgmarkedhypertrophyofthecircularmuscleandacorkscrewesophagusonbariumswallow.Ref-BaileyandLove,Shortpracticeofsurgery,27theditionpublishedin2018Pg1096,1099
589.Whatwillbetheappropriate managementforAbdominalaorticaneurysm- a)Monitortillsizereaches55mm b)Immediatesurgery c)USGmonitoringtill>70mmasymptomatic d)Notreatment CorrectAnswer-AAnswer-A.Monitortillsizereaches55mmAnasymptomaticabdominalaorticaneurysminanotherwisefitpatientshouldbeconsideredforrepairif>55mmindiameter(measuredbyultrasonography).Regularultrasonographicassessmentisindicatedforasymptomaticaneurysms<55mmindiameter.75%ofaneurysmsaresuitableforendovascular(minimallyinvasive)repair,usuallyviathefemoralarteriesinthegroin.Rupturedabdominalaorticaneurysmisasurgicalemergency.SymptomaticAAA-Anoperationisusuallyindicatedinpatientswhoareotherwisereasonablyfit.Painmaybeawarningsignofstretchingoftheaneurysmsacandimminentrupture;surgeryshouldbeperformedassoonaspossible(usuallyonthenextavailableoperatinglist).Ref-BaileyandLove,Shortpracticeofsurgery,27theditionpublishedin2018Pg961
590.WhatistheTstageofa2.5cmlung carcinoma,notinvolvingthepleura? a)T1a b)T2 c)T1b d)T1c CorrectAnswer-DAns:D.T1c Ref-BaileyandLove,Shortpracticeofsurgery,27thedition
publishedin2018Pg927
591.In,theprimarysurveywhichisnot included- a)CECTtolookforbleeding b)Exposureofthewholebody c)ABC d)RecordingBP CorrectAnswer-AAnswer?A.CECTtolookforbleedingTheprimarysurveyaimstoidentifyandmanagethemostimmediatelylife-threateningpathologiesfirstandfollowscABCDE.1.c-Exsanguinatingexternalhemorrhage2.A-airwaywithcervicalspinecontrol3.B-Breathing&ventilation4.C-Circulation&hemorrhage5.D-Disability6.E-ExposureAnassessmentofthehemodynamicstatusshouldbemadetoidentifyshockedpatients:theskinmaybepale,coolandsweaty,thepulserateraisedtoover100perminuteandthebloodpressurelow.Ref-BaileyandLove,Shortpracticeofsurgery,27theditionpublishedin2018Pg323
592.MClocationofgastrinomainMEN-1 syndrome? a)Duodenum b)Jejunum c)Pancreas d)Ileum CorrectAnswer-AAnswer-A.DuodenumPETsoccurinaround50-60VoofMEN1patients.Themostcommonfunctionaltumourisgastrinomainduodenumfollowedbyinsulinoma.Ref-BaileyandLove,Shortpracticeofsurgery,27theditionpublishedin2018Pg856
593.Patientshaveprecancerouslesionswith abdominalswellingandinguinalnodesareseen.Onexamination,lymphadenopathywasfound.Themostprobablecarcinomarelatedtothisconditionwillbe? a)capenis b)CaTestis c)caprostate d)cabladder CorrectAnswer-AAnswer-A.capenisCancerpenisspreadstoinguinalnodes.Amass,pruritusordischargeiscommon.Advancedtumoursmaycausefecalincontinencebytheinvasionofthesphincters.Analcanaltumoursarepalpableandirregularinduratedtenderulceration.Sphincterinvolvementmaybeevident.Ref-BaileyandLove,Shortpracticeofsurgery,27theditionpublishedin2018Pg1371
594.Amanunderalcoholintoxicationhad fallenintoamanholeandhadaperinealinjurywithswollenscrotumandupperthighalongwithbloodatmeatus.Thepatientishavingdifficultypassingurineaswell.Whatwillbetheinjuryassociatedduetothistrauma? a)Bladderrupture b)Penilefracture c)Bulbarurethra d)Membranousurethra CorrectAnswer-CAnswer-C.BulbarurethraThereisahistoryofablowtotheperineum,usuallyduetoafallastrideinjury.Thebulbarurethraiscrushedupwardsontothepubicbone,typicallywithsignificantbruising.Cyclingaccidents,loosemanholecoversandgymnasiumaccidentsastridethebeamaccountforanumberofcases.C/FThesignsofarupturedbulbarurethraareperinealbruisingandhematoma,typicallywithabutterflydistribution.Thereisusuallybleedingfromtheurethralmeatusandretentionofurineisalsotypicallypresent.Ref-BaileyandLove,Shortpracticeofsurgery,27theditionpublishedin2018Pg1479,1480
595.Patientwithahistoryofcarcinoma bladderpresentingwithdyspnoeawithclinicalsignsofDVTandtachycardia.TheriskforthepatienttodevelopPulmonaryembolismaccordingtoWELL'sscore a)High b)Medium c)Low d)Cannotcommentwithoutd-dimervalues CorrectAnswer-BAns.B.Medium Wellscriteria Score High >6.0 Moderate 2.0to6.0 Low <2.0 ModifiedWellscriteria ScorePElikely >4.0 PEunlikely <or=4.0 ModifiedWellsCriteriaforDVT Variable Score Lowerlimbtraumaorsurgeryorimmobilisationin 1 plastercast
Bedriddenfor>3daysorsurgeryinthelast4weeks 1 Tendernessalongthelineoffemoralorpoplitealveins 1 Entirelimbswollen 1 Calf>3cmlargercircumferencethantheotherside 1 10cmbelowthetibialtuberosity 1 Pittingedema 1 Dilatedcollateralsuperficialveins(notvaricoseveins) 1 PreviousDVT 1 Malignancy(includingtreatmentupto6monthsago) 1 Intravenousdrugabuse 3 AlternativediagnosismorelikelythanDVT -2 Ref-BaileyandLove,Shortpracticeofsurgery,27theditionpublishedin2018Pg988
596.After4monthsofrenaltransplantation,a patientcanlikelytodevelopwhichinfection- a)EBV b)CMV c)Candida d)Histoplasma CorrectAnswer-BAnswer-B.CMVCausesofallograftdysfunction1.EarlyPrimarynon-function(irreversibleischaemicdamage)Delayedfunction(reversibleischaemicinjury)HyperacuteandacuterejectionArterialorvenousthrombosisofthegraftvesselsDrugtoxicity(e.g.calcineurininhibitortoxicity)Infection(e.9.cytomegalovirusdiseaseingraftMechanicalobstruction(ureter/commonbileduct)2.LateChronicrejectionArterialstenosisRecurrenceoforiginaldiseaseinthegraft(glomerulonephritis,hepatitisC)Mechanicalobstruction(ureter,commonbileduct)Ref-BaileyandLove,Shortpracticeofsurgery,27theditionpublishedin2018Pg1551
597.Thepatientispresentwithfecal peritonitisandduringlaparotomy,adiverticularperforationisseen.WhichstageisclassifiedaccordingtoHinchey'sstage? a)1 b)2 c)3 d)4 CorrectAnswer-DAnswer-D.4Hincheyclassificationofcomplicateddiverticulitis. 1. GradeIMesentericorpericolicabscess2. GradellPelvicabscess3. GradelllPurulentperitonitis4. GradelVFaecalperitonitis Ref-BaileyandLove,Shortpracticeofsurgery,27theditionpublishedin2018Pg1274
598.Ayoungmanmetwithamotorbike accidentandhadinjuriestoileumandjejunum.Thereforetheentireileumandpartialjejunumwereresected.Whichofthefollowingwouldthepatientsufferfrom a)VitaminB12deficiency b)Atrophicgastritis c)Constipation d)None CorrectAnswer-AAnswer-A.VitaminB12deficiencyTheileumistheonlysiteofabsorptionofvitaminB12andbilesalts.Bilesaltsareessentialfortheabsorptionoffatsandfat-solublevitamins.Followingresectionoftheileum,thelossofbilesaltsincreasesandisnotmetbyanincreaseinsynthesis.Thejejunumistheprincipalsitefordigestionandabsorptionoffluid,electrolytes,iron,folate,fat,proteinandcarbohydrate,buttheabsorptionofbilesaltsandvitaminB12onlyoccursintheterminalileum,wheretherearespecifictransporters.Ref-BaileyandLove,Shortpracticeofsurgery,27theditionpublishedin2018Pg283,1241
599.Apatientafteraheavymealcomeswith epigastricpain.Onexaminationtendernessandrigidityintheupperabdomen.X-rayshowingpneumomediastinum.Whatcanbethecause- a)Spontaneousesophagealrupture b)Penetratingforeignbodyinjurytoesophagus c)Perforatedpepticulcer d)Ruptureofemphysematousbulla CorrectAnswer-AAnswer-A.SpontaneousesophagealruptureThisisaclinicalcaseofspontaneousesophagealrupture/Boerhaave'ssyndromeTheclinicalhistoryisusuallyofseverepaininthechestorupperabdomenafteramealoraboutofdrinking.Mackler'striad,whichconsistsof(1)vomitingfollowedby(2)chestpainand(3)subcutaneousemphysemaduetoanesophagealrupture.Achestradiographisoftenconfirmedwithairinthemediastinum,pleuraorperitoneum.InvestigationsChestX-ray--showspneumomdiastinum(`V'signofNaclerio).MRI/CTchest.Totalcount.Ref-BaileyandLove,Shortpracticeofsurgery,27thedition
publishedin2018Pg1073SriramBhatM,SRB'sManualofSurgery,4theditionpublishedin2013,Pg856
600.Transplantationbetweenidenticaltwins- a)Isograft b)Allograft c)Autograft d)Xenograft CorrectAnswer-AAnswer-A.IsograftGraft:Itisthetransferoftissuefromoneareatoanotherwithoutitsbloodsupplyornervesupply.Autograft:Itistissuetransferredfromonelocationtoanotheronthesamepatient.Isograft:Itistissuetransferbetweentwogeneticallyidenticalindividuals,i.e.betweentwoidenticaltwins.Allograft:Itistissuetransferbetweentwogeneticallydifferentmembers,e.g.kidneytransplantation(Humantohuman)(Homograft).Xenograft:Itistissuetransferfromadonorofonespeciestoarecipientofanotherspecies(Heterograft).
601.Thoracoscore,whatisnotthe component a)ASACLASSIFICATIONS b)SURGERYPRIORITIES c)Performancestatus d)Expectedcomplicationspost-surgery CorrectAnswer-BAnswer-B.SURGERYPRIORITIESTheThoracoscoreisthemostwidelyusedmodeltoassesstheriskofoperativemortalityinthoracicpatients
Ref-BaileyandLove,Shortpracticeofsurgery,27theditionpublishedin2018Pg915
602.Retrosternalgoiterwhichistrue? a)AllpatientsshouldundergoCTchest b)Allpatientsrequireamediansternotomy c)itreceivesbloodsupplyfromthoracicvessels d)Majorityofretrosternalgoitresshouldbeoperatedimmediately CorrectAnswer-AAnswer-A.AllpatientsshouldundergoCTchestRetrosternalgoitretendstoarisefromtheslowgrowthofamultinodularglanddownintothemediastinum.Itgetsitsbloodsupplyfromthemediastinumitself,notfromtheneck.InvestigationsAchestX-rayshowsasofttissueshadowunderthesternum.Aradioactiveiodinestudyisdiagnostic.CTscan/MRIisusefulandallpatientsshouldundergoit.Surgicalremovalofretrosternalthyroidisdone.(TOC)Commonlyitcanberemovedthroughanincisionintheneck.Ref-BaileyandLove,Shortpracticeofsurgery,27theditionpublishedin2018Pg810SriramBhatM,SRB'sManualofSurgery,4theditionpublishedin2013,Pg498
603.TrueforKing'sCriteriawithacute fulminantliverfailureexcept- a)Age b)Jaundice<7days c)Serumbilirubin>17.5mg/dl d)INR>3.5 CorrectAnswer-BAnswer-B.Jaundice<7daysKing'sCollegecriteriaforlivertransplantationinAHF.APAP-associatedAHF-pH?<?7.3orINR?>?6.5,serumcreatinine>3.4mg/dl,andgradeIII-IVencephalopathyAllothercausesofAHFINR>6.5OrThreeofthefollowingvariables:1.Age40years2.ThecauseisnonA,nonBhepatitisoridiosyncraticdrugreaction3.Durationofjaundicebeforeencephalopathy>7days4.INR?>?3.55.Serumbilirubin>17.5mg/dlAPAP,acetaminophen;INR,internationalnormalizedratio.
604.Apatientwhowaspostedforelective inguinalherniasurgeryhashistoryMIforwhichheunderwentCABG.Whatwillyoudoinpreoperativeassessment? a)History+c/e+routinelabs+angiographytolookforstent patency b)History+c/e+routinelabs c)History+c/e+routinelabs+stresstest d)History+c/e+routinelabs+V/Qscan CorrectAnswer-CAnswer-C.History+c/e+routinelabs+stresstestInthegivenscenariothepatienthadundergoneCABGnotanangioplasty,sothefirstoptionisruledout.Withroutinelabtests,thestresstestshouldbeincludedtocheckthecardiacfunctionaspreoperativeassessment
605.Historyoftraumawithastabinjuryto therightlowerchestwithlowBPandlowpulserate.ItcanbeimprovedwithIVfluidsandafterresuscitationinthetraumacenterpatient'sBPbecomesnormal.AchestX-rayshowedclearlungfields.Whatwillbethenextstep- a)EFAST b)Keepimmediatechesttube c)CECTabdomen d)CECTchest CorrectAnswer-AAnswer-A.EFASTInvestigationsaredrivenbythecardiovascularstatusofthepatient.Intorsotrauma,thebestandmostsensitivemodalityisaCTscanwithintravenouscontrastforbluntinjuryifthepatientisstable.Routineinvestigationintheemergencydepartmentofinjurytothechestisbasedonclinicalexamination,supplementedbychestradiography.Ultrasoundcanbeusedtodifferentiatebetweencontusionandtheactualpresenceofblood.Extendedfocusedassessmentwithsonarfortrauma(eFAST)isbecomingthemostcommoninvestigation.Ref-BaileyandLove,Shortpracticeofsurgery,27theditionpublishedin2018Pg366,372
606.Inapatientwithparathyroidadenoma, howdoweconfirmtheremovalofthecorrectglandaftersurgery? a)50%reductioninPTHafter10mins b)50%reductioninPTHafter5mins c)25%reductioninPTHafter10mins d)25%reductioninPTHafter5mins CorrectAnswer-AAnswer-A.50%reductioninPTHafter10minsMiamicriteriaTheIOPTHvaluesatbaseline(pre-incisionandpreexcision)andat5and10minutesafterparathyroidectomywerereviewedaccordingtotheMiamicriterion(>50%dropfromhighestbaselineIOPTHlevelat10minutesafterexcision),criterion1(>50%dropfrompre-incisionIOPTHlevelat10minutes),criterion2(>50%dropfromthehighestbaselineIOPTHlevelat10minutesandfinalIOPTHlevelwithinthereferencerange),criterion3(>50%dropfromthehighestbaselineIOPTHlevelat10minutesandfinalIOPTHlevellessthanthepre-incisionvalue),criterion4(>50%dropfromthehighestbaselineIOPTHlevelat5minutes),andcriterion5(>50%dropfrompreexcisionIOPTHlevelat10minutes).Ref-https://www.ncbi.nlm.nih.gov/pubmed/16702520
607.Flapcommonlyusedinbreast reconstructionis? a)DIEPbasedondeepinferiorepigastricvessels b)TRAMbasedonsuperiorglutealvessels c)Glutealflapbasedonthoracodorsalartery d)Latissimusdorsiflapbasedontheinferiorepigastricartery CorrectAnswer-AAnswer-A.DIEPbasedondeepinferiorepigastricvesselsDIEPflap(basedondeepinferiorepigastricvessels)TRAMflapbasedonItiseithersuperiorpediclebasedonthesuperiorepigastricvesselsorinferiorpediclebasedontheinferiorepigastricvessels.Latissimusdorsimuscle/myocutaneousflapItisbasedonthethoracodorsalartery,abranchofthesubscapulararterySuperiorglutealflapbasedonsuperiorglutealvessels.Ref-BaileyandLove,Shortpracticeofsurgery,27theditionpublishedin2018Pg879SriramBhatM,SRB'sManualofSurgery,4theditionpublishedin2013,Pg350,594
608.A30-year-oldmanispresentedwith crampingglutealpainafterwalking500m.Whichisthevesselinvolvedduringthis? a)Arterialdiseasewithaortoiliacinvolvement b)Arterialdiseasewithfemoralarteryinvolvement c)Femoralvenousinsufficiency d)None CorrectAnswer-AAnswer-A.ArterialdiseasewithaortoiliacinvolvementThemusclegroupaffectedbyclaudicationisclassicallyoneanatomicallevelbelowthelevelofarterialdiseaseandisusuallyfeltinthecalfbecausethesuperficialfemoralarteryisthemostcommonlyaffectedartery(70%ofcases).Aorto-iliacdisease(30%ofcases)maycausethighorbuttockclaudication.ButtockclaudicationinassociationwithsexualimpotenceresultingfromarterialinsufficiencyiseponymouslycalledLeriche'ssyndrome.Itisveryrare.Ref-BaileyandLove,Shortpracticeofsurgery,27theditionpublishedin2018Pg943
609.Whichisthemostcommonpancreatic endocrineneoplasm? a)Insulinoma b)Gastrinoma c)VIPoma d)Glucagonoma CorrectAnswer-AAnswer-A.InsulinomaInsulinoma(70-80%)>Nonfunctionaltumours(30-35%)>gastrinoma(20-25%)
610.Anelderlymanwithalong-standing moleoverthefacewhichisincreasinginsizeandshowingirregularborders.Whatwillbethediagnosis? a)Superficialspreadingmelanoma b)Lentigomaligna c)Acralmelanoma d)Nodularmelanoma CorrectAnswer-AAnswer-A.SuperficialspreadingmelanomaSuperficialspreadingmelanomaThisisthemostcommonpresentation(70%)usuallyarisinginapre-existentnaevusafterseveralyearsofslowchange,followedbyrapidgrowthintheprecedingmonthsbeforepresentation.Nodularmelanoma(NM)Nodularmelanomaaccountsforl5%olallMM.Typicallyappearasblue/blackpapules,l-2cmindiameter,andbecausetheylackthehorizontalgrowthphase.Lentigomalignamelanoma-Presentsasaslow-growing,variegatedbrownmaculeontheface,neckorhandsoftheelderly.Acrallentiginousmelanoma(ALM)ALMaffectsthesolesoffeetandpalmsofhands.Itusuallypresentsasaflat,irregularmaculeinlaterlife.Ref-BaileyandLove,Shortpracticeofsurgery,27theditionpublishedin2018Pg610

This post was last modified on 23 November 2021